Вы находитесь на странице: 1из 257

www.BankExamsToday.

com

Reasoning Made
Easy
By Ramandeep Singh

REASONING MADE EASY www.BankExamsToday.com

Index
Chapter 1
Chapter 2
Chapter 3
Chapter 4
Chapter 5
Chapter 6
Chapter 7
Chapter 8
Chapter 9
Chapter 10
Chapter 11
Chapter 12
Chapter 13
Chapter 14
Chapter 15
Chapter 16
Chapter 17
Chapter 18
Chapter 19
Chapter 20
Chapter 21
Chapter 22

Puzzles
Input Output
Data Sufficiency
Sitting Arrangements
Syllogism
Blood Directions
Analogy
Coded Inequality
Coding Decoding
Decision Making
Direction Sense
Letter and Number Series
Logical Arrangement
Mathematical Operations
Odd Man Out Series
Ranking Based problems
Arithmatic Reasoning
Classification
Dice Problem
Statement and Arguments
Statement and Assumptions
Statement and Conclusions

REASONING MADE EASY www.BankExamsToday.com

Chapter-1
Puzzles
Directions (Q. 1-2) Study the following information carefully and answer the
question given below:
In one floor of a building there are eight flats A, B, C, D, E, F, G and H in two
rows. One row of flats faces north and the other row faces south. D is exactly
opposite of F which faces North. Neither A nor C faces South. A is not at any ends
and is second to the left of F. E is exactly opposite of B which faces South. G is
third to the right of D.
1.

Which of the following pairs of flats facing south are at the two ends?
a) GD
b) GH
c) BH
d) Cannot be determined
e) None of these

2.

Which of the following pairs of flats facing North are at the two ends?
a) EA
b) EC
c) CF
d) Cannot be determined
e) None of these

Directions (Q. 3-4) Study the following information to answer the given
questions:
In a five letter English word (which may or may not be a meaningful
English word), there are two letters between L and P. S is not placed immediately
next to L. There is only one letter between S and A. S is towards the right of A. S is
not placed immediately next to E.
3.

Which of the following is correct with respect to the word thus formed?
a) E is at one of the extreme ends of the word
b) P is not placed immediately next to A
c) There are two letters between A and E in the word thus formed
d) P is placed second to the right of E
e) None of correct

4.
Which of the following words will be formed based on the given
conditions?
a) SPAEL
b) PEALS
c) LEAPS
d) SEPAL
e) LAPSE
Directions (Q. 5-9) Study the following information and answer the given
questions.

REASONING MADE EASY www.BankExamsToday.com


Eight chemicals A, B, C, D, E, F, G and H, contained in eight different
bottles, are placed around a circular table in such a manner that the tap fixed to each
bottle is directed outward from the centre of the table. Each chemical is opf a
different colour, viz Blue, Yellow, Orange, White, Green, Violet, Brown and Black,
but not necessarily in the same order.
i)
ii)
iii)
iv)
v)
vi)
vii)
viii)
ix)
x)
xi)

Chemical B is placed third to the right of Chemical D.


The Yellow chemical is placed on the immediate left of chemical B.
The colour of chemical B is not white.
Chemical F is placed fourth to the left of chemical A.
Neither chemical F nor chemical A is an immediate neighbour of chemical
B.
The colour of chemical C is violet and is placed third to the left of the
Yellow chemical.
The Green chemical is placed second to the right of the violet chemical.
The Brown chemical is placed second to the right of chemical B.
H is the Orange chemical and is placed exactly between chemical C and
chemical A.
The Blue chemical is placed second to the left of the Orange chemical.
Chemical G is placed third to the right of chemical C.

5.

Which of the following chemicals is of White colour?


a) D
b) E
c) G
d) F
e) None of these

6.

What is Hs position
a) Third to the left
b) Second to the left
c) Third to the right
d) Second to the right
e) Fourth to the left

7.

How many chemical bottles are placed between chemical A and G (counted
from G clockwise)?
a) One
b) Two
c) Three
d) Four
e) None of these

8.

Four of the following five are alike in a certain way and hence from a
group. Which of the folowing does not belong to that group?
a) Violet-Brown
b) Black-Green
c) Orange-White
d) Yellow-Blue
e) Green -Yellow

9.

What is the colour of chemical B?


a) Blue
b) Black
d) Cant say
e) None of these

with respect to F?

c) Yellow

REASONING MADE EASY www.BankExamsToday.com

Directions (Q. 10-14) Study the following information carefully and answer the
question given below:
P, Q, R, S, T, V and W are seven friends working in a call centre. Each of
them has different day offs in a week from Monday to Sunday not necessarily in the
same order. They work in three different shifts I, II and III with at least two of them
in each shift.
R works in shift II and his day off is not Sunday. Ps day off is Tuesday and
he does not work in the same shift with either Q or W. None of those who work in
shift I has day off either on Wednesday or on Friday. V workds with only T in shift
III. Ss day off is Sunday. Vs day off is immediate next day of that of Rs day off.
Ts day off is not on Wednesday. Ws day off is not on the previous day of Ps day
off. S works in shift I. Q does not work in the same shift with R and his day off is
not on Thursday.
10.

11.

Which of the following is Ws day off?


a) Tuesday
b) Monday
d) Data inadequate
e) None of these

c) Saturday

Which of the following is Rs day off?


a) Friday
b) Thursday
d) Wednesday
e) None of these

c) Tuesday

12.

Which of the following groups of friends work in shift II?


a) RP
b) RV
c) QWS
d) Data inadequate
e) None of these

13.

Which of the following is Qs day off?


a) Friday
b) Wednesday
d) Monday
e) None of these

14.

c) Thursday

Which of the following groups of friends work in shift I?


a) RV
b) RP
c) QWS
d) Data inadequate
e) None of these

Directions (Q. 15-18) Read the following passage carefully and answer the
question given below it.
Six friends Abhishek, Deepak, Mridul, Pritam, Ranjan and Salil married
within a year in the months of February, April, July, September, November and
December and in the cities of Ahmedabad, Bengaluru, Chennai, Delhi, Mumbai and
Kolkata but not necessarily following the above order. The brides names were
5

REASONING MADE EASY www.BankExamsToday.com


Geetika, Jasmine, Hema, Brinda, Ipsita and veena, once again not following any
order. The following are some facts about their weddings.
(i)
(ii)
(iii)
(iv)
(v)
(vi)
(vii)
15.

16.

Mriduls wedding took place in Chennai; however he was not m arried


to Geetika or Veena.
Abhisheks wedding took place in Ahmedabad and ranjans in Delhi
however neither of them was married to Jasmine or Brinda.
The wedding in Kolkata took place in February.
Hemas wedding took place in April, but not in Ahmedabad.
Geetika and Ipsita got married in February and November and in
Chennai and Kolkata but not following the above order.
Pritam visited Bengaluru and Kolkata only after his marriage in
December.
Salil was married to Jasmine to September.

Hemas husband is
a) Abhishek
d) Pritam

b) Deepak
e) Mridul

Deepaks wedding took place in


a) Bengaluru
b) Mumbai
d) Delhi
e) Chennai

c) Ranjan

c) Kolkata

17.

In Mumbai, the wedding of one of the friends took place in the month of
a) April
b) September
c) November
d) December
e) July

18.

Salils wedding was held in


a) Bengaluru
b) Chennai
d) Delhi
e) Mumbai

c) Kolkata

Directions (Q. 19-23) Read the following information carefully to answer the
given questions.
There are five married couples in a family and there is a child to every
couple. Ages of children are 3, 4, 5, 6 and 9 years. Name of males are M, N, O, P
and Q. Name of children are A, B, C, D and E. Name of females are V, W, X, Y
and Z. Name of Ms child is not C and E and he is not eldest or youngest. Age of
Xs child is 6 years and her husband is one among N, O and Q. Age of D is 3 years
but she is not a child of O. As age is multiple of 3 but she is not a child of M and
V. Ws husband is O or Q. Age of Zs child is 5 years but the name of child is not B
or E. V is wife of O.
19.

What is the age of B.


a) 4
d) 9

b) 6
e) None of these

c) 5
6

REASONING MADE EASY www.BankExamsToday.com

20.

What is the name of Ms wife?


a) W
b) X
d) Z
e) None of these

21.

Name of Mother, farther and age of E?


a) X, N and 6
b) V, O and 9
c) X, O and 6
d) V, N and 9
e) Cant be determined

22.

What is the name and the age of Ps child?


a) B and 6
b) E and 9
c) C and 4
d) C and 5
e) None of these
What are the names of Qs wife and his child?
a) W and D
b) X and A
c) W and C
d) X and D
e) None of these

23.

c) Y

Directions (Q. 24-27) Read the following information carefully and answer the
questions which follow.
Five friends A, B, C, D, and E are working in 5 different departments M, N,
O, P and Q and they earn different salaries i.e. 10,000, 15,000, 20,000, 25,000 and
30,000 and they all are of different ages i.e. 24, 26, 28, 30 and 32 years. These all
informations are not necessarily in the same order.
B works in department M and earns more than 20,000. Person who is 28
years old works in department Q. 32 years old person earns at least 20,000.
The person who is 26 years old earns 25,000. A earns 15,000, but does not
work in department N or P.
Person who is 30 years old earns highest salary but does not work in department M
and N. E does not work in department P or Q, and his age is not 32. The salary of D
is less than 20,000.
24.

Who works in department N?


a) B
b) C
c) D
d) E
e) Cant be determined

REASONING MADE EASY www.BankExamsToday.com


25.

If the name of the person represents its salary then which of the following is
true?
a) A + B = C
b) C + D = B
c) D + E = B
d) A + D = E
e) None of these

26.

Which of the following combination is definitely true?


a) B-30 years-M-30,000
b) D-24 years-N-10,000
c) A-24 years-P-15,000
d) E-30 years-O-30,000
e) None of these

27.

The person whose age is 30 works in which department?


a) M
b) N
c) O
d) P
e) None of these

Directions (Q. 28-29) Read the following information carefully and answer the
questions which follow:
Education is the most important investment one makes in life. Higher
studies and specialization in certain fields call for additional financial support from
time-to-time.
28.

Which of the following can be inferred from the given information? (An
inference is something that is not directly stated but can be inferred from
the given information)
a) People generally lack finances to pay for higher education
b) The rich and wealthy sections of our society are unwilling to spend money on the
education of their children
c) Higher education requires higher amount of money than primary education
d) Now-a-days even the poor and needy desire to educate their children
e) None of these
29.

Which of the following statements is an assumption implicit in the above


given proposition? (An assumption is something supposed or taken for
granted)
a) Higher education need not much finances as it is subsidized by the government
b) Education is one of the most essential parts of ones life
c) Education is mostly neglected in our society
d) Specialization in education is most sought after thing today
e) None of these

REASONING MADE EASY www.BankExamsToday.com


Directions (Q. 30-34) Read the following information carefully and answer the
questions given below.
Six executives Akash, Bipasha, Deepak, Jyoti, Kartik and Priya have to
advertise four products, i.e. Soap, watches, computers and chocolates on 3 different
channels i.e. go, one and spice either alone or in pairs. An executive can visit only
one channel and advertise only one product. Not more than two executives can
advertise on a channel.
(i) Bipasha and Jyoti both visit the same channel but advertise different product
(ii) Akash who visits Go advertises neither soap nor computers
(iii) Kartik does not advertise chocolates
(iv) No girl advertises soap
(v) The two executives who advertise chocolates visit spice
30.

31.

Who advertises watches?


a) Deepak
b) Kartik
d) Priya
e) None of these

c) Akash

Which channel does Kartik visit?


a) Go
b) Spice
d) Cannot be determined
e) None of these

c) One

32.

Which of the following channel product pairs is definitely incorrect?


a) Go-Watch
b) One-Computer
c) Go-Soap
d) One-Soap
e) One-Watch

33.

What will Jyoti advertise?


a) Chocolates
b) Watches
d) Cant be determined e) None of these

34.

c) Computers or watches

If Bipasha advertises computers which of the following must be true?


a) Jyoti advertises soap
b) Jyoti advertises watches
c) Kartik advertises computers
d) Kartik works for spice
e) None of these

Directions (Q. 35-40) Study the following information carefully and answer the
following questions.
An organisation decided to distribute seven types of laptops on week days
starting from Monday to Sunday. The seven laptops to be distributed are of Sony,
Acer, Asus, Lenovo, Dell, HP and HCL but non necessarily in the same order. Dell
laptops are to be given on Saturday. Lenovo laptops are given just after Asus
laptops and Asus laptops are not given on Wednesday. HP laptops are given two
days after the distribution of Asus laptop. Acer laptops are distributed just after
9

REASONING MADE EASY www.BankExamsToday.com


Sony laptop. HCL laptops are distributed before Acer laptops. HP laptops are
distributed after Dell laptops.
35.

On which day of the week Lenovo laptops are distributed?


a) Tuesday
b) Wednesday
c) Friday
d) Sunday
e) None of these

36.

Which laptops are distributed just between the days when Lenovo and HP
are distributed?
a) Asus
b) Dell
c) Sony
d) HCL
e) None of these

37.

Which laptops are distributed on Wednesday?


a) Acer
b) Lenovo
d) HP
e) None of these

c) HCL

38.

On which day of the week Asus are distributed?


a) Monday
b) Sunday
c) Tuesday
d) Wednesday
e) None of these

39.

If HCL is related with Tuesday, Sony is related with Wednesday; similarly


Lenovo is related with which day?
a) Thursday
b) Friday
c) Saturday
d) Sunday
e) None of these

40.

Which laptops are distributed in the middle of the week?


a) Acer
b) Asus
c) Lenovo
d) HCL
e) None of these

Directions (Q. 41-42) In the following questions below is given a statement


followed by two courses of action I and II. On the basis of the information given in
the statement, you have to assume everything in the statement to be true, then
decide which of the suggested courses of action logically follows for pursuing.
Give Answer:
a) Only I follow
b) Only II follows
c) Either I or II follows
d) Neither I nor II follows
e) Both I and II follows
41.

Statement:
Despite good economic progress of country, significant
number of under nourished children has been observed in the rural parts of
the country.
Courses of action
10

REASONING MADE EASY www.BankExamsToday.com


I. Government should increase wealth tax and income tax and use that
money for up-liftment of the deprived class
II. Government should introduce schemes like free meals in primary
schools and make primary education compulsory
42.

Statement:
The police department has come under a cloth with recent
revelations that atleast two senior police officials are suspected to have been
involved in the illegal sale of a large quantity of weapons from the state
police amoury.
Courses of action
I. A through investigation should be ordered by the State Government to
bring out all those who are involved in the illegal sales of arm.
II. State police armoury should be kept under Central Governments
control.

Directions (Q. 43-47) Study the following information and answer the following
questions
A, B, C, D, E, G and I are seven friends who study in three different
standards namely 5th, 6th and 7th such that not less than two friends study in the same
standard. Each friend also has a different favourite subject namely History, Civics,
English, Marathi, Hindi, Maths and Economics but not necessarily in the same
order.
A likes Maths and studies in the 5th standard with only one other
friend who likes Marathi. I studies with two other friends. Both the friends who
study with I like languages (Here, languages include only Hindi, Marathi and
English)
D studies in the 6th standard with only one person and does not like Civics.
E studies with only one friend. The one who likes history does not study in 5th or 6th
standard. E does not like languages. C does not like English, Hindi or Civics.
43. Which combination represents Es favourite subject and the standard in which
he studies?
a) Civic s and 7th
b) Economics and 5th
c) Civics and 6th
d) History and 7th
e) Economics and 7th
44.

Which of the following is Is favourite subject?


a) History
b) Civics
c) Marathi
d) Either English or Marathi
e) Either English or Hindi

45.

Who amongst the following studeis in the 7th standard?


a) G
b) C
c) E
d) D
e) None of these
11

REASONING MADE EASY www.BankExamsToday.com

46.

Which of the following combinations is definitely correct?


a) I and Hindi
b) G and English
c) C and Marathi
d) B and Hindi
e) E and Economics

47.

Which of the following subjects does G like?


a) Either Maths or Marathi
b) Either Hindi or English
c) Either Hindi or Civics
d) Either Hindi or Marathi
e) Either Civics or Economics

Directions (Q. 48-50) Read the following information carefully and answer the
given questions.
Despite repeated announcements that mobile phones were not allowed in
the examination hall, three stuidents were caught with their mobile phones.
A.
Mobile phones nowadays have a lot of features and it is easy to cheat with
their help.
B.
The invigilator must immediately confiscate the mobile phones and ask the
students to leave the exam hall immediately.
C.
Mobile phones are very expensive and leaving them in bags outside the
exam hall is not safe.
D.
There have been incidents where students who left the exam hall early stole
the mobile phones kept in the bags of the students who were writing the
exam.
E.
The school authorities must ask the students to leave their phones in the
custody of the invigilator before the exam in order to avoid thefts of mobile
phones.
F.
None of the other students were carrying their phones in the exam hall.
48.

Which of the following among A, B, C, and D may be a strong arguments


in favour of the three s tudents who were caught with the mobile phone?
a) Only A
b) A and B
c) C and D
d) Only C
e) B and D

49.

Which of the following among A, B, E and F may be the reason behind the
school making such announcements before the exam?
a) Only B
b) D and E
c) Only F
d) Only A
e) None of these

50.

Which of the following among A, B, D and F can be an immediate course


of action for the invigilator?
a) Only B
b) A and D
c) Only A
d) D and F
e) Only F
12

REASONING MADE EASY www.BankExamsToday.com


Directions (Q. 51-55) In these questions, a relationship between different
elements is shown in the statements. The statements are followed by two
conclusions. Give answer
a)
b)
c)
d)
e)

If only conclusion I is true.


If only conclusion II is true.
If either conclusion I or II is true.
If neither conclusion I nor II is true.
If both conclusions I and II are true.

(51-52):
Statement:

W>H=I>C=L<E

51.

Conclusions:

I. W > L

II. E > H

52.

Conclusions:

I. E < W

II. I > L

53.

Statement:
Conclusions:

54.

Statement:
Conclusions:

55.

P=R>E<S=N>T
I. P > T

II. N > R
C > B > L, Q = E > P = C

I. Q > B

II. L < E

Statement:

D<H=J<K>P>R

Conclusions:

I. D = K

II. K > D

Directions (Q. 56-60) Study the following information carefully and answer the
given questions:
A man has six sons U, V, W, X, Y and Z, who stay in four metro cities.
They work in different companies, viz Infosys, Tech Mahindra, Tata, Nike,
Walmart and Titan but not necessarily in the same order. Z stays in Mumbai and
works with Nike. Those who stay in Kilkata work with Tech Mahindra and
Walmart. One of the two Mumbaikars works with Titan. The one who works with
Infosys lives in Chennai. Z does not live in Chennai. U works with Tata. V works
with Tech Mahindra. Y works with Walmart.
56.

Who amoing the following works with Infosys?


a) U
b) W
c) X
d) Y
e) None of these

57.

Which of the following pairs lives in Kolkata?


13

REASONING MADE EASY www.BankExamsToday.com


a) XY
d) WU
58.

59.

60.

b) ZX
e) None of these

c) VY

Z works with which of the following companies?


a) Walmart
b) Titan
d) Cannot be determined
e) None of these

c) Tata

In which of the following metros does W live?


a) Delhi
b) Chennai
d) Cannot be determined
e) None of these

c) Kolkata

Which of the following combinations is true?


a) V Mumbai Walmart
b) Z Kolkata Walmart
c) Y Delhi Titan
d) W Chennai- Infosys
e) None of these

Directions (Q. 61-65) Study the following information carefully and give the
answers.
Sixteen teams have been invited to participate in the ABC Gold Cup cricket
tournament. The tournament is conducted in two stages. In the first stage, the teams
are divided in to two groups. Each group consists of eight teams, with each team
playing every other team in its group exactly once. At the end of the first stage, the
top four teams from each group advance to the second stage while the rest are
eliminated. The second stage comprises of several rounds. A round involves one
match for each team. The winner of a match in a round advances to the next round;
while the loser is eliminated the team that remains undefeated in the second stage is
declared the winner and claims the gold cup.
The tournament rules are such that each match results in a winner and a
loser with no possibility of a tie. In the first stage, a team earns one point for each
win and no points for a loss. At the end of the first stage, teams in each group are
ranked on the basis of total points to determine the qualifiers advancing to the next
stage. Ties are resolved by a series of complex tie- breaking rules so that exactly
four teams from each group advance to the next stage.
61.

What is the total number of matches played in the tournament?


a) 28
b) 55
c) 63
d) 35
e) None of these

62.

The minimum number of wins needed for a team in the first stage to
guarantee its advancement to the next stage is:
a) 5
b) 6
c) 7
d) 4
e) None of these
14

REASONING MADE EASY www.BankExamsToday.com


63.

What is the highest number of wins for a team in the first stage in spite of
which it would be eliminated at the end of first stage?
a) 1
b) 2
c) 3
d) 4
e) None of these

64.

What is the number of rounds in the second stage of tournament?


a) 1
b) 2
c) 3
d) 4
e) None of these

65.

Which of the following statements is true?


a) The winner will have more wins than any other team in the tournament.
b) At the end of the first stage, no team eliminated from the tournament will
have more wins than any of the team qualifying for the second stage.
c) It is possible that the winner will have the same number of wins in the
entire tournament as a team eliminated at the end of the first stage.
d) The number of tems with exactly one win in the second stage of the
tournament is 4.
e) None of these

Directions (Q. 66-70) Study the following information carefully and answer the
questions given below.
Five friends P, Q, R, S and T are Musician, Architect, Doctor, Engineer and
Artist by profssion and like White, Blue, Red, Yellow and Green colour but not
necessarily in that order.

66.

The person whose hobby is dancing preferred lemonade to cola while others
preferred cola to lemonade in beverages.
The four friends who took cola were P, the one who is an Engineer, the
person whose favourite colour is Green and t he o ne whose hobby is net
surfing.
S did not take lemonade and his favourite colour is White.
Qs favourite colour is Blue. He did not like lemonade.
Ts hobby is not painting, reading or gardening.
S clicks a picture of his friend who is an Engineer.
The person whose favourite colour is Red likes painting and the person who
is artist like gardening.
S is not a doctor. The person who is a doctor takes cola. The person who is
an Engineer likes Blue colour.
The musicians favourite colour is not yellow. Rs favourite colour is
Green.
Who among the following is a Doctor?
a) R
b) P
d) Cant say
e) None of these

c) S
15

REASONING MADE EASY www.BankExamsToday.com

67.

68.

69.

70.

Qs hobby is
a) Reading
d) Cant say

b) Painting
e) None of these

c) Gardening

The person who likes Blue colour is a/an


a) Architect
b) Musician
d) Cant say
e) None of these

c) Engineer

Whose favourite colour is Yellow?


a) T
b) R
d) Cant say
e) None of these

c) The one who is an artist

Which of the following combinations is not correctly matched?


a) T-Architect-Yellow-Dancing-Cola
b) R-Artist-Green-Gardening-Cola
c) Q-Engineer-Blue-Reading-Cola
d) P-Doctor-Red-Painting-Cola
e) None of these

Directions (Q. 71-73) Read the given information carefully and answer the
questioins that follow.
Independence Day is considered as a day of rejoicing. Thousands of heroes
of independence struggle made the supreme sacrifice of their lives so that we can
enjoy freedom. But see how we have misused the hard won freedom over the last 67
years exactly in the same way the racist representatives of the British, Winston
Churchill, predicted before India wrested independence from the British.
Independence Day and Republic Day do not kindle patriotic fervour in a large
section of our society any longer. They have only been non-working days where
people sit back at h ome and enjoy with their families. They produce only total
revulsion for our corrupt political class, self-serving bureaucracy, greedy business
empires and the completely undisciplined people. The gullible people of the country
have been taken for a ride by the promises made by their leaders all these years
while they have been bleeding the nation white in scam after scam. To millions of
patriotic Indians Independence Day will remain a sad day until all these parasites
are totally annihilated.
I.
II.
III.
IV.
71.

Communal harmony will be disturbed in the country.


Caste system will be promoted.
Terrorism activities will increase in the country.
The economy of the country will be affected.

Which of the following assumptions is implicit in the above passage?


a) People of India lack in nationalism
16

REASONING MADE EASY www.BankExamsToday.com


b) Increasing corruption in India has ended the patriotic feelings among the
Indians
c) Bureaucracy is the hurdle in national integrity
d) The freedom movement of India was not public; it was confined to only
a few individuals.
e) None of these
72.

Which of the following will be the effect of lack of patriotism in India?


a) Only I
b) Only I and II
c) Only III
d) Only III and IV
e) Only I, II and IV

73.

Which of the following statements are in line with the above passage?
a) The independence of India was very hard-earned.
b) People of India are not respecting the hard-earned freedom of India.
c) Increasing corrcuption in India is c reating anger among the people.
d) Independence day is just like a public holiday where people sit at home
and enjoy
e) All of these

Directions (Q. 74-78) Study the following information carefully and answer the
given questions.
Six friends P, Q, R, S, T and U from six different areas, viz. Rohini,
Indirapuram, Dwarka, Kaushambi, Munirka and Vaishali, go for shopping in six
different shopping complexes, viz. Sahara, GIP, CSM, V3S, Shipra and Select
Citywalk, but not necessarily in the same order. Three different types of transports
are used by them, viz. Car, Bus and Metro, in such a manner that two persons use
the same type of transport but not necessarily in the same order.

P uses car and lives in Vaishali but he does not go to CSM and Select
Citywalk.
The person going to Shipra Complex uses car and the one going to GIP uses
bus.
T uses the same type of transport as the person from Dwarka.
CSM Complex is chosen by the person who uses bus.
R is neither from Rohini nor from Munirka and he goes to Shipra Complex.
One of the persons who use Metro goes to Sahara Complex.
Q is from Kaushambi and goes to GIP. He does not use the same transport
as S.
U is neither from Indirapuram nor from Munirka.
Select Citywalk and GIP Complex are not visited by the persons who use
car.
The persons from Kaushambi and Rohini are using the same type of
transport.
17

REASONING MADE EASY www.BankExamsToday.com


74.

75.

76.

77.

78.

Who are using Metro?


a) S and T
b) U and S
d) P and U
e) None of these

c) T and P

Who is going to V, S Complex for shopping?


a) U
b) P
d) S
e) None of these

c) R

Who is using the same transport as U?


a) P
b) Q
d) R
e) None of these

c) T

S belongs to which of the following places?


a) Indirapuram
b) Vaishali
d) Dwarka
e) None of these

c) Munirka

R is using which of the following transports?


a) Car
b) Bus
d) Data inadequate
e) None of these

c) Metro

Directions (Q. 79-83) Study the following information carefully and answer the
given questions:
T, U, V, W, X, Y and Z seven persons belong to seven different families,
viz. Punjabi, Rajasthani, Gujarati, Marathi, Bengali, Kannada and Malayali, but not
necessarily in the same order. All of them are travelling in a train in AC tier II. They
occupy two lower berths, three middle berths and two upper berths. V is a Punjabi
and is not on the upper berths. The Gujarati is the only other person who occupies
the same type of berth as V. U and Y are not on the middle berths and they are
Malayali and Bengali respectively. T is a Marathi. Z is neithr a Kannada nor a
Gujarati. X occupies the same type of b erth as the Rajasthani.
79.

Z belongs to which of the following families?


a) Rajasthani
b) Kannada
d) Bengali
e) None of these

c) Gujarati

80.

Which of the following pairs occupy the lower berth?


a) U, V
b) Y, U
c) W, V
d) T, V
e) None of these

81.

Who belongs to the Gujarati family?


a) Y
b) W
d) Z
e) None of these

82.

c) X

Which of the following combinations is true?


a) W-Gujarati-Upper
18

REASONING MADE EASY www.BankExamsToday.com


b) Y-Bengali-Lower
c) X-Kannada-Lower
d) Z- Rajasthani-Middle
e) None of these
83.

Which of the following groups occupies the middle berths?


a) Z, Y, U
b) Z, X, V
c) X, T, Z
d) W, T, X
e) None of these

Directions (Q. 84-88) Study the given information carefully and answer the
given questions.
Seven plays A, B, C, D, E, F and G are to be held on seven consecutive
days (starting on Monday and ending on Sunday) not necessarily in the same order.
Only one play can be held on one day. Only two plays will be held after play G.
Only three plays will be held between play B and play E. Play B will not be held on
Sunday. Play A will be held before play D and play C (not necessarily immediately
before). Play C will be held after play D (not necessarily immediate after)
84.

Play D will be held on which day?


a) Monday
b) Tuesday
d) Thursday
e) Saturday

c) Wednesday

85.

Which play will be held immediately after Play C?


a) Play E
b) Play F
c) Play B
d) Play G
e) None of these

86.

Which play will be held on Monday?


a) Play F
b) Play B
d) Play D
e) Play A

c) Play E

87.

If all seven plays are held in the alphabetical order of their names starting
on Monday and ending on Sunday, the positions of how many will remain
unchanged as compared to the original schedule?
a) Three
b) More than three
c) One
d) None
e) Two

88.

Play F is related to Monday in a certain way based on the given schedule.


Similarly, Play G is related to Thursday. In the same way, Play B is related
to which of the following days?
a) Wednesday
b) Friday
c) Tuesday
d) Saturday
e) Sunday

19

REASONING MADE EASY www.BankExamsToday.com


Directions (Q. 89-93) Read the following information carefully and answer the
questions given below it.
(i)
(ii)
(iii)
(iv)
(v)
89.

There are eight books Mathematics, Sanskrit, English, Physics, History,


Geography, Hindi and Biology. (Not necessarily in the same order).
Geographys book is fourth from the bottom and Sanskrits book is
third from the top.
There are three books between the books of English and Biology.
Hind i book is at exact between the books of History and Biology.
The book of Physics is at just below the book of Mathematics.
Which book is at top?
a) Mathematics
d) Cannot find

b) Physics
e) None of these

c) Biology

90.

How many books are taken between the books of History and English?
a) Two
b) Three
c) Four
d) Cannot find
e) None of these

91.

The book of Physics is placed between


a) Biology-Mathematics
b) Geography-Biology
c) Mathematics-Sanskrit
d) Sanskrit-Biology

92.

Which of the following book is placed at bottom?


a) Physics
b) Biology
c) Hindi
d) Cannot find
e) None of these

93.

What is the position of the book of History from the top?


a) Fifth
b) Fourth
c) Sixth
d) Seventh
e) None of these

Directions (Q. 94-96) On the basis of the following information, answer the
questions given below.
(i)

(ii)
(iii)
(iv)
94.

Five persons lived in five houses out of six houses P, Q, R, S, T and U.


All houses are in series from North to South. Main gate of each house
was facing west.
From North, Dhirendras house is the third from Sanjays house
In North, Manojs house is at as much distance from Vijays house as in
South, Sanjays house is at from Dhirendras house.
In South, Kunals house is just aftre Vinays house.
Which of the following lived at last in South?
a) Kunal
b) Vinay

c) Dhirendra
20

REASONING MADE EASY www.BankExamsToday.com


d) Sanjay
95.

96.

e) None of these

Which house was empty?


a) P
b) Q
d) S or T
e) None of these

c) R

Which of the following statement was not necessary?


a) IV
b) III
c) II
d) I
e) None of these

Directions (Q. 97-101) these questions are based on the following information.
Study it carefully to answer the questions.
Seven officers L, M, N, P, Q, R and S work in three different shifts I, II and
III with atleast two persons working I n each shift. Each one of them has a different
weekly off from Monday to Sunday not necessarily in the same order.
M works in second shift only with R whose weekly off is on Friday. Qs
weekly off is on the next day of Ls weekly off and both of them work I n different
shifts. P works in third shift and his weekly off is on Saturday. S has a weekly off
on Monday and he works in first shift. The one who has a weekly off on Sunday
works in first shift. L and P do not work in the same shift; Ls weekly off is on
Tuesday.
97.

Whose weekly off falls on Thursday?


a) L
b) N
d) Cannot be determined
e) None of these

c) Q

98.

Which of the following combinations of shift, person and weekly off is


definitely correct?
a) II, M, Sunday
b) III, N, Sunday
c) II, P, Sunday
d) I, L, Tuesday
e) None of these

99.

Whose weekly off is on Sunday?


a) L
b) M
d) Q
e) None of these

100.

101.

c) N

On which day is Qs weekly off?


a) Tuesday
b) Wednesday
d) Cannot be determined
e) None of these

c) Thursday

Which of the following group of officers work in shift I?


a) L, N, S
b) L, S
c) N, S
d) L, P, Q
e) None of these

21

REASONING MADE EASY www.BankExamsToday.com


102.

In a certain code PARTICLE is written as USBQFMDJ, how is


DOCUMENT written in that code?
a) VDEPUONF
b) VDPENFUQ
c) VDPENFOU
d) VDPEUOFN
e) None of these

Directions (Q. 103-107): In these questions letters are to be coded by the digits
and symbols as per the scheme and conditions given below. In each question a
group of letters is given followed by four combinations of digits/symbols
numbered a, b, c and d. The serial number of the combination which correctly
represents the letter group is your answer. If none of the combinations is
correct your answer is e) i.e. None of these.
Letters:
Digit/Symbol Code:

KETJHIFALUBMORP
37%$4*1986#@25

Conditions
(i)
If the first as well as the last letter is a vowel their codes are to be
swapped.
(ii)
If the first as well as the last letter is a consonant b oth are to be coded
by $.
(iii)
If the first letter is as vowel and the last letter is a consonant, t he vowel
is to be coded by ^ and the consonant is to be added by --.
103.

104.

105.

106.

107.

TARIFM
a) $95*1$
%95*1%
d) @95*1@
AJTKLU
a) 9$%386
^$%38
d) --$%38^
ORBETH
a) 5#7%^
25#7%4
d) 45#7%2
KEOMPA
--72@^
372@9
d) 972@3
IJLTPU
a) *$8%6
6$8%6

b) %95*1@

c)

e) None of the above

b) $$%38$

c)

e) None of these

b) ^5 #7%--

c)

e) None of the above

b) ^72@--

c)

e) None of the above

b) *$8%*

c)
22

REASONING MADE EASY www.BankExamsToday.com


d) 6$8%*

e) None of the above

Directions (Q. 108-112): Study the following information carefully to answer


these questions.
Seven friends F, H, J, K, L, M and P are working in different cities viz.
Chennai, Mumbai, Bengaluru, Jaipur, Indore, Ahmedabad, and Hyderabad not
necessarily in the same order. Each one of them has a different profession viz.,
C.A., Doctor, Professor, Engineer, Banker, Lawyer and Architect not necessarily in
the same order. H is a Lawyer and he works in Jaipur. K is a Doctor and he does not
work I n Chennai. The C.A. works in Mumbai. M works in Indore. Architect works
in Hyderabad. L is a B anker and he works in Bengaluru. F is an Engineer. J does
not work in Mumbai.
108.

Who is an architect?
a) J
d) L

b) P
e) None of these

c) K

109.

Which of the following combinations of person, profession and city is


definitely correct?
a) F-Engineer-Bengaluru
b) K-Architect-Hyderabad
c) L-Banker-Mumbai
d) M-Professor-Jaipur
e) None of the above

110.

Who is working in Mumbai?


a) J
b) K
d) P
e) None of these

111.

112.

Engineer works in which city?


a) Ahmedabad b) Chennai
d) Bengaluru e) None of the above
Who is a professor?
a) P
d) M or J

c) M

c) Mumbai

b) J
e) None of these

c) M

113.

In a certain code, DOWN is written as 5139 and NEAR is written as 9486.


How is RODE written in that code?
a) 6514
b) 6154
c) 9154
d) 3154
e) None of these

114.

In a certain code ORGANISE is written as BHSPDRHM. How is


DOUBTFUL written in that code?
23

REASONING MADE EASY www.BankExamsToday.com


a) CVPEKTES
d) ATNCMVGU

b) CVPIMVGU
e) None of these

c) ATNCKTES

115.

If it is possible to make only one meaningful English word with the first,
the se cond, the fourth and the tenth letters of the word M A J E S T I C A
L, which of the following will be the second letter of that word? If no such
word can be made, give X as the an swer and if mor e than one such word
can be made, give Y is the answer
a) M
b) E
c) L
d) X
e) Y

116.

In a certain code language, pit ne means come here, ne ta ja means


come and go and ja sa re means you and me. What does ta means in
that code language?
a) come
b) and
c) here
d) go
e) Cannot be determined

117.

RT is related to VX and BD is related to FH, in the same way as


KM is related to
a) NP
b) OR
c) OQ
d) PR
e) None of these

118.

In a certain code GEAR is written as 5%9$ and FIRM is written as


3@$7. How is FAME written in that code?
a) 397%
b) 3%97
c) 597%
d) 5397
e) None of these

Directions (Q. 119-122) In each of the questions below are given three
statements followed by three conclusions numbered I, II and III. You have to
take the given statements to be true even if they seem to be at variance from
commonly known facts. Read all the conclusions and then decide which of the
given conclusions logically follows from the given statements disregarding
commonly known facts.
119.

Statements:

Some books are trees.


All trees are roads.
All roads are wheels.

Conclusions:

I. Some wheels are books.


II. Some roads are books.
III. Some wheels are trees.

a) Only I and II follow


b) Only II and III follow
c) Only I and III follow
d) All I, II, III follow
e) None of the above
24

REASONING MADE EASY www.BankExamsToday.com

120.

Statements:

All stones are rivers.


All rivers are cars.
Some cars are trains.

Conclusions:

I. Some trains are stones


II. Some cars are stones.
III. Some trains are rivers.

a) None follows
b) Only I follows
c) Only II follows
d) Only III follows
e) Only II and III follow
121.

Statements:

All desks are rooms.


Some rooms are halls.
All halls are leaves.

Conclusions:

I. Some leaves are desks.


II. Some halls are desks.
III. Some leaves are rooms.

a) None follows
b) Only I follows
c) Only II follows
d) Only III follows
e) Only II and III follow
122.

Statements:

Some bags are plates.


Some plates are chairs.
All chairs are tables.

Conclusions:

I. Some tables are plates.


II. Some chairs are bags.
III. No chair is bag.

a) Only I follow
b) Only II follows
c) Only III follows
d) Only II and III follow
e) None of these
Directions (Q. 123-128) In the following questions, the symbols $, %, @, and
* are used with the following meaning as illustrated below.
P % Q means P is not smaller than Q.
P $ Q means P is not greater than Q.
P * Q means P is neither greater than nor equal to Q.
25

REASONING MADE EASY www.BankExamsToday.com


P Q means P is neither smaller than nor equal to Q.
P @ Q means P is neither greater than nor smaller than Q.
123.

Statements:
Conclusions:

R @ K, K $ F, F * N
I. N R
II. F @ R
III. F R

a) Only I is true
b) Only either II or III is true
c) Only I and either II or III are true
d) Only III is true
e) None of the above
124.

Statements:
Conclusions:

J M, M * K, K % D
I. J D
II. D * M
III. K J

a) None is true
b) Only I is true
c) Only II is true
d) Only III is true
e) Only I and II are t rue
125.

Statements:
Conclusions:

H * T, T $ B, B R
I. R H
II. B H
III. T * R

a) Only I is true
b) Only I and II are true
c) Only I and III are true
d) Only II and III are true
e) None of the above
126.

Statements:
Conclusions:

R $ D, D @ N, N F
I. F * D
II. F * R
III. N % R

a) Only I and II are true


b) Only I and III are true
c) Only II and III are true
d) All are true
e) None of the above
127.

Statements:
Conclusions:

F $ B, B @ H, H % K
I. B % F
II. K @ F
26

REASONING MADE EASY www.BankExamsToday.com


III. K $ B
a) Only I is true
b) Only II is true
c) Only III is true
d) Only I and III are true
e) None of these
128.

Statements:
Conclusions:

M % D, D * K, K $ N
I. K M
II. N D
III. M N

a) Only I is true
b) Only II is true
c) Only III is true
d) Only I and II are true
e) None of these
Directions (Q. 129-133) Study the following information carefully and answer
the questions given below.
P, Q, R, S, T, V and W are seven students of a college. Each of them has a
favourite subject from Physics, Chemistry, English, Biology, History, Geography
and Philosopy, not necessarily in the same order. Each of them also has a favourite
sport from Football, Cricket, Hockey, Volleyball, Badminton, Table Tennis and
Basketball, not necessarily in the same order.
R likes Philospy and his favourite sport is Hockey. The one who likes
Football likes English. Ts favourite sport is not badminton or Table Tennis. V does
not like either History or Bilogy. The one whose favourite sport is Basketball does
not like Physics. W likes Chemistry and his favourite sport is Volleball. S likes
Geography. Qs favourite sport is Badminton. V does not like English and his
favourite sport is not Basketball. Ps favourite sport is Cricket. The one whose
favourite sport is Badminton does not like Biology.
129.

130.

131.

Who likes History?


a) P
d) V

b) R
e) Data inadequate

c) Q

Whose favourite sport is Basketball?


a) S
b) W
d) Data inadequate
e) None of these

c) Q

Which subject does T like?


a) Biology
b) Physics
d) Data inadequate
e) None of these

c) Chemistry

27

REASONING MADE EASY www.BankExamsToday.com


132.

133.

What is Qs favourite sport?


a) Cricket
b) Table Tennis
d) Badminton
e) None of these

c) Football

Which subject does P like?


a) History
b) Biology
d) Data inadequate
e) None of these

c) Chemistry

134.

Mustard is related to seed in the same way as Carrot is related to


a) Fruit
b) Stem
c) Flower
d) Root
e) None of these

135.

How many meaningful English words can be formed made with the letters
ESTR using each letter only once in each word?
a) None
b) One
c) Two
d) Three
e) More than three

136.

Four of the following five are alike in a certain way and so form a group.
Which is the one that does not belong to that group?
a) Cup
b) Jug
c) Tumbler
d) Plate
e) Pitcher

137.

Four of the following five are alike in a certain way and so form a group.
Which is the one that does not belong to that group?
a) Copper
b) Mercury
c) Iron
d) Aluminium
e) Zinc

138.

FI is related to LO in the same way as PS in related to


a) VY
b) VZ
c) WZ
d) UX
e) None of these

139.

Four of the following five are alike in a certain way and so form a group.
Which is the one that does not belong to that group?
a) 217
b) 143
c) 241
d) 157
e) 181

140.

Gram is related to Mass in the same way as Centimetre is related to


a) Area
b) Volume
c) Length
d) Sound
e) Energy

141.

Four of the following five are alike in a certain way and so form a group.
Which is the one that does not belong to that group?
a) 12
b) 28
c) 52
d) 68
e) 96

28

REASONING MADE EASY www.BankExamsToday.com


142.

If white means black, black means red, red means blue. blue
means yellow and yellow means grey then which of the following
represents the colour of clear sky?
a) Blue
b) Red
c) Yellow
d) Cant be determined e) None of these

143.

In a certain code, MODEL is written as 513#2 and DEAR is written as


3#%8. How is LOAD written in that code?
a) 21%3
b) 23%1
c) 25%3
d) 21#3
e) None of these

Directions (Q. 144-148) Study the following information carefully and answer
the questions given below.
P, Q, R, S, T and M are six students of a school, one each studies in Class IVI. Each of them has a favourite colour from red, black, blue, yellow, pink and
green, not necessarily in the same order.
Q likes black and does not study in Class IV or V. The one who studies in
Class IV does not like green. P studies in Class II. M likes blue and does not study
in Class IV. The one who likes yellow studies in Class VI. S likes pink and studies
in Class I. R do not study in Class VI.
144.

145.

146.

In which class does R study?


a) V
d) Data inadequate

b) III
e) None of these

c) IV

Which colour does R like?


a) Black
d) Blue

b) Yellow
e) None of these

c) Green

Which colour does P like?


a) Green
d) Data inadequate

b) Yellow
e) None of these

c) Red

147.

Which of the following combinations is correct?


a) P-II-Yellow
b) Q-III-Green
c) S-I-Black
d) T-V-Yellow
e) None of these

148.

In which class does M study?


a) IV
b) III
d) V
e) None of these

149.

c) II

How many meaningful English words can be formed with the letters
MASTE using each letter only once in each word?
a) None
b) One
c) Two
d) Three
e) More than three
29

REASONING MADE EASY www.BankExamsToday.com

150.

Four of the following five are alike in a certain way and so form a group.
Which is the one that does not belong to that group?
a) 78
b) 48
c) 72
d) 54
e) 42

Directions (Q. 151-155) Study the following information carefully and answer
the questions given below.
P, Q, R, S, T, V and W are seven students of a school. Each of them studies
in different standard from standard IV to standard X not necessarily in the same
order. Each of them has favourite subject from English, Science, History,
Geography, Mathematics, Hindi and Sanskrit not necessarily in the same order.
Q studies in VII standard and does not like either Mathematics or
Geography. R likes English and does not study either in V or in IX. T studies in
VIII standard and likes Hindi. One who likes Science studies in X standard? S
studies in IV standard. W likes Sanskrit. P does not study In X standard. One who
likes Geography studies in V standard.
151.

152.

153.

154.

155.

In which standard does W study?


a) VII
b) IX
d) Data inadequate
e) None of these

c) X

Which subject does P like?


a) Geography
b) Mathematics
d) History
e) None of these

c) English

Which subject does S like?


a) History
b) Geography
d) Data inadequate
e) None of these

c) Mathematics

In which standard does P study?


a) IV
b) VII
d) X
e) None of these

c) IX

Which of the following combination of student standard subject is correct?


a) T-VIII-Mathematics
b) W-VII-Sanskrit
c) Q-VII-Geography
d) V-X-Science
e) None of these

Directions (Q. 156-160) Study the following information carefully and answer
the questions given below
30

REASONING MADE EASY www.BankExamsToday.com


(i)
(ii)
(iii)
(iv)
(v)
(vi)

A, B, C, D, E, F, G and H are 8 students each having a different height.


D is shorter than A but taller than G.
E is taller than H but shorter than C.
B is shorter than D but taller than F.
C is shorter than G.
G is not as tall as F.

156.

Which of the following is definitely false?


a) G is shorter than F
b) C is shorter than F
c) F is taller than C
d) B is taller than E
e) All are true

157.

If another student J who is taller than E but shorter than G is added to the
group. Which of the following will be definitely true?
a) C and J are of same height
b) G is shorter than D
c) G is shorter than H
d) G is taller than A
e) None of the above

158.

Which of the following will definitely be the third from top when 8 students
are arranged in descending order of height?
a) B
b) F
c) G
d) B or G
e) Cannot be determined

159.

How many of them are definitely shorter than F?


a) Three
b) Four
c) Five
d) Data inadequate
e) None of these

160.

Which of the following is redundant to answer all the above questions?


a) (ii) only
b) (ii) and (iii) only
c) (iii) and (iv) only
d) (ii) and (v) only
e) All are necessary to answer the above questions

161.

A, B, C, D and E are five students in a class. D did not enter along with A
or E but entered before C. B did not enter before C but entered along with
A. E was n ot t h e last to enter. Which of the following I s definitely true?
a) C entered the class only after D
b) D entered the class only after E
c) B entered the class after A
d) A entered the class after D
e) None of the above
31

REASONING MADE EASY www.BankExamsToday.com

162.

Among A, B, C, D and E, each having a different weight, D is not lighter


than B and E is not heavier than A. C is not the heaviest. Who among them
is the lightest?
a) D
b) B
c) E
d) Data inadequate
e) None of these

163.

Each consonant in the word BISCUIT is replaced by the next letter in the
English alphabet and each vowel is replaced by the previous letter in the
English alphabet and the letters so obtained are rearranged in alphabetical
order, which of the following will be the 3rd from the left end after the
rearrangement?
a) C
b) D
c) H
d) T
e) None of these

164.

In a certain code DONE is written as 5139 and SEAL is written as 8942.


How is LOAD written in that code?
a) 2415
b) 2145
c) 2945
d) 2182
e) None of these

165.

Four of the following five are alike in a certain way and so form a group.
Which is the one that does not belong to that group?
a) 215
b) 143
c) 247
d) 91
e) 65

166.

Four of the following five are alike in a certain way and so form a group.
Which is the one that does not belong to that group?
a) Gold
b) Nickel
c) Platinum
d) Tungsten
e) Diamond

167.

Among P, Q, T, A and B each having a different h eight, T is taller than P


and B but shorter than A and Q. P is not the shortest. Who among them is
the tallest?
a) A
b) Q
c) P
d) P or B
e) Data inadequate

168.

How many meaningful English words can b e made with the letter RBAE
using each letter only once in each word?
a) None
b) One
c) Two
d) Three
e) More than three

169.

What should come next in the following letter series?


HGFEDCBAGFEDCBAGFEDCB
a) E
b) G
c) F
d) B
e) None of these
32

REASONING MADE EASY www.BankExamsToday.com


170.

Mohan correctly remembers that his fathers birthday is before 20 th January


but after 16th January whereas his sister correctly remembers that their
fathers birthday is after 18th January but before 23rd January. On which
date in January is definitely their fathers birthday?
a) 18th
b) 19th
c) 20th
d) Data inadequate
e) None of these

171.

If P denotes , R denotes +, M denotes and W denotes +


then
20 R 5 W 12 M 3 P 4 =?
a) 4
b) 16
c) 28
d) 52
e) None of these

172.

If the positions of 2nd and 3rd digits within each number are interchanged,
which of the following will be the sum of the first and 2nd digits of the 3rd
highest number?
a) 16
b) 10
c) 9
d) 15
e) None of these

173.

Which of the following are the sum of the first and the third digits of the
second lowest number?
a) 16
b) 10
c) 18
d) 5
e) None of these

174.

If the positions of the first and the second digits within each number are
interchanged, which of the following will be the difference between the
highest and the second highest number?
a) 203
b) 133
c) 385
d) 182
e) 144

175.

If the positions of the first and the third digits within each number are
interchanged, which of the following will be the sum of the second and
third digits of the lowest number?
a) 8
b) 11
c) 15
d) 12
e) None of these

Directions (Q. 176-178) Study the following information carefully and answer
the questions given below
There are six employees A, B, C, D, E and F in a company. A is more
experienced than two employees. D has more experience than employee C. D has
less experience than employee F. E has less experience than D. E does not have the
least experience. F is not the most experienced employee. The employee who is 2nd
most experienced has an experience of 13 years. The employee who is second least
experienced has an experience of five years.
33

REASONING MADE EASY www.BankExamsToday.com


176.

Who among the following may have an experience of 10 years?


a) E
b) D
c) A
d) Either A or D
e) F

177.

Who among the following is less experinced than only B?


a) F
b) C
c) D
d) A
e) E

178.

Who among the following is more experienced than only C?


a) A
b) D
c) E
d) F
e) Cannot be determined

Directions (Q. 179-183) Study the following information carefully and answer
the questions given below
Seven persons A, B, C, D, E, F and G bought different watches viz.,
Rado, Omni, Fas Track, Seiko, Casio, Tissot and Titan on different days of the
same week from Monday to Sunday, but not necessarily in the same order.
The watch of Omni Company was bought on Friday. F bought Titan watch
on Tuesday. Onle one watch was bought between the watches Omni and Rado. B
bought Tissot watch immediately after the person who bought Casio watch. Seiko
watch was not bought after the Tissot watch. G bought watch immediately after B
but before C and D. A does not buy the watch of Casio company. D does not buy
the watch of Rado Company.
179.

Who among the following bought Seiko watch?


a) D
b) C
c) E
d) A
e) None of these

180.

On which of the following days of the week D bought a watch?


a) Saturday
b) Sunday
c) Thursday
d) Friday
e) None of these

181.

Who among the following did buy Casio watch?


a) G
b) E
c) D
d) C
e) None of these

182.

Who among the following did buy Omni watch on Friday?


a) C
b) D
c) G
d) E
e) None of these

183.

Which of the following combinations of Day-Person-Watch is correct?


a) Friday-A-Fast Track
b) Monday-E-Omni
c) Wednesday-B-Casio
d) Saturday-C-Tissot
34

REASONING MADE EASY www.BankExamsToday.com


e) Sunday-C-Rado
184.

Present ages of father and son are in the ratio of 5:1 respectively. Seven
years later this ratio becomes 3:1. What is the sons present age in years?
a) 8
b) 7
c) 6
d) 5
e) None of these

185.

From the given alternative words, select the word which can be formed
using the letters of the given word: TRADITIONAL
a) NATION
b) RADIO
c) ANIMAL
d) DIRTY
e) None of these

186.

A shepherd had 17 sheep. All but nine died. How many sheep are left?
a) 9
b) 8
c) 7
d) 10
e) None of these

187.

Select the missing number from the given responses.


1
4
2
3
2
?
a) 2
b) 3
c) 4
d) 5
e) None of these

188.

Town A and Town B were 600 km. apart. Joshua left town A for town B
and travelled at an average speed of 65 km/h. At the same time, Menon left
town B for town A. The two of them met 5 hours later. Find Menons
average speed.
a) 55 km/h
b) 60 km/h
c) 65 km/h
d) 120 km/h
e) None of these

189.

In a certain code ALIVE is written as WFHBM. How is VALUE written in


that code?
a) VFMWB
b) VFKWB
c) WBAVF
d) WBKVF
e) None of these

190.

The letters in the word DANGEROUS are changed in such a way that the
consonants a rereplaced by the previous letter in the English alphabet and
the vowels are replaced by the next letter in the English alphabet. Which of
the following will be the 3rd letter from the left end of the new set of letters?
a) B
b) M
c) O
d) L
e) None of these

Directions (Q. 191-195) Study the following information carefully and answer
the questions given below
A, B, C, D, E, F, G and H are eight persons working in three different
departments viz. Operations, Sales and Finance of an organisation with at least two
35

REASONING MADE EASY www.BankExamsToday.com


and not more three in any department. They are in three different scales viz. I, II and
III with at least two in any one scale.
Both the employees in Operations departments are in Scale II. D works in
Sales department and belongs to Scale I. A works in Finance departments and does
not belong to Scale I. Two employees in Sales departments belong to one scale. F
works with only H in one of the departments. C works with E in one of the
departments. B does not work with either C or A in the same department. G does
not belong to Scale III. E does not belong to Scale I.
191.

Which of the following combinatins is correct?


a) Operation-F-I
b) Operation-H-III
c) Sales-B-II
d) Finance-E-III
e) All are correct

192.

Which of the following groups of employees work in sales department?


a) DBE
b) DBC
c) DBG
d) Data inadequate
e) None of these

193.

H belongs to which scale?


a) I
b) II
d) Data inadequate
e) None of these

c) III

G belongs to which scale?


a) II
b) III
d) I or II
e) None of these

c) I

C belongs to which scale?


a) I
b) II
d) I or II
e) None of these

c) III

194.

195.

Answers:
1.
2.
3.
4.
5.
6.
7.
8.
9.
10.
11.

Option A
Option C
Option E
Option C
Option A
Option A
Option B
Option E
Option B
Option C
Option D
36

REASONING MADE EASY www.BankExamsToday.com


12.
13.
14.
15.
16.
17.
18.
19.
20.
21.
22.
23.
24.
25.
26.
27.
28.
29.
30.
31.
32.
33.
34.
35.
36.
37.
38.
39.
40.
41.
42.
43.
44.
45.
46.
47.
48.
49.
50.
51
52.
53.
54.
55.
56.
57.

Option A
Option D
Option C
Option C
Option C
Option D
Option A
Option A
Option C
Option B
Option D
Option A
Option E
Option E
Option D
Option C
Option C
Option B
Option C
Option A
Option D
Option A
Option B
Option C
Option B
Option A
Option E
Option C
Option B
Option E
Option A
Option C
Option A
Option A
Option C
Option B
Option C
Option D
Option A
Option A
Option B
Option D
Option E
Option C
Option B
Option C
37

REASONING MADE EASY www.BankExamsToday.com


58.
59.
60.
61.
62.
63.
64.
65.
66.
67.
68.
69.
70.
71.
72.
73.
74.
75.
76.
77.
78.
79.
80.
81.
82.
83.
84.
85.
86.
87.
88.
89.
90.
91.
92.
93.
94.
95.
96.
97.
98.
99.
10.
101.
102.
103.

Option B
Option B
Option D
Option C
Option A
Option A
Option C
Option C
Option B
Option A
Option C
Option A
Option A
Option B
Option D
Option E
Option A
Option B
Option B
Option D
Option A
Option A
Option C
Option B
Option D
Option C
Option D
Option A
Option E
Option E
Option C
Option A
Option E
Option C
Option B
Option C
Option A
Option C
Option C
Option E
Option D
Option C
Option B
Option A
Option D
Option A
38

REASONING MADE EASY www.BankExamsToday.com


104.
105.
106.
107.
108.
109.
110.
111.
112.
113.
114.
115.
116.
117.
118.
119.

Option E
Option B
Option C
Option D
Option A
Option E
Option D
Option B
Option C
Option B
Option A
Option E
Option D
Option C
Option A
Optoin D

120.

Option C

39

REASONING MADE EASY www.BankExamsToday.com

121.

122.

Option D

Option E

40

REASONING MADE EASY www.BankExamsToday.com

123.

Option C
R= K, K F, F < N
R=KF<N
N > R, F > R, F = R
So, either II or III and I are true.

124.

Option A
J > M, M < K, K D
J> M < K D
The relations cannot made between J and D, D and M and K and J. So none
is true.

125.

Option E
H < T, T B, B > R
H < T B > R or B > H
Only II is true.

126.

Option B
R D, D = N, N > F
RD=N>F
F < D and N R
Only I and III are true.

127.

Option A
F B, B = H, H K
FB =H K
41

REASONING MADE EASY www.BankExamsToday.com


Only I is true.
128.

Option B
M D, D < K, K N
MD<KN
Only II is true.

Student
Sports

P
Cricket

Subject

Bilogy

129.
130.
131.
132.
133.

Option D
Option A
Option E
Option D
Option B

134.

Option D

Q
Badmint
on
History

R
Hocke
y
Philos
ophy

S
Basket
ball
Geogra
phy

T
Footbal
l
English

V
Table
Tennis
Physic
s

W
Volleyba
ll
Chemistr
y

As mustard is related to seed in the same way carrot is related to root.


135.

Option B
Meaningful word is REST.

136.

Option D
Plate is normally used for solid matters.

137.

Option B
Except mercury all are available in solid form at general or normal
temperature.

138.

Option A
As, F + 6 L
I+ 6 O

139.

Similarly P + 6 V
S + 6 Y

Option A

42

REASONING MADE EASY www.BankExamsToday.com


140.

Option C
As gram is related to mass in the same way ` centimetre is related to
length.

141.

Option B
28 is divisible by 7.

142.

Option C
Colour of clear sky is blue and according to question, blue means yellow so
the colour of clear sky will be yellow.

143.

Option A
M
O
5
1

D
3

E
#

D
3

E
#

A
%

R
8

L
2

O
1

A
%

D
3

Student
P
Q
R
S
T
M

Class
II
III
IV
I
VI
V

L
2

Favaourite colour
Green
Black
Red
Pink
Yellow
Blue

144.
145.
146.
147.
148.

Option C
Option E
Option A
Option E
Option D

149.

Option E
Meaningful words = MATES, STEAM, TAMES and TEAMS

150.

Option A
Except 78, if subtract 1 all other give a prime number.

Student
P

Class
V

Favourite Subject
Geography
43

REASONING MADE EASY www.BankExamsToday.com


Q
R
S
T
V
W

VII
VI
IV
VIII
X
IX

151.
152.
153.
154.
155.

History
English
Mathematics
Hindi
Science
Sanskrit

Option B
Option A
Option C
Option E
Option D

(i)
There are 8 students A, B, C, D, E, F, G and H
(ii)
A >D> G
(iii)
C > E> H
(iv)
D > B > F
(v)
G> C
(vi)
F> G
From all statements.
A>D>B>F>G>C>E>H
156.

Option E
All are true.

157.

Option B
A>D > B > F>G >C >E >H

158.

Option A
A>D>B>F>G>C>E>H

159.

Option B
G, C, E and H is smaller than F.

160.

Option E
All statements are necessary.

161.
162.

Option D
Option D

163.

Option C
B
I
S
C
According to question,
C
H
T
D

U
44

REASONING MADE EASY www.BankExamsToday.com


According to alphabet,
C
D
H
H
rd
So, H is 3 from left.
164.

Option B
D
O
5
1

N
3

E
9

S
8

E
9

A
4

L
2

So,

L
2

O
1

A
4

D
5

165.

Option A

Apart from 215 all are divisible by 13.


143 = 13 11; 247 = 13 19; 91 = 13 7; 65 = 13 5
215 = 5 43
166.
167.
168.

Option E
Option E
Option C
Meaningful words are BEAR, BARE

169.

Option C
HGFEDCBA, GFEDCBA, GFEDCB, FEDCB
Next letter series = F

170.

Option B
According to Mohan, birthday may be 17, 18 or 19th January. According to
Mohans sister, birthday may be at 19, 20, 21 or 22th Jan. So common day
= 19 Jan.

171.

Option A

172.

20 R 5 W 12 M 3 P 4 = ?
? = 20 5 + 12 3 4
? = 4 + 12 12 = 4
Option A
832 823
719 791
654 645
967 976

45

REASONING MADE EASY www.BankExamsToday.com


481 418
3rd largest number is 791 and the sum of their first and second digit = 7 + 9
= 16
173.

Option B
Second smallest number is 654. So, the sum of the first and the third digit is
6 + 4 = 10

174.

Option E
After interchanging first and second digit of number.
832 382
719 179
654 564
967 697
481 841
Highest number = 841
Second highest number = 697
Difference between them = 841 697 = 144

175.

Option D
Interchanging the first and third digit of number
832 238
719 917
654 456
967 769
481 184
Smallest number = 184
So, the sum of second and third digit = 8 + 4 = 12

176.

Option D
F is not the most experienced. So, B is the most experienced.
Now, B > F > D > A > E > C

177.

Option A
F has less experience than only B.

178.

Option C
E is more experienced than only C.

Day
Monday
Tuesday
Wednesday
Thursday
Friday

Person
A
F
E
B
G

Watch
Seiko
Titan
Casio
Tissot
Omni
46

REASONING MADE EASY www.BankExamsToday.com


Saturday
Sunday

D
C

Fast Track
Rado

179.

Option D
A bought Seiko watch on Monday.

180.

Option A
D bought Fast Track watch on Saturday.

181.

Option B
E bought Casio watch on Wednesday.

182.

Option C
G bought Omni watch on Friday.

183.

Option E
The combination Sunday-C-Rado is correct.

184.

Option B
Suppose the present age of son be x years
Therefore, the present age o f father would b e 5x years
According to question,
5x + 7 / x + 7 = 3/1
5x + 7 = 3x + 21
5x 3x = 21 7
2x = 14
X=7

185.

Option B
There is only one N in the given word. Therefore, the word NATION
cannot be formed. There is no M letter in the given word. Therefore, the
word ANIMAL cannot be formed. There is no Y letter in the given word.
Therefore, the word DIRTY cannot be formed.

186.

Option A
There are 17 sheep. All but nine died. It means 9 sheep are left.

187.

Option D
1+3=4
4 2 = 2
2+1=3
3 1 = 2
2+3=5

188.

Option A
47

REASONING MADE EASY www.BankExamsToday.com


Joshua covered a distance of 65 5 = 325 km. in 5 hours
Menon has to cover a distance of (600 325) = 275 km. in 5 hours
Speed of Menon = 275/5 = 55 kmph
189.
190.

Option B
Option B

Person
A
B
C
D
E
F
G
H

191.

Department
Finance
Sales
Finance
Sales
Finance
Operations
Sales
Operations

Scale
III
III
III
I
III
II
I
II

Option D
The combination Finance-E-III is correct.

192.

Option C
B, D and G work in Sales department.

193.

Option B
H belongs to scale II

194.

Option C
G belongs to Scale I

195.

Option C
C belongs to scale III

48

REASONING MADE EASY www.BankExamsToday.com

49

REASONING MADE EASY www.BankExamsToday.com

Chapter-2
Input-Output
Directions (Q. 1-5) A word can and number arrangement machine when given
an input line of words and numbers rearranges them following a particular
rule in each step. The following is an illustration of input and rearrangement.
Input : go now 52 38 17 for again 65
Step I: 65 go now 52 38 17 for again
Step II: 65 again go now 52 38 17 for
Step III: 65 again 52 go now 38 17 for
Step IV: 65 again 52 for go now 38 17
Step V: 65 again 52 for 38 go now 17
Step VI: 65 again 52 for 38 go 17 now
Step VI is the last step of the rearrangement.
As per the rules followed in the above steps, find out in each of the following
questions the appropriate step for the given input.
Input: show 51 36 new far 81 46 goal
1.

Which of the following steps will be the last but one?


a) VII
b) VIII
c) VI
d) V
e) None of these

Input: home turf 39 24 86 44 roll over


2.
Which of the following steps will be the last?
a) X
b) IX
d) VII
e) None of these

c) VIII

Step II of an input is: 76 ask 12 32 begin over join 42.


3.

How many more steps will be required to complete the rearrangement?


a) Four
b) Five
c) Six
d) Three
e) None of these

Step IV of an input is: 58 box 47 dew 15 21 town pot.


4.
Which of the following steps will be the last?
a) VII
b) VI
d) IX
e) None of these

c) VIII

Step III of an input is: 94 car 86 window shut 52 31 house.


50

REASONING MADE EASY www.BankExamsToday.com

5.

Which of the following is definitely the input?


a) 94 car window 86 shut 52 31 house
b) 80 window 94 car shut 52 31 house
c) car shut window 86 52 31 house 94
d) Cannot be determined
e) None of these

Directions (Q. 6-11) Given an input line the machine arranges the words and
numbers in steps in a systematic manner as illustrated below
Input
line 59 dress fine shine 32 66 72 offer
Step I
72 56 dress fine shine 32 66 offer
Step II
72 shine 56 dress fine 32 66 offer
Step III
72 shine 66 56 dress fine 32 offer
Step IV
72 shine 66 offer 56 dress fine 32
Step V
72 shine 66 offer 56 fine dress 32
Step VI
72 shine 66 offer 56 fine 32 dress
Step VI is the last step and the output in Step Vi is the final output.
As per the rules followed in the above steps, find out in each of the following
questions the appropriate step for the given input.
6. Step IV of an input is 62 sound 56 sleep roam present 33 49. What will be the
input definitely?
a) sound 62 sleep 56 roam present 33 49
b) sleep sound 62 56 roam present 33 49
c) 62 sound sleep 56 roam present 33 49
d) Cannot be determined
e) None of these
7. Which of the following will be the t hird step for input jockey firm 36 43
growth chart 22 45?
a) 45 jockey 43 growth firm 36 chart 22
b) 45 jockey 43 firm growth 36 chart 22
c) 45 jockey 43 growth 36 firm chart 22
d) 45 jockey 43 firm 36 growth chart 22
e) None of these
8. Step II of an input is 53 window 42 50 door lock key 36. How many more
steps will be required to complete the arrangement?
a) Three
b) Four
c) Five
d) Six
e) None of these

51

REASONING MADE EASY www.BankExamsToday.com


9. What will be the fifth step of an input whose first step is 85 journey train 36 54
daily 28 mansion?
a) 85 train 54 mansion 28 journey daily 36
b) 85 train 54 mansion journey 36 daily 28
c) 85 train 54 mansion 36 journey daily 28
d) There is no such step
e) None of these
10. Which step will bed the last step for an I nput whose second step is 63 Sour 18
56 grapes healthy 32 rise?
a) IV
b) V
c) VIII
d) VII
e) None of these
11. Which word/number will be sixth from right in step fifth whose second step is
63 Sour 18 56 grapes healthy 32 rise?
a) Rise
b) 56
c) Sour
d) 32
e) 18
Directions (Q. 12-16) Read the following information carefully and answer the
given questions.
A word/number arrangement machine when given an input line of words
and numbers rearrange them following a particular rule in each step. The following
is an illustration of input and rearrangement.
(All the numbers are two digits numbers)
Input: 33 food water 19 42 air 27 54 72 fire lunch 62 cool not
Step 1: 19, 33 food 42 air 27 54 72 fire lunch 62 cool not water
Step 2: 27 19 33 food 42 air 54 72 fire lunch 62 cool water not
Step 3: 33 27 19 food 42 air 54 72 fire 62 cool water not lunch
Step 4: 42 33 27 19 air 54 72 fire 62 cool water not lunch food
Step 5: 54 42 33 27 19 air 72 62 cool water not lunch food fire
Step 6: 62 54 42 33 27 19 air 72 water not lunch food fire cool
Step 7: 72 62 54 42 33 27 19 water not lunch food fire cool air
And step 7 is the last step of the above input, as the desired arrangement is obtained.
As per the rules followed in the above steps, find out in each of the following
questions the appropriate step for the given input.
Input: Rail 43 27 Book Coach 56 Ticket waiting confirm 62 35 72 sleeper 16
(All the numbers are two digit numbers)
12.

Which step number is the following output?


56 43 35 27 16 Book Coach 62 72 waiting ticket sleeper rail confirm
a) Step 4
b) Step 5
c) Step 6
d) Step 7
e) None of these
52

REASONING MADE EASY www.BankExamsToday.com


13.

Which of the following step represents the maximum gap between Ticket
and 35?
a) Step 2
b) Step 3
c) Step 4
d) Step 5
e) None of these

14.

In step 5, if Book is related to waiting and in the same way 27 is


related to 62. Which of the following would 72 be related to in the same
pattern?
a) Ticket
b) Rail
c) Sleeper
d) 35
e) None of these

15.

In step 4, which of the following word/number would be at 4th position


from the left end?
a) 16
b) 27
c) Book
d) Rail
e) None of these

16.

Which of the following would be one of the steps?


a) 27 16 Rail 43 Book Coach 56 Confirm 62 35 sleeper 72 waiting Ticket
b) 43 35 27 16 Book Coach Confirm 62 56 72 waiting sleeper Ticket Rail
c) 62 56 43 35 27 16 72 waiting sleeper ticket Rail confirm Coach Book
d) 35 27 16 Rail 43 Book Coach 56 Confirm 62 72 waiting Ticket sleeper
e) 56 43 35 27 16 Book confirm 62 72 waiting Ticket sleeper Rail coach

Directions (Q. 17-23) Study the following information carefully and answer the
questions based on it.
A set of words and numbers is passed through on arrangement machine and
following rearrangement is obtained.
Input
talk 48 11 rude 97 84 35 walk jug home 25 bag 77 alone
Step I
97 talk 48 11 rude 84 35 walk jug home 25 77 alone bag
Step II
97 84 talk 48 11 rude 35 walk jug 25 77 alone bag home
Step III
97 84 77 talk 48 11 rude 35 walk 25 alone bag home jug
Step IV
97 84 77 48 talk 11 35 walk 25 alone bag home jug rude
Step V
97 84 77 48 35 11 nwalk 25 alone bag home jug rude talk
Step VI
97 84 77 48 35 25 11 alone bag home jug rude talk walk
Step VI is the last step of rearrangement.
Now, a set of words and letters is given below. As per the rules followed in above
steps, answer the questions based on it.
Input: 81 who sit 19 32 not but ink flow 51 27 van 68 92
17.

Which is the last step of this input?


a) V
b) VI
d) VIII
e) None of these

c) VII

53

REASONING MADE EASY www.BankExamsToday.com


18.

Which of the following is the Step III?


a) 92 81 68 who sit 19 32 not 51 27 van but flow ink
b) 92 81 68 51 who sit 32 19 not 27 van but flow ink
c) 92 81 68 51 who sit 19 32 not 27 van but flow ink
d) 92 81 68 51 32 who sit 19 not 27 van but flow ink
e) None of these

19.

Which step is 92 81 68 51 32 27 who 19 van but flow ink not sit?


a) Step IV
b) Step V
c) Step III
d) Step VI
e) None of these

20.

Which is third element to the right from the sixth element from the right
end in Step II?
a) 32
b) van
c) 27
d) floe
e) 25

21.

How many elements are there between who and van in Step IV?
a) 3
b) 5
c) 4
d) 0
e) 1

22.

In Step V, if 92 is related to sit and 81 is related to not, similarly, 68


is related to which element?
a) ink
b) flow
c) but
d) sit
e) none of these

23.

In Step II, if not is related 92, 51 is related to 68, van is related to


sit, in the same way flow is related to which element?
a) 81
b) who
c) 19
d) 32
e) 23

Directions (Q. 24-28) Study the given information and answer the following
questions.
When a word and number arrangement machine is given an input line of
words and numbrs, it arranges them following a particular rule. The following is an
illustration of input and rearrangement. (All the numbers are two digit numbers)
Input
Step I
Step II
Step III
Step IV
Step V
Step VI
Step VII

40 made butter 23 37 cookies salt extra 52 86 92 fell now 19


butter 19 40 made 23 37 cookies salt extra 52 86 92 fell now
cookies 23 butter 19 40 made 37 salt extra 52 86 92 fell now
extra 37 cookies 23 butter 19 40 made salt 52 86 92 fell now
fell 40 extra 37 cookies 23 butter 19 made salt 52 86 92 now
made 52 fell 40 extra 37 cookies 23 butter 19 salt 86 92 now
now 86 made 52 fell 40 extra 37 cookies 23 butter 19 salt 92
salt 92 now 86 made 52 fell 40 extra 37 cookies 23 butter 19
54

REASONING MADE EASY www.BankExamsToday.com


Step VII is the last step of the above arrangement as the intended
arrangement is obtained.
As per the rules followed in the given steps, find out the appropriate steps
for the given input.
Input
32 proud girl beautiful 48 55 97 rich family 61 72 17 nice life
24.

How many steps will be required to complete the given input?


a) Five
b) Six
c) Seven
d) Eight
e) Nine

25.

Which of the following is the third element from the left end of step VI?
a) Beautiful
b) Life
c) 61
d) Nice
e) 17

26.

Which of the following is step III of the given input?


a) proud 72 girl 48 family 32 beautiful 17 55 97 rich 61 nice life
b) life 55 girl 48 family 32 beautiful 17 proud 97 r ich 61 72 n icd)e
c) girl 48 family 32 beautiful 17 proud 55 97 rich 61 72 nice life
d) family 32 beautiful 17 proud girl 48 55 97 rich 61 72 nice life
e) girl 48 life 55 family 32 beautiful 17 proud 97 rich 61 72 nice

27.

What is the position of nice from the left end in the final step?
a) 5th
b) 6th
c) 7th
th
th
d) 8
e) 9

28.

Which element is third to the right of family in step V?


a) Beautiful
b) 17
c) Proud
d) 18
e) None of these

Directions (Q. 29-33) Study the given information and answer the following
questions.
When a word and number arrangement machine is given an input line of
words and numbers, it arranges them following a particular rule. The following is an
illustration of an input and rearrangement.
Input: 45 navel change 33 48 down town frown 62 88 98 gesture orange 21
Step I: 45 navel change 33 48 down frown 62 88 98 gesture orange town 21
Step II: 45 navel change 48 down frown 62 88 98 gesture town 21 orange 33
Step III: change 48 down frown 62 88 98 gesture t own 21 orange 33 navel 45
Step IV: change down frown 62 88 98 town 21 orange 33 navel 45 gesture 48
Step V: change down 88 98 town 21 orange 33 navel 45 gesture 48 frown 62
Step VI: change 98 town 21 orange 33 navel 45 gesture 48 frown 62 down 88
Step VII: town 21 orange 33 navel 45 gesture 48 frown 62 down 88 change 98

55

REASONING MADE EASY www.BankExamsToday.com


Step VII is the last step of the above arrangement as the intended arrangement is
obtained. As per the rules followed inj the given steps, find out the appropriate steps
for the given input.
Input: 35 quant hear cute 50 65 98 silent giant 71 82 19 Oliver melody
29.

How many steps will be required to complete the given input?


a) Five
b) Six
c) Seven
d) Eight
e) None of these

30.

Which of the following is the fourth element from the left end of Step VI?
a) cute
b) melody
c) 71
d) oliver
e) None of these

31.

What is the position of melody from the left end in the final step?
a) Sixth
b) Seventh
c) First
d) Fifth
e) Ninth

32.

Which element is fourth to the right of giant in Step V?


a) cute
b) 19
c) quant
d) 98
e) 35

33.

Which of the following is Step III of the given input?


a) quant 82 hear 50 giant 35 cute 19 65 98 silent 71 oliver melody
b) hear cute 65 98 giant 71 82 melody silent 19 quant 35 oliver 50
c) hear cute 98 giant 71 82 silent 19 quant 35 oliver 50 melody 65
d) hear cute 98 65 giant 71 82 melody silent 19 quant 35 oliver 50
e) None of these

Directions (Q. 34-38) Study the following information to answer the given
questions.
A word and number arrangement machine when given an input line of
words and numbers rearranges them following a particular rule. The following is an
illustration of input and rearrangement. (All the numbers are two-digit numbers.)
Input: yogurt jovial 48 cinema total 20 correct sunny thin 78
Step I: 20 total yogurt jovial 48 cinema correct sunny thin 78
Step II: 20 total 48 jovial yogurt cinema correct sunny thin 78
Step III: 20 total 48 jovial 78 thin yogurt cinema correct sunny
Step IV: 20 total 48 jovial 78 thin yogurt sunny cinema correct
Step V: 20 total 48 jovial 78 thin yogurt sunny correct cinema

56

REASONING MADE EASY www.BankExamsToday.com


Step V is the last step of the rearrangement. As per the rules followed in the above
steps, find out in each of the following questions the appropriate step for the
following input.
Input: united madrid 89 chair 21 aircraft liverpool beneath honey 11 everyday
34.

Which step number will be the following output?


11 honey 21 chair 89 madrid united aircraft liverpool beneath everyday
a) Step II
b) Step III
c) Step IV
d) There will be no such step
e) None of these

35.

How many steps will be required to get the final output?


a) Five
b) Six
c) Seven
d) Four
e) None of these

36.

Which word/number would be the fifth from the right in Step V?


a) Madrid
b) United
c) 89
d) Liverpoor
e) None of these

37.

What is the position of beneath in Step IV?


a) 9th from the left
b) 11th from the left
c) 2nd from the right
d) 4th from the right
e) None of these

38.

If in a certain way honey is related to aircraft and chair is related to


beneath then madrid would be related to which of the following in the
last step?
a) 21
b) United
c) Everyday
d) Liverpool
e) None of these

Directions (Q. 39-43) Given an input line the machine arranges the words and
numbers in steps in a systematic manner as illustrated below:
Input:
Step I:
Step II:
Step III:
Step IV:
Step V:
Step VI:

56 dress fine shine 32 66 72 offer


72 56 dress fine shine 32 66 offer
72 shine 56 dress fine 32 66 offer
72 shine 66 56 dress fine 32 offer
72 shine 66 offer 56 dress fine 32
72 shine 66 offer 56 fine dress 32
72 shine 66 offer 56 fine 32 dress
57

REASONING MADE EASY www.BankExamsToday.com

Step VI is the last step and the output in Step VI I is the final output.
As per the rules followed in the above steps, find out in each of the following
questions the appropriate step for the given input.
39.

Step IV of an input is 62 sound 56 sleep roam present 33 49. What willl


be the input definitely?
a) sound 62 sleep 56 roam present 33 49
b) sleep sound 62 56 roam present 33 49
c) 62 sound sleep 56 roam present 33 49
d) Cannot be determined
e) None of these

40.

Which of the following will be the third step for input:


jockey firm 36 43 growth chart 22 45 ?
a) 45 jockey 43 growth firm 36 chart 22
b) 45 jockey 43 firm growth 36 chart 22
c) 45 jockey 43 growth 36 firm chart 22
d) 45 jockey 43 firm 36 growth chart 22
e) None of these

41.

Step II of an input is 53 wi ndow 42 50 door lock key 36. How many


more steps will be required to complete the arrangement?
a) Three
b) Four
c) Five
d) Six
e) None of these

42.

What will be the fifth step of an input whose first step is 85 journey train
36 54 daily 28 mansion?
a) 85 train 54 mansion 28 journey daily 36
b) 85 train 54 mansion journey 36 daily 28
c) 85 train 54 mansion 36 journey daily 28
d) There is no such step
e) None of these

43.

Which step will be the last step for an input whose second step is 63 sour
18 56 grapes healthy 32 rise?
a) IV
b) V
c) VIII
d) VII
e) None of these

Directions (Q. 44-49) Study the following information carefully and answer the
given questions.
A word and number arrangement machine when given an input line of
words and numbers rearranges them following a particular rule in each step. The
following is an illustration of input and rearrangement.
58

REASONING MADE EASY www.BankExamsToday.com

Input:

shop 17 table 20 53 oven desk 39

Step I:
Step II:
Step III:
Step IV:
Step V:

17 shop table 20 53 oven desk 39


17 table shop 20 53 oven desk 39
17 table 20 shop 53 oven desk 39
17 table 20 shop 39 53 oven desk
17 table 20 shop 39 oven 53 desk

And step V is the last step of the rearrangement.


As per the rules followed in the above steps, find out in each of the following
questions that appropriate step for the given input.
44.

Step II of an input is : 15 yes 62 51 48 talk now gone


Which of the following will be step VI?
a) 15 yes 48 talk 51 now gone 62
b) 15 yes 48 talk 51 62 now gone
c) 15 yes 48 talk 51 now 62 gone
d) There will be no such step
e) None of the above

45.

Step III of an input is: 21 victory 30 joint 64 47 all gone


How many more steps will be required to complete the rearrangement?
a) Three
b) Four
c) Five
d) Six
e) None of these

46.

Input : 89 bind 32 goal house 61 12 joy


How many steps will be required to complete the arrangement?
a) Four
b) Five
c) Six
d) Seven
e) None of these

47.

Input:
save 21 43 78 them early 36 for
Which of the following steps will be the last but one?
a) VI
b) VII
c) VIII
d) V
e) None of these

48.

Input: desire 59 63 all few 38 46 zone


How many steps will be required to complete the rearrangement?
a) Four
b) Five
c) Six
d) Seven
e) None of these

49.

Input: win 92 task 73 59 house range 34


Which of the following will be step IV of the above input?
a) 34 win 59 task 73 range 92 house
b) 34 win 92 59 task 73 house range
c) 34 win 92 task 73 59 house range
59

REASONING MADE EASY www.BankExamsToday.com


d) There will be no such step
e) None of the above
Directions (Q. 50-54) Study the following information carefully and answer the
given questions.
A word and number arrangement machine when given an input line of
words and numbers rearranges them following a particular rule in each step. The
following is an illustration of input and rearrangement.
Input:
Step I:
Step II:
Step III:
Step IV:
Step V:

base 35 or gone 62 49 87 ahead


87 base 35 or gone 62 49 ahead
87 ahead base 35 or gone 62 49
87 ahead 62 base 35 or gone 49
87 ahead 62 base 49 35 or gone
87 ahead 62 base 49 gone 35 or

And Step V is the last step of the rearrangement. As per the rules followed in the
above steps, find out in each of the following question the appropriate step for the
given input.
50.

Input: how was your stay 56 25 36 64


Which of the following will be step VI?
a) 64 how 56 was your stay 25 36
b) 64 how 56 stay 36 was 25 your
c) 64 how 56 stay 36 was your 25
d) There will be no such step
e) None of the above

51.

Input:
power fail now 52 24 75 gate 34
Which of the following steps will be the last but one?
a) IV
b) V
c) VI
d) VII
e) None of these

52.

Step III of an input is: 91 car 85 14 27 few new house


Which of the following is definitely the input?
a) 85 14 91 car 27 few new house
b) car 91 85 14 27 few new house
c) car 85 14 27 few new house 91
d) Cannot be determined
e) None of the above

53.

Step II of an input is: 75 down 16 24 farm eager 62 sky


How many more steps will be required to complete the rearrangement?
a) Four
b) Five
c) Six
d) Seven
e) None of these
60

REASONING MADE EASY www.BankExamsToday.com


54.

Input: 14 35 when they same 61 48 home


How many steps will be required to complete the rearrangement?
a) Four
b) Five
c) Six
d) Seven
e) None of these

Directions (Q. 55-60) A word and number aran gement machine when given an
input line of words and numbers rearranges them following a particular rule
in each step. The following is an illustration of input and rearrangement.
Input:
Step I:
Step II:
Step III:
Step IV:
Step V:
Step VI:

but 32 71 glory fair south 65 84


south but 32 71 glory fair 65 84
south 84 but 32 71 glory fair 65
south 84 glory but 32 71 fair 65
south 84 glory 71 but 32 fair 65
south 84 glory 71 fair but 32 65
south 84 glory 71 fair 65 but 32

And Step VI is the last step of the rearrangement.


As per the rules followed in the above steps, find out in each of the following
questions the appropriate step for the given input.
55.

Step III of an input is: year 92 ultra 15 23 strive house 39. How many more
steps will be required to complete the rearrangement?
a) Three
b) Four
c) Two
d) Five
e) None of these

56.

Input:
any how 49 24 far wide 34 69
Which of the following steps will be the last but one?
a) VI
b) VII
d) VIII
e) None of these

57.

Step II of an input is: town 74 pair 15 31 nice job 42


Which of the following is definitely the input?
a) pair 15 31 town nice job 42 74
b) pair 15 town 31 74 nice job 42
c) pair 15 town 74 31 nice job 42
d) Cannot be determined
e) None of the above

58.

Input:
play over 49 37 12 match now 81
Which of the following will be step IV?
a) play 81 over 49 37 match now
b) play 8 over 49 37 12 match now
c) play 81over 49 now 37 match 12
d) There will be no such step

c) V

61

REASONING MADE EASY www.BankExamsToday.com


e) None of the above
59.

Step II of an input is: war 58 box cart 33 49 star 24


Which of the following steps will be the last?
a) V
b) VI
c) IV
d) VII
e) None of these

60.

Input: shower fall water 34 51 67 98 goal


How many steps will be required to complete the rearrangement?
a) Three
b) Four
c) Six
d) Five
e) None of these

Directions (Q. 61-66) Study the following information carefully and answer the
given questions.
A word and number arrangement machine when given an input line of words and
numbers rearranges them following a particular rule in each step. The following is
an illustration of input and rearrangement.
Input:
Step I:
Step II:
Step III:
Step IV:
Step V:

joy far 35 27 16 96 height star


96 joy far 35 27 16 height star
96 far joy 35 27 16 height star
96 far 35 joy 27 16 height star
96 far 35 height joy 27 16 star
96 far 35 height 27 joy 16 star

And step V is the last step of the rearrangement.


As per the rules followed in the abov steps, find out in each of the following
questions the appropriate step for the given input.
61.

Input:
organize 19 12 stable room 35 72 house
How many steps will be required to complete the arrangement?
a) Five
b) Six
c) Seven
d) Four
e) None of the above

62.

Input:
bake never store 51 26 33 age 49
Which of the following will be step V?
a) 51 age 49 bake 33 never 26 store
b) 51 age 49 bake never store 26 33
c) 51 age bake never store 26 33 49
d) 51 bake never store 26 33 age 49
e) There will be no such step

63.

Input: always go there 39 62 47 time 24


Which of the following steps will be the last but one?
62

REASONING MADE EASY www.BankExamsToday.com


a) VI
d) IX

b) VII
e) None of these

c) VIII

64.

Step II of an input is: 67 ask 34 12 46 for my date


Which o f the f ollowing is definitely the input?
a) 34 12 46 for my date ask 67
b) 34 12 46 for my date 67 ask
c) 12 34 67 ask 46 for my date
d) Cannot be determined
e) None of the above

65.

Step III of an input is: 84 for 56 29 17 won loss game


Which of the following steps will be the last?
a) VIII
b) IX
c) VII
d) V
e) None of these

66.

Step III of an input is: 86 box 63 18 gear card 51 new


How many more steps will be required to complete the arrangement?
a) Three
b) Two
c) Four
d) Five
e) None of these

Directions (Q. 67-71) Given an input line the machine rearranges them step by
step as per the illustration given below.
Input:
Step I:
Step II:
Step III:
Step IV:
Step V:
Step VI:
Step VII:

show must 73 85 go 69 on 32
32 show must 73 85 go 69 on
32 go show must 73 85 69 on
32 go 69 show must 73 85 on
32 go 69 must show 73 85 on
32 go 69 must 73 show 85 on
32 go 69 must 73 on show 85
32 go 69 must 73 on 85 show

Step VII is the last step and the arrangement in Step VII is the final arrangement.
As per the rules followed in the above steps, find out in each of the following
questions the appropriate step for the given input.
67.

If the third step of an input is 14 but 26 not just 63 fundamental 47, which
of the following is definitely the input?
a) 26 14 but not just 63 fundamental 47
b) 63 26 but 14 not just 63 fundamental 47
c) fundamental 26 but not just 63 47
d) Cannot be determined
e) None of the above

63

REASONING MADE EASY www.BankExamsToday.com


68.

First step of an input is 34 fire dress well 63 43 prime 52. Which step is
the last but one?
a) IV
b) III
c) VI
d) VII
e) V

69.

What will be the fourth step for the following input?


Input:
just in time for 36 48 14 59
a) 14 for 36 in just time 48 59
b) 14 for 36 just in time 48 59
c) 14 for 36 in 48 just time 59
d) Cannot be determined
e) None of the above

70.

If the second step of an input is 23 fine 49 sixty rely 38 56 next, how


many more steps will be r equired t o complete the arrangement?
a) 2
b) 5
c) 6
d) 4
e) None of these

71.

If the second step for an input is 17 do foreign 95 74 heights mountain


29, what will be the fifth step?
a) 17 do 29 foreign heights 74 95 mountain
b) 17 do 29 foreign 74 mountain heights 95
c) 17 do 29 foreign 74 heights 95 mountain
d) There will be no such step
e) None of the above

Answers:
1.
2.
3.
4.
5.
6.
7.
8.
9.
10.
11.
12.
13.
14.
15.
16.

Option C
Option E
Option A
Option B
Option D
Option D
Option A
Option B
Option C
Option E
Option B
Option B
Option B
Option B
Option A
Option D
64

REASONING MADE EASY www.BankExamsToday.com


17.
18.
19.
20.
21.
22.
23.
24.
25.
26.
27.
28.
29.
30.
31.
32.
33.
34.
35.
36.
37.
38.
39.
40.
41.
42.
43.
44.

Option C
Option C
Option C
Option B
Option A
Option A
Option D
Option C
Option D
Option C
Option A
Option B
Option C
Option E
Option C
Option C
Option B
Option B
Option B
Option B
Option C
Option C
Option D
Option A
Option B
Option C
Option E
Option C
Step II:
Step III:
Step IV:
Step V:
Step VI:

45.

15 yes 62 51 48 talk now gone


15 yes 48 62 51 talk now gone
15 yes 48 talk 62 51 now gone
5 yes 48 talk 51 62 now gone
15 yes 48 talk 51 now 62 gone

Option E
Step III:
21 victory 30 joint 64 47 all gone
Step IV:
21 victory 30 joint 47 64 all gone
Step V:
21 victory 30 joint 47 gone 64 all
So, step V is the last step. Hence, two steps will be required.

46.

Option C
Input:
Step I:

89 bind 32 goal house 61 12 joy


12 89 bind 32 goal house 61 joy
65

REASONING MADE EASY www.BankExamsToday.com


Step II:
12 joy 89 bind 32 goal house 61
Step III:
12 joy 32 89 bind goal house 61
Step IV:
12 joy 32 house 89 bind goal 61
Step V:
12 joy 32 house 61 89 bind goal
Step VI:
12 joy 32 house 61 goal 89 bind
Step VI is the last step. So, to complete the arrangement, six steps will be
required.
47.

Option E
Input:
save 21 43 78 them early 36 for
Step I:
21 save 43 78 them early 36 for
Step II:
21 them save 43 78 early 36 for
Step III:
21 them 36 save 43 78 early for
Step IV:
21 them 36 save 43 for 78 early
Step IV is the last step and from last step first one is step IV.

48.

Option B
Input:
desire 59 63 all few 38 46 zone
Step I:
38 desire 59 63 all few 46 zone
Step II:
38 zone desire 59 63 all few 46
Step III:
38 zone 46 desire 59 63 all few
Step IV:
38 zone 46 few d esire 59 63 all
Step V:
38 zone 46 few 59 desire 63 all
So, the last step is step V.

49.

Option E
Input:
Step I:
Step II:
Step III:
Step IV:

50.

win 92 task 73 59 house range 34


34 win 92 task 73 59 house range
34 win 59 92 task 73 house range
34 win 59 task 92 73 house range
34 win 59 task 73 92 house range

Option D
Input:
how was your stay 56 25 36 64
Step I:
64 how was your stay 56 25 36
Step II:
64 how 56 was your stay 25 36
Step III:
64 how 56 stay was your 25 36
Step IV:
64 how 56 stay 36 was your 25
Step V:
64 how 56 stay 36 was 25 your
So, sixth step is not possible because fifth step will be last step.

51.

Option D
66

REASONING MADE EASY www.BankExamsToday.com

Input:
power fail now 52 24 75 gate 34
Step I:
75 power fail now 52 24 gate 34
Step II:
75 fail power now 52 24 gate 34
Step III:
75 fail 52 power now 24 gate 34
Step IV:
75 fail 52 gate power now 24 34
Step V:
75 fail 52 gate 34 power now 24
Step VI:
75 fail 52 gate 34 now power 24
Step VII:
75 fail 52 gate 34 now 24 power
So, the first step from the last is step (VII).
52.

Option D
Input cannot be determined.

53.

Option A
Step II:
75 down 16 24 farm eager 62 sky
Step III:
75 down 62 16 24 farm eager sky
Step IV:
75 down 62 eager 16 24 farm sky
Step V:
75 down 62 eager 24 16 farm sky
Step VI:
75 down 62 eager 24 farm 16 sky
Four more steps are required to complete the rearrangement.

54.

Option C
Input:
14 35 when they came 61 48 home
Step I:
61 14 35 when they came 48 home
Step II:
61 came 14 35 when they 48 home
Step III:
61 came 48 14 35 when they h ome
Step IV:
61 came 48 home 14 35 when they
Step V:
61 came 48 home 35 14 when they
Step VI:
61 came 48 home 35 they 14 when
To complete the rearrangement 6 steps are required.

55.

Option B
Step III:
year 92 ultra 15 23 strive house 39
Step IV:
year 92 ultra 39 15 23 strive house
Step V:
year 92 ultra 39 strive 15 23 house
Step VI:
year 92 ultra 39 strive 23 15 house
Step VII:
year 92 ultra 39 strive 23 house 15
So, four more steps are required.

56.

Option C
67

REASONING MADE EASY www.BankExamsToday.com


Input:
any how 49 24 for wide 34 69
Step I:
wide any how 49 24 for 34 69
Step II:
wide 69 any how 49 24 for 34
Step III:
wide 69 how any 49 24 for 34
Step IV:
wide 69 how 49 any 24 for 34
Step V:
wide 69 how 49 for any 24 34
Step VI:
wide 69 how 49 f or 34 any 24
So, step V is first step from last.
57.

Option D
Step II: town 74 pair 15 31 nice job 42
Input cannot be determined.

58.

Option D
Input:
play over 49 37 12 match now 81
Step I:
play 81 over 49 37 12 match now
Step II:
play 81 over 49 now 37 12 match
Step III:
play 81 over 49 now 37 match 12
th
4 step cannot be determined.

59.

Option B
Step II:
war 58 box cart 33 49 star 24
Step III:
war 58 star box cart 33 49 24
Step IV:
war 58 star 49 box cart 33 24
Step V:
war 58 star 49 cart box 33 24
Step VI:
war 58 star 49 cart 33 box 24
So, last step is step VI.

60.

Option D
Input:
shower fall water 34 51 67 98 goal
Step I:
water shower fall 34 5167 98 goal
Step II:
water 98 shower fall 34 51 67 goal
Step III:
water 98 shower 67 fall 34 51 goal
Step IV:
water 98 shower 67 goal fall 34 51
Step V:
water 98 shower 67 goal 51 fall 34
Five steps required to complete the rearrangement.

61.

Option D
Input:
Step I:
Step II:

organize 19 12 stable room 35 72 house


72 organize 19 12 stable room 35 house
72 house organize 19 12 stable room 35
68

REASONING MADE EASY www.BankExamsToday.com


Step III:
72 house 35 organize 19 12 stable room
Step IV:
72 house 35 organize 19 room 12 stable
So, four steps will required to complete the arrangement.
62.

Option A
Input:
Step I:
Step II:
Step III:
Step IV:
Step V:

63.

Option E
Input:
Step I:
Step II:
Step III:
Step IV:

64.

bake never store 51 26 33 age 49


51 bake never store 26 33 age 49
51 age bake never store 26 33 49
51 age 49 bake never store 26 33
51 age 49 bake 33 never store 26
51 age 49 bake 33 never 26 store

always go there 39 62 47 time 24


62 always go there 39 47 time 24
62 always 47 go there 39 time 24
62 always 47 go 39 there time 24
62 always 47 go 39 there 24 time

Option D
Input cannot be determined by given step.

65.

Option D
Step III:
84 for 56 29 17 won loss game
Step IV:
84 for 56 game 29 17 won loss
Step V:
84 for 56 game 29 loss 17 won
So, the last step is step V.

66.

Option A
Step III:
86 box 63 18 gear card 51 new
Step IV:
86 box 63 card 18 gear 51 new
Step V:
86 box 63 card 51 18 gear new
Step VI:
86 box 63 card 51 gear 18 new
So, to complete the arrangement three steps are required.

67.
68.
69.
70.
71.

Option D
Option C
Option A
Option D
Option C
69

REASONING MADE EASY www.BankExamsToday.com

Chapter - 3
Data Sufficiency
Directions (Q. 1-3) Each of the questions below consists of a question and three
statements numbered I, II and III given below it. You have to decide whether
the data provided in the statements are suficient to answer the question.
Give Answer:
1.
How many daughters does W have?
I.
B and D are sisters of M.
II.
Ms father T is husband of W
III.
Out of the three children which T has, only one is a boy.
a) Only I and III
b) All I, II and III
c) Only II and III
d) Question cannot be answered even with all I, II and III
e) Only I and II
2.

Who among A, B, C, D, E and F each having a different height, is the


tallest?
I.
B is taller than A but shorter than E.
II.
Only two of them are shorter than C.
III.
D is taller than only F.
a) Only I and II
b) Only I and III
c) Only II and III
d) All I, II and III are required to answer the question
e) All I, II and III are not sufficient to answer the question

3.

Towards which direction is village J from village W?


I.
Village R is to the west of village W and to the north of village T.
II.
Village Z is to the east of village J and to the South of village T.
III.
Village M is to the noirth east of Village J and north of Village Z.
a) Only III
b) Only II and III
c) All I, II and III are required to answer the question
d) Question cannot be answered even with all I, II and III
e) None of these

70

REASONING MADE EASY www.BankExamsToday.com


Directions (Q. 4-8) Below is given a passage followed by several possible
inferences which can drawn from the facts stated in the passage. You have to
examine each inference separately in the context of the passage and decide
upon its degree of truth or falsity.
Give answer:
a)
If the inference is definitely true i.e. it properly from the statement of facts
given.
b)
If the inference is probably true through not definitely true in the light of
the facts given.
c)
If the data are inadequate, i.e. from the facts given you cannot say whether
the inference is likely to be true or false.
d)
If the inference is probably false, though not definitely false in the light
of the facts given.
e)
If the inference is definitely false, i.e. it cannot possibly be drawn from
the facts given or it contradicts the given facts.
Between 2002-03 and 2006-2007, Indian economy grew annualy at 8.7%
led by the services sector at 9% per year. In 1990, Indias share of services, at 40%
of GDP was consistent with its per capita income for low-income country. By 2001
its share of one-half of the GDP was higher by five percentage points, compared to
the average for low-income countries. Economic reforms that energized the private
corporate sector and technological changes that opened up new vistas in
telecommunications, IT and outsourcing are believed to be responsible for th e
impressive performance. However, the services led growth remains a puzzle at a
low per capita income, with 55% of the work force still engaged in agriculture, and
when agriculture decelerated and industry stagnated-3 defying a styled fact in
economics.
4.

India has now emerged as a high per capita income country.

5.

Growth in Indias services sector post 2005 is more than 9%.

6.

Less than half of total workforce is engaged in Agricultural sector in India.

7. Share of services sector in Indias GDP has crossed the half way mark in early
2000.
8. In early minutes, the share of services sector in GDP for low per capita income
group of countries is about 40%

71

REASONING MADE EASY www.BankExamsToday.com

Directions (Q. 9-13) below are given a passage followed by several possible
inferences which can be drawn from the facts stated in the passage. You have
to examine each in ference separately in the context of the passage and decide
upon its degree of truth or falsity.
Give answer:
a)
If the inference is definitely true i.e. it properly follows from the
statements of facts given.
b)
If the inference is probably true thuogh not definitely true in the light of
the facts given.
c)
If the data are inadequate i.e. from the facts given you can not say whether
the inference is likely to be true or false.
d)
If the inference is probably false though not definitely false in the light
of the facts given.
e)
If the inference is definitely false i.e. it cannot possibly be drawn from the
facts given or it contradicts the given facts.
A recent report that satellite data shws groundwater levels in northern India
depleting by as much as a foot per year, over the past decade, is a matter of concern.
The clear writing on the wall is that India faces a turbulent water future and
veritable crisis without proactive policy and sustainable practices. Besides, the poor
monsoon this season and the resultant drought situation pan-India, calls series of
glaring anomalies do need to be addressed. For one, theres far too much reliance on
groundwater. For another, out water infrastructure for storage and supply is sorely
inadequate. Worse, policy distortions in artificially under pricing key agri-inputs
like power have perversely incentivised cultivation of water intensive crops like
paddy in traditionally wheat growing areas.
9. India has failed to take measures to recharge groundwater adequately in
northern part.
10.

Wheat cultivation in India requires comparatively more water than paddy.

11.

Water level in other parts of India is stable during the last decade.

12. India has now put in place a system to reduce over dependence on groundwater.
13. Adequate monsoon helps in drawing less ground water for cultivation and thus
preserve balance.
In making decisions about important questions, it is desirable to be able to
distinguish between strong arguments and weak arguments. Strong
arguments must be both important and directly related to the question. Weak
72

REASONING MADE EASY www.BankExamsToday.com


arguments may not be directly related to the question and may be of minor
importance or m ay be related to the trivial aspects of the question.
Directions (Q. 14-18) Each of the questions below consists of a question and
two statements numb ered I and II given below it. You have to decide whether
the data provided in the statements are sufficient to answer the question. Read
both the statements and give answer:
a)
b)
c)
d)
e)

If the data in Statement I alone are sufficient to answer the question, while
the data in Statement II alone are not sufficient to answer the question.
If the data in Statement II alone are sufficient to answer the question, while
the data in Statement I alone are not sufficient to answer the question.
If the data either in Statement I alone or in Statement II alone are sufficient
to answer the question.
If the data even in both Statements I and II together are not sufficient to
answer the question.
If the data in both Statements I and II together are necessary to answer the
question.

14.

What is the position of point F with respect to point I?


I.
Point G is 5 km east of point F. Point S is 5 km north of point G.
Point H is the midpoint of points G and S. Point I is to the south of
point H in such a manner that point G is the midpoint of points H
and I.
II.
Point A is 10 km east of point F. Point B is 5 km south of point A.
Point H is the midpoint of points A and B. Point I is 5 km south of
point H. Point I is to the east of point L at a distance of 5 km.

15.

On which day of a week Lalita did her reservation for the journey?
I.
The husband of Lalita correctly remembers that she has done
reservation after Monday but before Sunday. However, it is almost
certain that she did not make her reservation on Wednesday.
II.
The daughter of Lalita correctly remembers that h er mother has
done her reservation after Tuesday but before Saturday. Since her
mother was busy on Thursday so she did not make her reservation
on Thursday.

16.

Seven persons - P, Q, R, S, T, U and V are standing in a straight line facing


towards north. Find the position of S from the right end.
I.
P is standing on any of the extreme ends of the line. There are two
persons between P and R. There are two persons between R and Q.
U is not standing to the immediate left of R.
II.
R is standing exactly at the centre. T is standing exactly between R
and V. There is only one person between T and U. S is standing to
the immediate right of U.
73

REASONING MADE EASY www.BankExamsToday.com


17.

How are N and D related?


I.
A is the sister of D.
II.
D is the mother of R who is the only son of N.

18.

Who amongst Ronak, Sanjay, Anish and Rohan is the tallest?


I.
Ronak is as tall as Sanjay.
II.
Anish is taller than Rohan.

Answers:
1.
2.
3.
4.
5.
6.
7.

8.
9.
10.
11.
12.
13.
14.
15.

16.
17.

18.

Option C
Option C
Option E
Option A
Option B
Option D
Option A
It is true as given in second line of the passage In 1990, Indias share of
services at 40% of GDP ____
Option D
Option D
Option B
Option A
Option E
Option E
Option C
Option E
From Statement I
Lalita made her reservation on either Tuesday, Thursday, Friday or
Saturday.
From Statement II
Lalita made her reservation on Wednesday or Friday.
From both the statements Lalita made her reservation on Friday.
Option B
Option B
From statement I
A is the sister of D. There is no information about N.
From statement II
D is the mother of R.
R is the son of N.
So, N is husband of D.
Option D
From statement I
Ronak = Sanjay
From statement II
74

REASONING MADE EASY www.BankExamsToday.com


Anish > Rohan
From both the statements
Anish > Rohan
Ronak = Sanjay

75

REASONING MADE EASY www.BankExamsToday.com

Chapter - 4
Sitting Arrangements
Directions (Q. 1-5) Study the following information to answer the given
questions.
i)
ii)
iii)
iv)
1.

2.

There are 9 friends A, B, C, D, E, F, G, H seated in a circle facing the


centre.
AC, DG, HE and FB are seated adjacent to each other. A is also seated
adjacent to H.
B is 2nd to the right of H.
E is 3rd to the right of C.
Who is 2nd to the left of A?
a) D
d) Cannot be determined

b) G
e) None of these

c) F

Who is 3rd to the left of C?


a) G
d) Cannot be determined

b) D
e) None of these

c) B

3.

What is Cs position with reference to E?


a) 5th to the right
b) 4th to the left
c) 4th to the right
d) 3rd to the right
e) Cannot be determined

4.

Who is 2nd to the right of A?


a) B
d) Cannot be determined

5.

b) E
e) None of these

c) F

Who among the following pairs may not be seated adjacent to each other?
a) AH
b) DC
c) EB
d) Cannot be determined
e) None of these

Directions (Q. 6-11) Read the following information carefully and answer the
questions that follow.
Seven friends A, B, C, D, E, F and G are sitting around a circular table
facing either the centre or outside. Each one of them belongs to a different
department viz. Finance, Marketing Sales, HR, Corporate Finance, Investment
Banking and Operations but not necessarily in the same order.
C sits third to the right of G. G faces the centre. Only one person sits
between C and the person working in the HR department immediate neighbours of
C face outside. Only one person sits between F and D. Both F and D face the centre.
D does not work in the HR department. A works in Investment Banking
76

REASONING MADE EASY www.BankExamsToday.com


Department. A faces the centre. Two people sit between the persons who work in
Investment Banking and Marketing Departments. The person who works in
Corporate Finance sits to the immediate left of E. C faces same direction as E. The
person who works in corporate finance sits to the immediate left of the person who
works for Operations Department.
6.

For which of the following departments does B work?


a) Finance
b) Marketing
c) HR
d) Corporate Finance e) Operationsd

7.

What is position of B with respect to the person who works for Sales
Department?
a) Immediate right
b) Third to the left
c) Second to the right
d) Second to the left
e) Fourth to the right

8.

Who sits to the immediate right of E?


a) The person who works for Marketing Department
b) C
c) B
d) The person who works for HR Department
e) A

9.

Who amongst the following sits exactly between C and the person who
works for HR Department?
a) B
b) The person who works for Marketing Department
c) The person who works for Operations Department
d) D
e) G

10.

Who amongst the following sit between the persons who work for
Marketing and Investment Banking departments when counted for the left
hand side of the person working for Marketing Department?
a) F and G
b) E and C
c) C and B
d) F and D
e) B and D

11.

How many people sit between the person who workds for Operations
Department and A, when counted from the right hand side of A?
a) One
b) Two
c) Three
d) Four
e) None of these

Directions (Q. 12-16) Read the following information carefully to answer the
given questions.
Eight persons from different banks viz., Bank of India, Punjab National
Bank, Canara Bank, Bank of Baroda, Oriental Bank of Commerce, Dena Bank,
77

REASONING MADE EASY www.BankExamsToday.com


Union Bank of India and Bank of Maharashtra are sitting in two parallel rows
containing four people each, in such a way that there is an equal distance between
adjacent persons. The names of these persons are A, B, C, D, E, F, G and H. Each
members seated in a row faces another member of the other row. (All the
information given above does not necessarily represents the order of seating as in
the final arrangement)
A faces north and seated second to the left of the person, who is from
Punjab National Bank. The person from Oriental Bank of Commerce is
opposite to one who is second to the left of H.
The person from Bank of Baroda is not near to H and F and not at the end
of the row. The person from Canara Bank is opposite to the person who is
from Dena Bank.
The person from Bank of India is not at the end of the row but left of the
person who is from Canara Bank.
D is immediately right of the person who is from Bank of Maharashtra. The
person from Canara Bank faces north while D faces south and opposite to
B.
The person from Union Bank of India is not opposite to E. The person from Canara
Bank is opposite to C.
12.

Which of the following is true regarding E?


a) The person from Bank of Maharashtra faces E
b) The person from Dena Bank is an immediate neighbour of E
c) The person who is second to the right of G faces E
d) E is from Union Bank of India
e) E sits at one of the extreme end of the row

13.

Who is seated between G and the person from Bank of Maharashtra?


a) The person from Bank of Baroda
b) H
c) F
d) E
e) The person from Dena Bank

14.

Who amongst the following sit at extrememe end of the row?


a) A and the person from Bank of Maharashtra
b) The person from Punjab National Bank and Union Bank of India
c) The person from Dena Bank and E
d) The person from Union Bank of India and C
e) G and B

15.

Who amongst the following faces the person from Bank of Baroda?
a) The person from Bank of India
b) F
c) A
78

REASONING MADE EASY www.BankExamsToday.com


d) The person from Union Bank of India
e) The person from Oriental Bank of Commerce
16.

B is related to Dena Bank in the same way as F is related to Bank of


Baorda on the given arrangement, who amongst the following is G
related to following the same pattern?
a) Oriental Bank of Commerce
b) Punjab National Bank
c) Bank of India
d) Canara Bank
e) Union Bank of India

17.

Four of the following five are alike in a certain way based on the given
seating arrangement and thus form a group, which is the one that does not
belong to that group?
a) Canara Bank
b) C
c) Union Bank of India
d) Oriental Bank of Commerce
e) Bank of Maharashtra

18.

G is from which of the following banks?


a) Oriental Bank of Commerce
b) Dena Bank
c) Bank of Maharashtra
d) Canara Bank
e) Punjab National Bank

19.

Who amongst the following is from Punjab National Bank?


a) E
b) H
c) G
d) C
e) None of these

Directions (Q. 20-26) Study the following information carefully to answer the
questions that follows.
Eight persons namely A, B, C, D, E, F, G and H are sitting in a straight line
from East to West. Some are facing North direction and some are facing South
direction.
H sits to the third left of C and faces south. F is sitting at one of the extreme
ends and is facing north. B and G are facing in same direction and person sitting
between them is facing in opposite direction. A is sitting third from left end and is
facing South direction. A is sitting adjacent to E and both are facing in opposite
direction. D is sitting to the right of B and both are facing in same direction. C is
facing in North direction. A and D are not neighbours. Neighbors of H are facing in
same direction which is opposite to the direction of H is facing. E sits to the right of
A.
79

REASONING MADE EASY www.BankExamsToday.com

20.

Who is sitting between A and H?


a) F
b) E
b) B
e) None of these

c) G

21.

Which of the following pairs has the person facing the same direction?
a) E, A
b) G, H
c) A, G
d) H, D
e) B, D

22.

How many persons are facing in North direction?


a) 2
b) 3
c) 4
d) 5
e) 6

23.

Who is sitting third from the eastern end of the arrangement?


a) E
b) G
c) B
d) C
e) None of these

24.

How many persons are sitting between E and B?


a) 2
b) 3
c) 4
d) 5
e) 6

25.

Which one of the following statement is correct?


a) A sits between G and B
b) D sits to thesecond left of H
c) H and B are facing same direction
d) Person sitting to the left of A faces direction
e) All of the above

26.

Who sits 3rd to the left of A?


a) B
b) D
d) G
e) None of these

c) H

Directions (Q. 27-31) Study the following information and answer the question
given below.
Eight people E, F, G, H, J, K, L and M are sitting around a circular table
facing the centre. Each of them is of a different professio n Chartered Accountant,
Colomnist, Doctor, Engineer, Financial Analyst, Lawyer, Professor and Scientist
but not necessarily in the same order. F is sitting second to the left of K. The
Scientist is an immediate neighbour of K. There are only three people between the
Scientist and E. Only one person sits between the Engineer and E. The Coloumnist
is to t he immediate r ight of the Engineer. M is second to the right of K. H is the
Scientist. G and J are immediate neighbours of each other. Neither G nor J is an
Engineer. The Financjial Analyst is to the immediate left of F. The Lawyer is
second to the right of the Coloumnist. The Professor is an immediate neighbour of
the Engineer. G is second to the right of the Chartered Accountant.
80

REASONING MADE EASY www.BankExamsToday.com

27.

Who is sitting second to the right of E?


a) The Lawyer
b) G
c) The Engineer
d) F
e) K

28.

Who amongst the following is the Professor?


a) F
b) L
d) K
e) None of these

c) M

29.

Four of the following five are alike in a certain way based on the given
arrangement and hence from a group. Which of the following does not
belong to that group?
a) Chartered Accountant
b) M Doctor
c) J Engineer
d) Financial Analyst L
e) Lawyer - K

30.

What is the position of L with respect to the Scientist?


a) Third to the left
b) Second to the right
c) Second to the left
d) Third to the right
e) Immediate right

31.

Which of the following statements is true according to the given


arrangement?
a) The Lawyer is second to the left of the Doctor
b) E is an immediate neighbour of the Financial Analyst
c) H sits exactly between F and the Financial Analyst
d) Only four people sit between the Coloumnist and F
e) All of the given statements are true

Directions (Q. 32-36) Study the following information carefully and answer the
given questions:D, E, F, H and I are seated in a circle facing the centre. A, B and C are also
seated in the same circle but two of them are not facing the centre. F is second to the
left of C. E is third to the right of A. B is third to left of D, who is immediate
neighbor of H and I. C is second to the right of D and third to the right of B.
32.

Which of the following pairs is not facing the centre?


a) BC
b) AC
c) CA
d) BD
e) None of these

33.

What is the position of D with respect to F?


81

REASONING MADE EASY www.BankExamsToday.com


a) Second to the right
b) Third to the left
c) Fifth to the right
d) Cannot be determined
e) None of these
34.

Who among the following is second to the left of B?


a) A
b) H
c) I
d) Either H or I
e) None of these

35.

If H is on the immediate right of E then what is Hs position with respect to


C?
a) Third to the left
b) Third to the right
c) Fifth to the left
d) Cannot be determined
e) None of these

36.

Which of the following is/are the possible position(s) of I with respect to A?


a) Fourth to the right
b) Second to the left
c) Fourth to the left
d) Only a and c
e) All of the above

Directions (Q. 37-41) Study the following information and answer the questions
that follow:
Eight students A, B, C, D, E, F, G and H in a school sports competition
participated in a game in which they were sitting around a circulating. The seats of
the ring are not directed towards the centre. All the eight students are in four groups
I, II, IIII and IV, i.e two students in each group, but not necessarily in the same
order. These students are from different sport houses, viz. Maharishi Vyas,
Aryabhatt, Vashistha, Shankaracharya, Balmiki, Dhruv and Dayanand.

No two students of the same group are sitting adjacent to each other except
those of group III. Students from group IV are sitting opposite each other.
D is neither in Dayanand nor in Aryabhatt house.
The student from Dhruv house is sitting on the immediate right of the
students from Dayanand house.
C, who is in Vashistha house, is in group I. She is sitting on the immediate
right of F, who is in group III.
F is not in Aryabhatt house and she has also participated in other sports.
B from Vyas house is neither in group IV nor in group I or II.
82

REASONING MADE EASY www.BankExamsToday.com

37.

38.

B is sitting opposite E. Only Balmiki participant A is sitting b etween


Dhruv participaint E and t he Shankaracharya participant.
Both the students of group II are sitting adjacent to students of group IV.
Who is in Maharishi house?
a) G
b) F
d) Cant say
e) None of these

c) H

H is in which house?
a) Dayanand
b) Shankaracharya
d) Cant say
e) None of these

c) Maharishi

39.

Who among the following students are in group IV?


a) A and B
b) A and C
c) G and H
d) Data inadequate
e) None of these

40.

The student from which house is sitting opposite the Vashistha house
participant?
a) Dayanand
b) Shankracharya
c) Maharishi
d) Cant say
e) None of these

41.

Which of the following statements is false?


a) The Balmiki representative is sitting opposite the Maharishi house
representative
b) The Vyas house representative is sitting opposite the Dhruv house
representative
c) H and G are sitting opposite each other
d) The two girl participants are sitting opposite each other
e) None of these

Directions (Q. 42-46) Read the following information carefully and answer the
questioins that follow.
Twelve friends are sitting in two squares. One square is inside another. P,
Q, R, S, T and V are in the outer square facing inward. A, B, C, D, E and F are
sitting in the inner square facing outward. They all are s itting in such a way that in
each square four persons are sitting in the middle of the sides and two persons are
sitting on diagonally opposite corners. Each friend in the inner square is facing
another friend of the outer square. There are exactly two persons sitting between P
and T. E si ts second to the left of A. R is on the immediate left of the one who is f
aci ng A. Neither E nor A faces either T or P. S is facing F and Q is not opposite R
in the outer square. T is not sitting adjacent to S. B is not facing P. Between B and
D there are as many persons as between R and V.
42.

Who is sitting opposite B?


83

REASONING MADE EASY www.BankExamsToday.com


a) V
d) Data inadequate
43.

44.

b) Q
e) None of these

c) T

V is facing which of the following persons?


a) E
b) D
d) Data inadequate
e) None of these

c) A

Who is sitting on the immediate left of R?


a) A
b) S
d) Data inadequate
e) None of these

c) P

45.

Four of the five are alike in a certain manner. Select the one which is not
similar to the other four.
a) R, E
b) A, S
c) D, T
d) F, Q
e) C, E

46.

Who is sitting second to the left of D?


a) A
b) E
d) Data inadequate
e) None of these

c) F

Directions (Q. 47-51) Study the following information carefully and answer the
given questions:
E, F, G, H, I, J, K and L are eight feiends sitting around a circle facing the
centre, but not necessarily in the same order.
I is second to the left of F, who is next to the right of L. J is not the
neighbour of F or K, and is on the immediate left of H.
47.

Which of the following pairs represents the neighbours of G?


a) H, F
b) E, H
c) J, K
d) I, E
e) None of these

48.

Who among the following is on the immediate left of J?


a) K
b) H
c) I
d) L
e) None of these

49.

Who among the following is on the immediate right of E?


a) F
b) L
c) G
d) H
e) None of these

50.

Which of the following statements is true with respect to H?


a) Immediate left of J
b) Opposite L
c) Second to the right of F
d) All are true
84

REASONING MADE EASY www.BankExamsToday.com


e) None of these
51.

How many persons sit between G and I?


a) None
b) One
d) Three
e) None of these

c) Two

Directions (Q. 52-56) Study the given information carefully and answer the
given questiions.
Eight people A, B, C, D, E, F, G and H are sitting around a circular table
facing the centre, not necessarily in the same order. Three people are sitting
between A and D. B is sitting second to the right of A. C is to the immediate right of
F. D is not an immediate neighbour of either F or E. H is not an immediate
neighbour of B.
52.

What is Es position with respect to G?


a) Third to the left
b) Second to the right
c) Third to the right
d) Second to the left
e) None of these

53.

Four of the following are based on ab ove arrangement and so form a group.
Which one does not belong to the group?
a) GE
b) DC
c) AF
d) AB
e) CE

54.

Who is sitting third to the right of the one who is sitting to the immediate
right of H?
a) A
b) B
c) E
d) C
e) G

55.
Which of the following is true regarding the given arrangement?
a) E is second to the left of C
b) B is an immediate neighbour of G
c) H is an immediate neighbour of A
d) D is not an immediate neighbour of H
e) None of these
56.

How many people are sitting between H and A when counted from the right
side of H?
a) Three
b) None
c) More than three
d) One
e) Two

85

REASONING MADE EASY www.BankExamsToday.com


Directions (Q. 57-61) Study the following Information to answert these
questions.
(i)
(ii)
(iii)
(iv)

P, Q, R, S, T, U and V are sitting along the circle facing the centre.


T is 2nd to t he left of P.
R is 3rd to the right of U who is the immediate left of V.
Q is between P and S.

57.

Which of the following is true statement?


a) P is between Q and T
b) R is between P and V
c) P is 4th to the right of U
d) U is between V and Q
e) None of these

58.

Which of the following pairs has its second member sitting to the
immediate left of first member?
a) VT
b) PQ
c) SU
d) UV
e) None of these

59.

Which of the following is the false statement?


a) R is to the immediate left of P
b) V is to the immediate right of T
c) R is 4th to the right of S
d) T is between V and R
e) None of these

60.

What is the position of S?


a) To the immediate left of U
b) Between V and Q
c) 3rd to the right of T
d) To the immediate right of P
e) None of these

61.

Which of the following pairs has members sitting adjacent to each other?
a) PS
b) QU
c) UT
d) TR
e) None of these

Directions (Q. 62-66) Study the following information and answer the questions
which follow.
(i)
(ii)
(iii)
(iv)

K, L, M, N, O, P and Q are sitting around a circular table facing the


centre.
L sits between N and O.
K is third to the left of O.
Q is second to the left of M, who is to the immediate left of P.
86

REASONING MADE EASY www.BankExamsToday.com

62.

Which is the correct position of L with respect to Q?


a) Second to the right
b) First to the left
c) First to the right
d) Third to the right
e) None of these

63.

Which of the following pairs has the first person sitting to the immediate
left of the second person?
a) LO
b) MK
c) QN
d) LN
e) None of these

64.

Who sits second to the left of L?


a) P
b) M
d) Q
e) None of these

c) K

Who sits between M and Q?


a) O
b) N
d) L
e) None of these

c) K

65.

66.

Which of the following has the middle person sitting between the other
two?
a) NQL
b) PMK
c) MOP
d) POK
e) None of these

Directions (Q. 67-72) Study the follow ing information carefully and answer
the questions given below.
A, B, C, D, E, F, G and H are sitting around a circle facing at the centre. B
is second to the right of H and third to the left of A. D is not an immediate
neighbuor of either B or H and is second to the right of F. C is fourth to the right of
G.
67.

If E and F interchange their places, who will be second to the right of B?


a) F
b) C
c) D
d) Data inadequate
e) None of these

68.

In which of the following pairs is the second person sitting to the immediate
left of the first person?
a) BC
b) HE
c) FA
d) GD
e) None of these

69.

Who is 4th to the right of H?


a) B
d) Data inade quate

b) A
e) None of these

c) F
87

REASONING MADE EASY www.BankExamsToday.com

70.

71.

72.

Who is to the immediate right of B?


a) C
b) E
d) Data inadequate
e) None of these

c) H

Who is third to the right of E?


a) C
d) Data inadequate

c) D

b) F
e) None of these

In a row of 45 boys facing South, T is 8th to the right of H, who is 10th from
the right end. H is 14th to the left of R. What is Rs position from the left
end?
a) 21st
b) 23rd
c) 24th
d) Data inadequate
e) None of these

Directions (Q. 73-78) Study the following information carefully and answer the
questions given below:
P, M, D, A, F, H, R and B are sitting around a circle facing at the centre. R
is 4th to the right of A, who is 3rd to the right of P. M is second t o the l eft of H, who
is 2nd to the left of P. D is 3rd to the right of B.
73.

Who is 3rd to the left of H?


a) M
b) A
d) F
e) Data inadequate

c) B

74.

Which of the following pairs represents the immediate neighbours of B?


a) FM
b) AP
c) MP
d) AF
e) None of these

75.

If R and B interchange their positions, who will be 2nd to the left of M?


a) B
b) H
c) P
d) Data inadequate
e) None of these

76.

Who is the immediate right of P?


a) F
b) R
d) Data inadequate
e) None of these

c) B

Who is the immediate right of M?


a) H
b) A
d) Data inadequate
e) None of these

c) D

77.

78.

In a row of 40 children, Q is 14th from the left end and there are 16 children
between Q and M. What is Ms position from the right end of the row?
a) 11th
b) 10th
c) 30th
d) Data inadequate
e) None of these
88

REASONING MADE EASY www.BankExamsToday.com

Directions (Q. 79-83) Study the following information carefully and answer the
questions given below.
B, D, M, K, P, Q, W and H are sitting around a circle facing at the centre.
M is to the immediate right of B who is 4th to the right of K. P is 2nd to the left of B
and is 4th to the right of W. Q is 2nd to the right of D who is 2nd to the right of H.
79.

Who is 3rd to the right of B?


a) W
b) M
d) H
e) None of these

c) K

80.

Which of the following represents the immediate neighbours of D?


a) PQ
b) KH
c) PH
d) KQ
e) PK

81.

Who is 3rd to the right of W?


a) P
b) D
d) R
e) Data inadequate

c) K

Who is 2nd to the left of P?


a) D
b) H
d) Data inadequate
e) None of these

c) K

Who is to the immediate left of B?


a) Q
b) P
d) Data inadequate
e) None of these

c) W

82.

83.

Directions (Q. 84-88) Study the following information carefully and answer the
questions given below.
A, M, D, P, R, T, B and H are sitting around a circle facing at the centre. M
is 3rd to the left of Awho is 2nd to the left of T. D is 2nd to the right of H who is 2nd to
the right of T. R is 2nd to the right of B who is not an immediate neighbour of T.
84.

Which of the following combinations represents the first and the second to
the left of B respectively?
a) MD
b) DH
c) AM
d) AR
e) DM

85.

Who is 3rd to the right of T?


a) D
b) B
d) M
e) None of these

86.

Who is to the immediate left of H?


a) P
b) M

c) H

c) T
89

REASONING MADE EASY www.BankExamsToday.com


d) R
87.

88.

e) Data inadequate

Who is 2nd to the left of B?


a) D
b) H
d) Data inadequate
e) None of these

c) M

In which of the following combinations 3rd person is 2nd to the left of 2nd
person?
a) BAR
b) DBM
c) TPH
d) PMH
e) None of these

Directions (Q. 89-93) Study the following information carefully and answer the
questions given below
10 people are sitting in two parallel rows containing five people each, in
such a way that there is an eaual distance between adjacent people. In row-1, Q, R,
S, T and U are seated and all of them are facing north. In Row-2, L, M, N, O and P
are seated and all of them are facing south. Therefore in the given sitting
arrangement each member seated in a row faces another member of the other row.
S is sitting 2nd to the left of Q. Q is not sitting at any of the ends of the line.
There are two persons between P and L. The person who faces T is to the
immediately left of M. M is sitting at the extreme right end. Q is an immediate
neighbour of U. The person who faces U is an immediate neighbour of both P and
N.
89.

Who among the following is sitting 2nd to the right of O?


a) P
b) M
c) L
d) Cannot be determined
e) None of these

90.

Who among the following sits exactly between Q and S?


a) T
b) U
c) R
d) There is no person between Q and S
e) Cannot be determined

91.

Who among the following is sitting at the extreme right end of the Row-1?
a) Q
b) U
c) S
d) R
e) Cannot be detrmined

92.

Which of the following statements is true regarding R?


a) R is at the extreme left end of the row
b) R is an immediate neighbour of U
c) R is sitting second to the right of T
d) R is sitting exactly between T and U
90

REASONING MADE EASY www.BankExamsToday.com


e) There two persons between R and S
93.

Who among the following is not seated at any extreme end of Row-1 and
Row-2?
a) P
b) S
c) M
d) R
e) O

Directions (Q. 94-98) Study the following information carefully and answer the
questions given below
8 persons E, F, G, H, I, J, K and L - are sitting around a circle at
equidistance but not necessarily in the same order. Some of them are facing towards
the centre while some others are facing outside the centre. L is sitting 3rd to the left
of K. Both K and L are facing towards the outside. H is not an immediate neighbour
of K or L. J faces the just opposite direction of H. (It implies that if H is facing
towards the centre, J would face outside the centre). J is sitting 2nd to the left of H.
Both the immediate neighbours of G face just opposite direction of G. E is an
immediate neighbour of K. Both the immediate neighbour of H faces just opposite
direction of H. E faces towards the centre and he is an immediate n eighbour of both
K and I. I face towards the centre.
94.

Who amongst the following are not facing towards the centre?
a) E, F and I
b) F, G and J
c) H, K and L
d) G, I and J
e) None of these

95.

What is the position of G with respect to E?


a) 2nd to the left
b) 3rd to the right
c) 4th to the left
d) 2nd to the right
e) 5th to the right

96.

Who among the following is sitting exactly between F and K?


a) J
b) I
c) E
d) G
e) None of these

97.

How many persons are sitting between H and K if we move clockwise


starting from H?
a) Three
b) Four
c) Five
d) Two
e) One

98.

Which of the following statements is not true regarding the given sitting
arrangement?
a) L is sitting exactly between G and I
b) H is sitting 3rd to the right of K
c) F and I are sitting just opposite to each other
91

REASONING MADE EASY www.BankExamsToday.com


d) E is sitting 3rd to the left of G
e) All are true
Directions (Q. 99-103) Study the following information carefully and answer
the questions.
A, B, C, D, E, F, G and H are sitting around a circle, facing the centre. A
sits fourth to the right of H while second to the left of F. C is not the neighbour of F
and B. D sits third to the right of C. H never sits next to G.
99.

Who amongst the following sits between B and D?


a) G
b) F
c) H
d) A
e) C

100.

Which of the following pairs sit between H and G?


a) BH
b) EF
c) CE
d) DB
e) None of these

101.

Four of the following are alike I n a certain way based on their positions in
the sitting arrangement and so form a group. Which is the one that does not
belong to that group?
a) AE
b) HF
c) BD
d) GE
e) CH

102.

Who is immediate right of A?


a) C
b) D
d) Data inadequate
e) None of these

c) G

Who sits second to the right of B?


a) A
b) C
d) E
e) None of these

c) D

103.

Directions (Q. 104-108) Study the following carefully and answer the questions.
A, B, C, D, E, F, G and H are sitting around a circle, facing the centre. E
and G always sit next to each other. D sits third to the right of C. F sits to the left of
H. C never sits next to A while D never sits next to G. H is not the neighbour of D
and C.
104.

105.

Who sits to immediate right of F?


a) D
b) C
d) A
e) None of these

c) B

Four of the following are alike in a certain way based on their positions in
the sitting arrangement and so form a group. Which is the one that does not
belong to that group?
92

REASONING MADE EASY www.BankExamsToday.com


a) CH
d) DG

b) BA
e) AC

c) FE

106.

Which of the following pairs sits between B and F?


a) HB
b) FD
c) BG
d) GC
e) AH

107.

Who sits second to the left of B?


a) F
b) G
d) E
e) None of these

c) A

Who sits between A and D?


a) B
b) F
d) E
e) None of these

c) C

108.

Directions (Q. 109-113) Study the following information carefully to answer


these questions.
Ashwini, Priya, Sudha, Rani, Meeta, Geeta and Mukta are sitting around a
circle facing the centre. Ashwini is third to the left of Mukta and to the immediate
right o f Rani. Priya is second to the left of Geeta who is not an immediate
neighbour of Meeta.
109.

110.

Who is to the immediate right of Priya?


a) Meeta
b) Sudha
d) Cannot be determined
e) None of the above
Who is second to the left of Rani?
a) Ashwini
b) Meeta
d) Sudha
e) None of these

c) Mukta

c) Priya

111.

Which of the following pairs of persons has the first person sitting to the
immediate left of second person?
a) Rani-Meeta
b) Ashwini-Geeta
c) Sudha-Priya
d) Geeta-Sudha
e) None of these

112.

Which of the following groups has the first person sitting between the other
two?
a) Meeta-Ashwini-Geeta
b) Sudha-Rani-Geeta
c) Mukta-Priya-Rani
d) Mukta-Priya-Sudha
e) None of these

113.

Which of the following is the correct position of Rani with respect to


Mukta?
93

REASONING MADE EASY www.BankExamsToday.com


i)
Third to the right
ii)
Third to the left
iii)
Fourth to the left
iv)
Fourth to the right
a) i) only
d) Both ii) and iv)

b) ii) only
e) Both i) and iii)

c) Both i) and ii)

Directions (Q. 114-118) Study the following information carefully to answer


these questions.
A, B, C, D, E, F, G and H are sitting around a circle facing the centre. F is
third to the right of C and second to the left of H. D is not an immediate neighbour
of C or H. E is to the immediate right of A, who is second to the right of G.
114.

115.

Who is second to the left of C?


a) A
b) B
d) D
e) None of these

c) E

Who is to the immediate right of C?


a) A
b) B
d) B or D
e) None of these

c) D

116.

Which of the following pairs of persons has first person sitting to the right
of the second person?
a) CB
b) AE
c) FG
d) HA
e) DB

117.

Who sits between G and D?


a) H
b) D
d) E
e) None of these

118.

c) F

Which of the following is the correct position of B with respect to H?


I.
Second to the right
II.
Fourth to the ri ght
III.
Fourth to the left
IV.
Second to the left
a) Only I
b) Only II
c) Only III
d) Both II and III
e) None of these

Answers:
1.
2.
3.
4.
5.

Option D
Option E
Option A
Option B
Option B
94

REASONING MADE EASY www.BankExamsToday.com


6.
7.
8.
9.
10.
11.
12.
13.
14.
15.
16.
17.
18.
19.
20.
21.
22.
23.
24.
25.
26.
27.
28.
29.
30.
31.
32.
33.
34.
35.
36.
37.
38.
39.
40.
41.
42.
43.
44.
45.
46.
47.
48.
49.
50.
51.

Option A
Option D
Option D
Option B
Option C
Option A
Option B
Option A
Option D
Option A
Option B
Option E
Option A
Option B
Option C
Option E
Option E
Option C
Option B
Option B
Option A
Option B
Option D
Option C
Option B
Option A
Option A
Option E
Option D
Option B
Option E
Option B
Option D
Option C
Option B
Option D
Option C
Option A
Option C
Option E
Option A
Option B
Option A
Option C
Option B
Option D
95

REASONING MADE EASY www.BankExamsToday.com


52.
53.
54.
55.
56.
57.
58.
59.
60.
61.
62.
63.
64.
65.
66.
67.
68.
69.
70.
71.
72.
73.
74.
75.
76.
77.
78.
79.
80.
81.
82.
83.
84.
85.
86.
87.
88.
89.

Option D
Option B
Option C
Option B
Option C
Option C
Option E
Option B
Option A
Option D
Option E
Option D
Option A
Option C
Option B
Option E
Option D
Option C
Option A
Option B
Option E
Option B
Option D
Option E
Option A
Option C
Option A
Option D
Option E
Option B
Option C
Option A
Option E
Option D
Option A
Option C
Option B
Option C
L is sitting 2nd to the right of O.

90.

Option A
T sits exactly between Q and S.

91.

Option D
96

REASONING MADE EASY www.BankExamsToday.com

R is sitting at the extreme right end of the Row-1.


92.

Option B
R is at the extreme right end.
R is to the immediate right of U.
R is sitting third to the right of T.
R is at one of the ends.
There are three persons T, Q and U between R and S

93.

Option E
O is second from the left in Row-2.

94.

Option C
H, K and L are facing outside.

95.

Option D
E is facing t owards the centre. G is 2nd to the right or 5th to the left of E.
97

REASONING MADE EASY www.BankExamsToday.com

96.

Option A
J is sitting exactly between F and K.

97.

Option B
There are four persons G, L, I and E between H and K if we move
clockwise from H.

98.

Option E
All the statements are true.

99.
100.
101.
102.
103.
104.
105.
106.
107.
108.
109.
110.
111.
112.
113.
114.
115.
116.
117.
118.

Option B
Option C
Option A
Option B
Option D
Option D
Option E
Option E
Option C
Option B
Option C
Option E
Option D
Option B
Option E
Option A
Option B
Option E
Option C
Option D

98

REASONING MADE EASY www.BankExamsToday.com

Chapter - 5
Syllogisms
Directions (Q. 1-5) In each of the questions below are given three statements
followed by two conclusions numbered I and II. You have to take the given
statements to be true even if they seem to be at variance withcommonly known
facts. Read all the conclusions and then decide which of the given conclusions
logically follows from the given statements disregarding commonly known
facts.
Give answer:
a) If only conclusion I follows
b) If only conclusion II follows
c) If either conclusion I or II follows
d) If neither conclusion I nor II follows
e) If both conclusions I and II follow
1.

2.

3.

4.

Statements:

All benches are cots.


No cot is lamp.
Some lamps are candles.

Conclusions:

I. Some cots are benches.


II. Some candles are cots.

Statements:

Some cats are dogs.


All dogs are goats.
All goats are walls.

Conclusions:

I. Some walls are dogs.


II. Some walls are cats.

Statements:

Some buildings are sofas.


Some sofas are benches.
Some benches are tables.

Conclusions:

I. Some tables are sofas.


II. No table is sofa.

Statements:

All rats are bats.


Some bats are desks.
All desks are chairs.

Conclusions:

I. Some desks are rats.


99

REASONING MADE EASY www.BankExamsToday.com


II. Some chairs are rats.
5.

Statements:

Some roads are ponds.


All ponds are stores.
Some stores are bags.

Conclusions:

I. Some bags are ponds.


II. Some stores are roads.

Directions (Q. 6-11) In each question below are four statements followed by
four conc lusions numbered I, II, III and IV. You have to take the four given
statements to be true even if they seem to be at variance from commonly
known facts and than decide which of the given conclusions logically follows
from the four given statements disregarding commonly known facts. Then
decide which of the answers a, b, c, d and e is correct answer.
6.

Statements:

All belts are rollers.


Some rollers are wheels.
All wheels are mats.
Some mats are cars.

Conclusions:

I. Some mats are rollers.


II. Some mats are belts.
III. Some cars are rollers.
IV. Some rollers are belts.

a) Only I and II follow


b) Only I, III and IV follow
c) Only I and IV follow
d) Only II, III and IV follow
e) None of the above
7.

Statements:

Some tyres are rains.


Some rains are flowers.
All flowers are jungles.
All jungles are tubes.

Conclusions:

I. Some jungles are tyres.


II. Some tubes are rains.
III. Some jungles are rains.
IV. Some tubes are flowers.

a) Only I, II and III follow


b) Only II, III and IV follow
c) Only I, III and IV follow
d) All follow
e) None of the above
100

REASONING MADE EASY www.BankExamsToday.com


8.

Statements:

All desks are chairs.


All chairs are tables.
All tables are boxes.
All boxes are trunks.

Conclusions:

I. Some trunks are tables.


II. All chairs are boxes.
III. Some boxes are desks.
IV. All desks are trunks.

a) Only I, II and III follow


b) Only I, II and IV follow
c) Only II, III and IV follow
d) All follow
e) None of the above
9.

Statements:

Some birds are goats.


Some goats are horses.
Some horses are lions.
Some lions are tigers.

Conclusions:

I. Some tigers are goats.


II. No tiger is goat.
III. Some lions are birds.
IV. No lion is bird.

a) Only either I or II follows


b) Only either III or IV follows
c) Only either I or II and either III or IV follow
d) Only I and III follow
e) None of the above
10.

Statements:

All papers are bottles.


All bottles are cups.
Some cups are jugs.
Some jugs are plates.

Conclusions:

I. Some plates are cups.


II. Some plates are bottles.
III. Some cups are papers.
IV. Some bottles are papers.

a) Only III and IV follow


b) Only I and II follow
c) Only I and II follow
d) Only II and IV follow
e) None of the above
11.

Statements:

All bulbs are wires.


101

REASONING MADE EASY www.BankExamsToday.com


No wire is cable.
Some cables are brushes.
All brushes are paints.
Conclusions:

I. Some paints are cables.


II. Some wires are bulbs.
III. Some brushes are wires.
IV. Some cables are bulbs.

a) None follows
b) Only I and II follow
c) Only II follows
d) Only III follows
e) Only IV follows
Directions (Q. 12-16) In each of the questions below are given four statements
followed by three conclusions numbered I, II and III. You have to take the
given statements to be true even if they seem to be at variance from commonly
known facts. Read all the conclusions and then decide which of the given
conclusions logically follows from the given statements disregarding commonly
known facts.
12.

Statements:

All booklets are packets.


All packets are bottles.
Some bottles are cans.
Some cans are pitchers.

Conclusions:

I. Some pitchers are bottles.


II. Some cans are packets.
III. Some bottles are booklets.

a) None follows
b) Only I follow
c) Only II follows
d) Only III follows
e) Only II and III follow
13.

Statements:

Some ropes are walls.


Some walls are sticks.
All sticks are chairs.
All chairs are tables.

Conclusions:

I. Some tables are walls.


II. Some chairs are ropes.
III. Some sticks are ropes.

a) None follows
b) Only I follows
c) Only II follows
102

REASONING MADE EASY www.BankExamsToday.com


d) Only III follows
e) Only II and III follow
14.

Statements:

Some rivers are jungles.


Some jungles are horses.
Some horses are tents.
Some tents are buildings.

Conclusions:

I. Some buildings are horses.


II. Some tents are jungles.
III. Some horses are rivers.

a) None follows
b) Only I follows
c) Only II follows
d) Only III follows
e) Only I and II follow
15.

Statements:

Some pens are knives.


All knives are pins.
Some pins are needles.
All needles are chains.

Conclusions:

I. Some chains are pins.


II. Some needles are knives.
III. Some pins are pens.

a) Only I follows
b) Only II follows
c) Only III follows
d) Only II and III follow
e) None of these
16.

Statements:

All fields are ponds.


No pond is tree.
Some trees are huts.
All huts are goats.

Conclusions:

I. Some goats are fields.


II. No goat is field.
III. Some goats are trees.

a) Only I follows
b) Only II follows
c) Only III follows
d) Only eithr I or II follows
e) Only either I or II and III follow

103

REASONING MADE EASY www.BankExamsToday.com


Directions (Q. 17-28) In each of the questions below are given three statements
followed by three conclusions numbered I, II and III. You have to take the
given statements to be true even if they seem to be at vari ance from commonly
known facts. Read all the conclusions and then decide which of the given
conclusions logically follow from the given statements disregarding commonly
known facts.
17.

Statements:

All books are tents.


Some tents are lakes.
All lakes are ponds.

Conclusions:

I. Some ponds are books.


II. Some ponds are tents.
III. Some lakes are books.

a) None follows
b) Only I follows
c) Only II follows
d) Only III follows
e) Only II and III follow
18.

Statements:

Conclusions:

All pictures are walls.


Some walls are rooms.
Some rooms are windows.
I. Some windows are walls.
II. Some windows are pictures.
III. Some rooms are walls.

a) None follows
b) Only I follows
c) Only II follows
d) Only III follows
e) Only II and III follow
19.

Statements:

All baskets are marbles.


Some marbles are sticks.
No stick is garden.

Conclusions:

I. Some gardens are baskets.


II. Some sticks are baskets.
III. No garden is baskets.

a) Only I follows
b) Only III follows
c) Only either I or III follows
d) Only II follows
e) None of the above
20.

Statements:

Some bulbs are tubes.


104

REASONING MADE EASY www.BankExamsToday.com


Some tubes are wires.
Some wires are lamps.
Conclusions:

I. Some lamps are tubes.


II. Some wires are bulbs.
III. Some lamps are bulbs.

a) None follows
b) Only III follows
c) Only II follows
d) Only I follows
e) Only I and II follow
21.

Statements:

All buildings are rivers.


All rivers are jungles.
All jungles are mountains.

Conclusions:

I. Some mountains are rivers.


II. Some jungles are buildings.
III. Some mountains are buildings.

a) Only I and II follow


b) Only I and III follow
c) Only II and III follow
d) All I, II and III follow
e) None of the above
22.

Statements:

All chairs are keys.


All keys are baloons.
Some balloons are mirrors.
Some mirrors are desks.

Conclusions:

I. Some desks are keys.


II. Some balloons are chairs.
III. Some mirrors are balloons.

a) Only I follows
b) Only II follows
c) Only III follows
d) Only II and III follow
e) All I, II and III follow
23.

Statements:

Some drums are posters.


All posters are windows.
Some windows are tablets.
All tablets are books.

Conclusions:

I. Some windows are drums.


II. Some books are posters.
105

REASONING MADE EASY www.BankExamsToday.com


III. Some tablets are drums.
a) None follows
b) Only I follow
c) Only II follows
d) Only III follows
e) Only I and II follow
24.

Statements:

Some boxes are toys.


Some toys are nails.
Some nails are stores.
Some stores are shops.

Conclusions:

I. Some nails are boxes.


II. Some toys are shops.
III. No nail is box.

a) Only I follows
b) Only III follows
c) Only either I or III follows
d) Only II follows
e) None of the above
25.

Statements:

All doors are windows.


No window is house.
Some houses are buildings.
All buildings are skies.

Conclusions:

I. Some skies are doors.


II. Some skies are houses.
III. Some buildings are doors.

a) Only I follows
b) Only II follows
c) Only III follows
d) Only II and III follow
e) None of these
26.

Statements:

All rivers are walls.


All walls are stones.
All stones are clothes.
All clothes are trees.

Conclusions:

I. Some trees are stones.


II. Some clothes are rivers.
III. All walls are clothes.

a) Only I and II follow


b) Only I and III follow
c) Only II and III follow
106

REASONING MADE EASY www.BankExamsToday.com


d) All I, II and III follow
e) None of the above
27.

Statements:

Some letters are glasses.


Some glasses are plates.
All plates are buses.
All buses are cars.

Conclusions:

I. Some cars are letters.


II. Some cars are glasses.
III. Some buses are glasses.

a) Only I and II follow


b) Only I and III follow
c) Only II follows
d) Only III follows
e) Only II and III follow
28.

Statements:

All books are pens.


Some pens are ropes.
All ropes are discs.
Some discs are bricks.

Conclusions:

I. Some bricks are ropes.


II. Some discs are books.
III. Some bricks are pens.

a) None follows
b) Only I follows
c) Only II follows
d) Only III follows
e) Only II and III follow

Directions (Q. 29-34) In each of the questions below are given four statements
followed by four conclusions number I, II, III and IV. You have to take the
given statements to be true even if they seem to be of variance from commonly
known facts. Read all the conclusions and then decide which of the given
conclusions logically follows from the given statements disregarding commonly
known facts.
29.

Statements:

Some robots are machines.


Some computers are both robots and machines.
Some animals are machines.
Some toys are animals.

Conclusions:

I. Some toys are robots.


II. Some toys are machines.
107

REASONING MADE EASY www.BankExamsToday.com


III. Some animals are computers.
IV. Some robots are not toys.
a) None follows
b) Only II and III follow
c) Only I and III follow
d) Only III follows
e) Only either I or IV follows
30.

Statements:

All suns are stars.


All moons are stars.
Some planets are suns.
Some stars are gases.

Conclusions:

I. Some starts are planets.


II. Some suns are gases.
III. No moon is a planet.
IV. Some gases are moons.

a) None follows
b) Only I follows
c) Only I and II follows
d) Only III and IV follow
e) Only I and III follow
31.

Statements:

All books are diaries.


Some diaries are pens.
Some pens are drawers.
All drawers are chairs.

Conclusions:

I. Some drawers are diaries.


II. Some chairs are pens.
III. Some pens are books.
IV. Some diaries are books.

a) None follows
b) Only II follows
c) Only II and III follow
d) Only II and IV follow
e) All follow
32.

Statements:

Some buildings are rivers.


Some mountains are both buildings and rivers.
Some roads are buildings.
All roads are trucks.

Conclusions:

I. Some mountains are roads.


II.Some buildings are trucks.
III.Some rivers are roads.
108

REASONING MADE EASY www.BankExamsToday.com


IV. Some trucks are rivers.
a) None follows
b) Only I follows
c) Only II follows
d) Only III follows
e) All follow
33.

Statements:

All tables are round.


Some hills are round.
Some rivers are hills.
All rivers are conical.

Conclusions:

I. Some rivers are round.


II. Some hills are conical.
III. Some rivers are both hills and round.
IV. Some tables are conical.

a) None follows
b) Only II follows
c) Only I and III follow
d) Only II and III follow
e) All follow
34.

Statements:

All sharks are fishes.


Some fishes are birds.
All birds are trees.
All trees are insects.

Conclusions:

I. Some insects are sharks.


II. Some sharks are trees.
III. All insects are birds.
IV. Some birds are sharks.

a) None follows
b) Only II follows
c) Only I and IV follow
d) Only II and III follow
e) All follow

Answers:
1.
Option A

109

REASONING MADE EASY www.BankExamsToday.com

So, only I follows.


2.

Option E

So, both I and II follow.


3.

Option C

110

REASONING MADE EASY www.BankExamsToday.com

So, either I or II follows


4.

Option D

So, neither I nor II follows.


5.

Option B

111

REASONING MADE EASY www.BankExamsToday.com

So, only II follows.


6.

Option C

Only I and IV follow


7.

Option B

112

REASONING MADE EASY www.BankExamsToday.com

So, only II, III and IV follow


8.

Option D

All follows
9.

Option C

113

REASONING MADE EASY www.BankExamsToday.com

So, either III or IV follow


10.

Option A

So, only III and IV follow


11.

Option B
114

REASONING MADE EASY www.BankExamsToday.com

Only I and II follow.


12.

Option D

Only III follows.


13.

Option B

115

REASONING MADE EASY www.BankExamsToday.com

Only I follows.
14.

Option A

None follows
15.

Option E

116

REASONING MADE EASY www.BankExamsToday.com

Only I and III follow.


16.

Option E

Either I or II and III follow


17.

Option C

117

REASONING MADE EASY www.BankExamsToday.com

So, only II follows


18.

Option D

Only III follows.


19.

Option C
118

REASONING MADE EASY www.BankExamsToday.com

So, either I or III follows


20.

Option A

So, None follows


21.

Option D

119

REASONING MADE EASY www.BankExamsToday.com

So, all I, II and III follow


22.

Option D

Only II and III follow.


120

REASONING MADE EASY www.BankExamsToday.com


23.

Option B

Only I follows.
24.

Option C

Either I or III follows


25.

Option B

121

REASONING MADE EASY www.BankExamsToday.com

Only II follows
26.

Option D

So, All I, II and III follow


27.

Option E
122

REASONING MADE EASY www.BankExamsToday.com

Only II and III follow


28.

Option A

None follows
29.
30.
31.
32.
33.
34.

Option E
Option B
Option D
Option C
Option B
Option A

123

REASONING MADE EASY www.BankExamsToday.com

Chapter - 6
Blood Relations
1.

R is sister of M who is brother of H. D is mother of K who is brother of M.


How is R related to D?
a) Sister
b) Daughter
c) Mother
d) Data inadequate
e) None of these

2.

K is brother of T. M is mother of K. W is brother of M. How is W related


to T?
a) Maternal uncle
b) Paternal uncle
c) Grandfather
d) Data inadequate
e) None of these

3.

Pointing to a woman, Nirmal said, She is the daughter of my wifes


grandfathers only child. How is the women releated to Nirmal?
a) Wife
b) Sister-in-law
c) Sister
d) Data inadequate
e) None of these

4.

D is brother of K. M is sister of K. T is father of R who is brother of M. F


is mother of K. At least how many sons does T and F have?
a) Two
b) Three
c) Four
d) Data inadequate
e) None of these

Directions (Q. 5-6) Study the following information carefully and answer the
questions given below
S has two daughters R and A. G is married to R. G is father of N. L is son
of A.
5.

6.

How L is related to S?
a) Nephew
b) Brother
d) Great Grandson
e) None of these

c) Grandson

Which of the following statement is/are true on the basis of information


given above?
a) G is son-in-law of S
b) R is wife of G
c) A is mother of L
d) L is cousin of N
e) All are true
124

REASONING MADE EASY www.BankExamsToday.com


7.

M is sister of K. D is brother of K. F is mother of M. How is K related to


F?
a) Son
b) Daughter
c) Son or Daughter
d) Data inadequate
e) None of the above

Directions (Q. 8-10) Read the following information carefully and answer the
questions which follow:
A B means A is wife of B
A + B means A is brother of B
A B means A is daughter of B
A B means A is son of B

8. How is Q related to P if P R T Q?
a) Granddaughter
b) Mother-in-law/Father-in-law
c) Grandmother
d) Grandmother/Grandfather
e) None of the above
9. How is R related to Q, if P Q + R T?
a) Brother/Sister
b) Niece
d) Nephew/Niece
e) None of these

c) Sister

10. How is T related to P if P Q T + R?


a) Mother
b) Father-in-law
c) Mother-in-law
d) Mother-in-law/Father-in-law
e) None of the above
Directions (Q. 11-15) Read the information carefully and answer the following
questions.
If A + B means A is the father of B.
If A B means A is the sister of B.
If A $ B means A is the wife of B.
If A % B means A is the mother of B.
If A B means A is t he son of B.
11.

What should come in place of the question mark, to establish that J is the
brother of T in the expression?
JP%H?T%L
a) x
b)
c) $
d) Either or x
e) Either + or

12.

Which among the given expressions indicate that M is the daughter of D?


125

REASONING MADE EASY www.BankExamsToday.com


a) L % R $ D + T M
b) L + R $ D + M T
c) L % R % D + T M
d) D + L $ R + M T
e) L $ D R % M T
13. Which among the f ollowing options is true, if thue expression I + T % J L
K is definitely true?
a) L is the daughter of T
b) K is the son-in-law of I
c) I is the grandmother of L
d) T is the father of J
e) J is the brother of L
14. Which among the following expressions is true, if Y is the son of X is definitely
false?
a) W % L T Y X
b) W + L T Y X
c) X + L T Y W
d) W $ X + L + Y + T
e) W % X + T Y L
15. What should come in place of the question mark, to establish that T is the sisterin-law of Q I n the expression?
R%TP?Q+V
a)
b) %
c)
d) $
e) None of these
Directions (Q. 16-20) Study the following information carefully and answer the
given questions:
There are eight family members- I, J, K, L, M, N, P and Q. J has only two
children K and L. I is mother of K, who is sister of L. N is son of L. Q is brother of
N. M is mother of Q. P is granddaughter of I.
16.

17.

18.

How is Q related to L?
a) Mother
d) Sister

b) Son
e) None of these

c) Daughter

Who among the following is father of P?


a) J
b) L
d) Cannot be determined
e) None of these

c) N

How is M related to I?
a) Son
d) Cannot be determined

c) Daughter

b) Son-in-law
e) None of these

126

REASONING MADE EASY www.BankExamsToday.com


19.

20.

How many children does M have?


a) One
b) Two
d) Either b or c
e) None of these

c) Three

How is K related to M?
a) Sister
d) Mother-in-law

c) Sister-in-law

b) Daughter
e) None of these

Directions (Q. 21-23) Read the following information carefully and answer the
questions which follow.
If A B means A is father of B.
If A + B means A is wife of B.
If A B means A is daughter of B.
If A B means A is son of B.
21.

How is L related to Q in the expression L M O P Q?


a) Granddaughter
b) Niece
c) Daughter-in-law
d) Daughter
e) cannot be determined

22.

What will come in the place of the question mark, to establish that Q is the
nephew of T in the expression Q ? R S T?
a) +
b)
c)
d)
e) Either or

23.

Which of the following relations are true based upon the relations given in
the equation:
A B C + D E?
a) C is mother of A
b) E is wife of B
c) D is brother of A
d) E is mother-in-law of C
e) None is true

Directions (Q. 24-26) Study the following information carefully and answer the
questions below.
(i)
(ii)
(iii)
(iv)
24.

P Q means P is brother of Q.
P Q means P is mother of Q.
P + Q means P is father of Q.
P Q means P is sister of Q.
Which of the following means M is niece of N ?
a) M R N
b) N J + M D
d) N J M
e) None of these

c) N J + M

127

REASONING MADE EASY www.BankExamsToday.com

25.

Which of the following means B is grandfather of F?


a) B + J F
b) B J + F
d) B T + F
e) None of these

26.

c) B T F

How is M related to K in the expression B + K T M?


a) Son
b) Daughter
c) Son or daughter
d) Data inadequate
e) None of these

Directions (Q. 27-32) Read the following information carefully and answer the
questions which follow:
(i)
(ii)
(iii)
(iv)
27.

28.

A B means A is father of B
A + B means A is daughter of B
A B means A is mother of B
A B means A is brother of B

If P R Q T how is P related to T?
a) Grandmother
b) Mother-in-law
d) Grandfather
e) None of these

c) Sister

If P Q + R T how is T related to Q?
a) Aunt
b) Sister
d) Grandson
e) None of these

c) Brother

29.

Which of the following means that R is wife of P?


a) P R Q T
b) P T + R Q
c) P R Q + T
d) P T Q + R
e) None of these

30.

If R P J Q how is J related to R?
a) Son/daughter
b) Nephew
d) Grandson
e) None of these

31.

32.

If P + Q R T, how is T related to P?
a) Aunt
b) Aunt/Uncle
d) Grandmother
e) None of these

c) Niece

c) Father

If P T Q + R how is R related to P?
a) Daughter
b) Husband
d) Son-in-law/Daughter-in-law e) None of the above

c) Son-in-law

Answers:
128

REASONING MADE EASY www.BankExamsToday.com

1.
2.
3.

Option B
Option A
Option A
Woman = daughter of Nirmals wifes grandfathers only child = daughter
of Nirmals wifes father = Nirmals wife

4.
5.

Option A
Option C
A is daughter of S. L is son of A. So, L is grandson of S.

6.

Option E
All the statements are true. R is wife of G. R is daughter of S. So, G is sonin-law of S. L is son of A and A is daughter of S. So, A is mother of L. R
and A are sisters. N is child of R and L is son of A. So, L is cousin of N.

7.
8.
9.
10.
11.
12.
13.
14.
15.
16.
17.
18.
19.
20.
21.
22.
23.
24.
25.
26.
27.
28.
29.
30.
31.
32.

Option C
Option D
Option E
Option B
Option A
Option B
Option B
Option D
Option D
Option B
Option B
Option E
Option C
Option C
Option A
Option B
Option E
Option B
Option A
Option D
Option A
Option E
Option D
Option B
Option E
Option C

129

REASONING MADE EASY www.BankExamsToday.com

Chapter 7
Analogy
Directions (Q.1-9) In each of the following questions, select the related
letter/word/number from the given alternatives.
1.

2.

3.

4.

5.

6.

7.

8.

9.

Lion : Forest : : Fish : ?


a) Cage
b) Nest
d) Water
e) None of these

c) Sky

Polio : Virus : : Anthrax : ?


a) Fungus
b) Bacteria
d) Insect
e) None of these

c) Virus

Money : Yenom : : Right : ?


a) HTIRG
b) THGIR
d) IRGHT
e) None of these

c) GIRHT

NIIOLUSL : ILLUSION : : TBERVARE : ?


a) STRANGE
b) VERTEBRA
d) ERAVEBT
e) None of these

c) VABTERE

RUST : 9687 : : TSUR : ?


a) 7896
b) 7869
d) 6789
e) None of these

c) 7689

13 : 169 : : ?
a) 12 : 140
d) 9 : 81

b) 3 : 6
e) None of these

c) 11 : 111

23 : 29 : : 41 : ?
a) 43
d) 49

b) 45
e) None of these

c) 47

6 : 10 : : 9 : ?
a) 10
d) 18

b) 12
e) None of these

c) 15

River : Tributary : : Tree : ?


a) Stem
b) Root
d) Flower
e) None of these

c) Branch

130

REASONING MADE EASY www.BankExamsToday.com


Answers:
1.

Option D
Forest is the habitat of Lion. Similarly, water body is the habitat of fish.

2.

Option B
The causative organism of polio is virus. Similarly, the causative organism
of anthrax is bacteria.

3.

Option B

4.

Option B
From the jumbled letters NIIOLUSL, we can form the meaningful word
ILLUSION. Similarly, we can form VERTEBRA from the jumbled letters
TBERVARE.

5.

Option B
R
U
9
6
Similarly,
T
S
7
8

S
8

T
7

U
6

R
9

6.

Option D
The relation is x : x square
13 13 = 169
Similarly,
9 9 = 81

7.

Option A
Two consecutive prime numbers are given:
23 29
Similarly,
41 43

8.

9.

Option C
23=6
2 5 = 10
Similarly,
33=9
3 5 = 15

Option C
Tributary is a part of river. Similarly, branch is a part of tree and is analogus
to tributary in the case of river.
131

REASONING MADE EASY www.BankExamsToday.com

Chapter - 8
Coded Inequality
Directions (Q. 1-5) In the following questions, the symbols @, , $, % and *
are used with the following meanings as illustrated below.
P Q means P is not greater than Q.
P % Q means P is not smaller than Q.
P * Q means P is neither smaller than nor equal to Q.
P @ Q means P is neither greater than nor equal to Q.
P $ Q means P is neither greater than nor smaller than Q.
Now in each of the following questions, assuming the given statements to be true,
find which of the two conclusions I and II given below them is/are definitely true.
Give answer:
a) If only conclusion I is true.
b) If only conclusion II is true.
c) If either conclusion I or II is true.
d) If neither conclusion I nor II is true.
e) If both conclusions I and II are true.
1.

Statements:
Conclusions:

K @ V, V N, N % F
I. F @ V
II. K @ N

2.

Statements:
Conclusions:

H W, W $ M, M @ B
I. B * H
II. M % H

3.

Statements:
Conclusions:

D % B, B * T, T $ M
I. T D
II. M D

4.

Statements:
Conclusions:

M * T, T @ K, K N
I. N * T
II. N * M

5.

Statements:
Conclusions:

R $ J, J % D, D * F
I. D $ R
II. D @ R

Directions (Q. 6-11) In the following questions, the symbols , @, $, % and *


are used with following meanings as illustrated below.
P % Q means P is not greater than Q
132

REASONING MADE EASY www.BankExamsToday.com


P % Q means P is not smaller than Q
P * Q means P is neither smaller than nor equal to Q
P Q means P is neither greater than nor equal to Q
P $ Q means P is neither greater than nor smaller than Q
In each question four statements showing relationship have been given, which are
followed by four conclusions I, II, III and IV. Assuming that the given statements
are true, find out which conclusion (s) is/are definitely true.
6.

Statements:
Conclusions:

M D , D * K, K @ R, R * F
I. F K
II. D * F
III. M R
IV. D * R

a) None is true
b) Only I is true
c) Only II is true
d) Only III is true
e) Only IV is true
7.

Statements:
Conclusions:

B % K, K $ T, T * F, H F
I. B $ T
II. T B
III. H K
IV. F B

a) Only either I or II is true


b) Only III is true
c) Only IV is true
d) Only III and IV are true
e) Only either I or II and III and IV are true
8.

Statements:
Conclusions:

W * B, B @ F, F R, R $ M
I. W * F
II. M * B
III. R * B
IV. M * W

a) Only I and IV are true


b) Only II and III are true
c) Only I and III are true
d) Only II and IV are true
e) None of the above
9.

Statements:
Conclusions:

E @ K, K $ T, T N, B % N
I. T % E
II. K N
III. B * T
133

REASONING MADE EASY www.BankExamsToday.com


IV. B * E
a) Only I, II and III are true
b) Only II, III and IV are true
c) Only I, III and IV are true
d) All are true
e) None of the above
10.

Statements:
Conclusions:

Z $ B, B % M, M F, F @ R
I. Z * M
II. F * B
III. R * M
IV. M @ Z

a) Only I and II are true


b) Only I, III and IV are true
c) Only III and IV are true
d) Only either I or IV and III are true
e) None of the above
11.

Statements:
Conclusions:

H @ T, T $ N, F N, B % F
I. F @ H
II. F T
III. B * T
IV. B % H

a) None is true
b) Only I is true
c) Only II is true
d) Only II and IV are true
e) Only II and III are true
Directions (Q. 12-16) In the following questions, the symbols ^, #, %, @ and *
are used with the following meaning as illustrated below:
P # Q means P is neither greater nor smaller than Q.
P ^ Q means P is not smaller than Q.
P @ Q means P is neither smaller than nor equal to Q.
P * Q means P is not greater than Q.
P % Q means P is neither greater than nor equal to Q.
Now in each of the following questions assuming the given statements to be true,
find which of the three conclusions I, II and III given below them is/are definitely
true and give your answer accordingly.
12.

Statements:
Conclusions:

D % F, F @ H, H * N
I. N @ F
II. D % N
III. H % D

a) None is true
134

REASONING MADE EASY www.BankExamsToday.com


b) Only I is true
c) Only II is true
d) Only III is true
e) Only I and II are true
13.

Statements:
Conclusions:

B ^ D, D % T, T * M
I. B @ T
II. M @ D
III. B @ M

a) Only I is true
b) Only II is true
c) Only III is true
d) Only II and III are true
e) None of these
14.

Statements:
Conclusions:

K # W, M @ W, R ^ M
I. K % M
II. W % R
III. R @ K

a) Only I and II are true


b) Only I and III are true
c) Only II and III are true
d) All I, II and III are true
e) None of the above
15.

Statements:
Conclusions:

M @ K, K ^ T, T # J
I. J # K
II. M @ J
III. J % K

a) Only I is true
b) Only II is true
c) Only III is true
d) Only either I or III is true
e) Only either I or III and II are true
16.

Statements:
Conclusions:

R * N, N % B, B # T
I. B @ R
II. T @ N
III. R % T

a) Only I and II are true


b) Only I and III are true
c) Only II and III are true
d) All I, II and III are true
e) None of the above

135

REASONING MADE EASY www.BankExamsToday.com


Directions (Q. 17-21) In the following questions, the symbols @, , $, % and *
are uses with the following meaning as illustrated below.
P Q means P is not smaller than Q
P % Q means P is not greater than Q
P * Q means P is neither smaller than nor equal to Q.
P @ Q means P is neither greater than nor smaller than Q
P $ Q means P is neither greater than nor equal to Q
Now in each of the following questions assuming the given statements to be true,
find which of the conclusions I, II and III given below them is/are definitely true:
17.

Statements:
Conclusions:

F % T, T @ J, J * W
I. J @ F
II. J * F
III. W $ T

a) Only I is true
b) Only II is true
c) Only III is true
d) Only either I or II is true
e) Only either I or II and III are true
18.

Statements:
Conclusions:

R * D, D K, K $ M
I. M * R
II. K $ R
III. D * M

a) None is true
b) Only I is true
c) Only II is true
d) Only III is true
e) Only II and III are true
19.

Statements:
Conclusions:

Z F, F $ M, M % K
I. K * F
II. Z * M
III. K * Z

a) Only I is true
b) Only II is true
c) Only III is true
d) Only II and III are true
e) None of the above
20.

Statements:
Conclusions:

H @ B, B R, A $ R
I. B * A
II. R % H
III. A $ H

a) Only I and II are true


136

REASONING MADE EASY www.BankExamsToday.com


b) Only I and III are true
c) Only II and III are true
d) All I, II and III are true
e) None of the above
21.

Statements:
Conclusions:

M $ J, J * T, K T
I. K * J
II. M $ T
III. M $ K

a) None is true
b) Only I is true
c) Only II is true
d) Only III is true
e) Only II and III are true
Directions (Q. 22-27) In the following questions, the symbols @, , #, $ and *
are used with the following meaning illustrated.
P Q means P is not smaller than Q.
P * Q means P is neither greater than nor smaller than Q.
P @ Q means P is neither greater than nor equal to Q.
P $ Q means P is neither smaller than nor equal to Q.
P # Q means P is neither smaller than nor equal to Q.
In each of the following questions assuming the given statements to be true, find out
which of the three conclusions I, II and III given below them is/are definitely true.
22.

Statements:
Conclusions:

M @ T, T $ R, R J
I. J # M
II. R # M
III. J * T

a) Only I is true
b) Only II is true
c) Only III is true
d) Only I and II are true
e) None of the above
23.

Statements:
Conclusions:

D B, B # H, H * F
I. F @ B
II. F @ D
III. H @ D

a) Only I is true
b) Only II is true
c) Only III is true
d) Only I and II are true
e) All are true
137

REASONING MADE EASY www.BankExamsToday.com


24.

Statements:
Conclusions:

H * M, M @ T, T $ K
I. K # M
II. T # H
III. H @ K

a) Only I is true
b) Only I and II are true
c) Only II and III are true
d) Only I and III are true
e) All are true
25.

Statements:
Conclusions:

N $ A, A # J , J D
I. N @ J
II. A D
III. D @ A

a) Only I is true
b) Only II is true
c) Only III is true
d) Only II and III are true
e) None of the above
26.

Statements:
Conclusions:

R * T, T @ M, M $ K
I. K @ R
II. M # R
III. K # T

a) Only I and II are true


b) Only II and III are true
c) Only I and III are true
d) All are true
e) None of the above
27.

Statements:
Conclusions:

F # W, W $ M, M R
I. R $ W
II. F # R
III. W * R

a) None is true
b) Only I is true
c) Only II is true
d) Only III is true
e) Only II and III are true
Directions (Q. 28-31) Read each statement carefully and answer the following
questions.
28. Which of the following expressions will be true, if the expression R > O = A >
S < T is definitely true?
a) O > T
b) S < R
c) T > A
138

REASONING MADE EASY www.BankExamsToday.com


d) S = O

e) None of these

29. Which of the following symbols should replace the question mark (?) in the
given expression in order to make the expressions P > A as well as T < L
definitely true?
a)
b) >
c) <
d)
e) None of these
30. Which of the following should be placed in the blank spaces respectively (in the
same order from left to right) in order to complete the given expression in such
a manner that makes the expression A < P definitely false?
__ __ < __ > __
a) L, N, P, A
b) L, A, P, N
c) A, L, P, N
d) N, A, P, L
e) P, N, A, L
31. Which of the following symbols should be placed in the blank spaces
respectively (in the same order from left to right) in order to complete the given
expression in such a manner that makes t he expression F > N and U > D
definitely false?
F __ O __ U __ N __ D
a) <, <, >, =
b) <, =, =, >
c) <, =, =, <
d) , =, =,
e) >, >, =, <
Answers:
1.

Option B
K@VK <V
VN V N
N % F N F
K <V N F
Conclusions: I. F @ V F < V (F)
II . K @ N K < N (T)
Only II is true.
2.

Optoin E
H W H W
W$ MW= M
M @ BM < B
H W= M < B
Conclusions: I. B * H B > H (T)
II. M % H M H (T)
So, both I and II are true.
3.

Option D
D%BD B

139

REASONING MADE EASY www.BankExamsToday.com


B *T B>T
T$MT=M
DB > T=M
Conclusions: I. T D T D (F)
II. M D M D (F)
Neither I nor II is true.
4.

Option A
M*TM>T
T@KT<K
KNKN
M>T<KN
Conclusions: I. N * T N > T (T)
II. N * M N > M (F)
Only I is true.
5.

Option C
R$JR=J
J%DJD
D*FD>F
R=JD>F
Conclusions: I. D $ R D = R
II. D @ R D < R
Either I or II is true.
6.

Option A
MDM<D
D*KD>K
K@RKR
R*FR>F
Conclusions : I. F K F < K (False)
II. D * F D > F (False)
III. M R M < R (False)
IV. D * R D > R (False)
None is true.
7.

Option E
B%KBK
K$TK=T
T*FT>F
HFH<F
So, B K = T > F > H
Conclusions: I. B $ T B = T
II. T B T < B
III. H K H < K (True)

140

REASONING MADE EASY www.BankExamsToday.com


IV. F B F < B (True)
Either T is smaller than B or equal to.
So, only III, IV and either I or II are true.
8.

Option B
W*BW>B
B@FB F
FRF<R
R$MR=M
So, W > B F < R = M
Conclusions: I. W * F W > F (False)
II. M * B M > B (True)
III. R * B R > B (True)
IV. M * W M > W (False)
So, Only II and III are true.
9.

Option C
E@KEK
K$TK=T
TNT<N
B%NBN
Conclusions: I. T % E T E (True)
II. K N K < N (False)
III. B * T B > T (True)
IV. B * E B > E (True)
Only I, III and IV are true.
10.

Option C
Z$BZ=B
B%MBM
MFFR
F@RFR
Conclusions: I. Z * M Z > M (False)
II. F * B F > B (False)
III. R * M R > M (True)
IV. M @ Z M Z (True)
Only III and IV are true.
11.

Option C
H@THT
T$NT=N
FNF<N
B%FBF
So, H T = N > F B
Conclusions: I. F @ H F H (False)

141

REASONING MADE EASY www.BankExamsToday.com

Only II is true.

II. F T F < T (True)


III. B * T B > T (False)
IV. B % H B H (False)

12.
13.
14.
15.
16.
17.

Option A
Option B
Option D
Option E
Option D
Option E
F%TFT
T@JT=J
J*WJ>W
From all the statements together,
FT=J>W
Conclusions: I. J @ F J = F
II. J * F J > F
III. W $ T W < T
J is either greater than or equal to F. So either I or II and III are true.

18.

Option C
R*DR>D
DKDK
K$MK<M
From all the statements together,
R> D K < M
Conclusions: I. M * R M > R (False)
II. K $ R K < R (True)
III. D * M D > M (False)
So, only II is true.

19.

Option A
ZFZF
F$MF<M
M%KMK
From all the statements together,
ZF<MK
Conclusions: I. K * F K > F (True)
II. Z * M Z > M (False)
III. K * Z K > Z (False)
So, only I is true.

20.

Option D
H@BH=B

142

REASONING MADE EASY www.BankExamsToday.com


BRBR
A$RA<R
From all the three statements together,
H=BR>A
Conclusions: I. B * A B > A (True)
II. R % H R H (True)
III. A $ H A < H (True)
So, all I, II and III are true.
21.

Option A
M$JM<J
J*TJ>T
KTKT
From all the three statements together,
M<J<TK
Conclusions: I. K * J K > J (False)
II. M $ T M < T (False)
III. M $ K M < K (False)
None is true.

22.
23.
24.
25.
26.
27.
28.
29.
30.
31.

Option B
Option E
Option E
Option C
Option B
Option A
Option B
Option B
Option E
Option C

143

REASONING MADE EASY www.BankExamsToday.com

Chapter 9
Coding Decoding
Directions (Q. 1-3) Study the information given below carefully to answer the
following questions.
In a certain code language the following lines written as lop eop aop fop
means Traders are above laws
fop cop bop gop means Developers were above profitable
aop bop uop qop means Developers stopped following traders
cop jop eop uop means Following maps were laws
1.

Developers are following laws would be correctly written as


a) bop cop uop eop
b) lop bop eop uop
c) oup cop lop aop
d) gop cop uop qop
e) None of these

2.

qop gop cop eop would correctly mean


a) profitable laws were stopped
b) developers stopped following laws
c) traders were above profitable
d) were laws profitable traders
e) None of these

3.

aop qop bop would correctly mean


a) following were above
b) traders stopped developers
c) developers are laws
d) traders above stopped
e) laws are stopped

Directions (Q. 4-8) Study the information and answer the following questions.
In a certain code language.
Economics is not money is writen as ka la ho ga
Demand and supply economies is written as mo ta pa ka
Money makes only part is written as zi la ne ki
Demand makes supply economics is written as zi mo ka ta
4.

What is the code for money in the given code language?


a) ga
b) mo
c) pa
d) ta
e) la

5.

What is the code for supply in the given code language?


144

REASONING MADE EASY www.BankExamsToday.com


a) Only ta
d) Only pa

b) Only mo
e) Either mo or ta

c) Either pa or mo

6. What may be the possible code for demand only more in the given code
language?
a) xi ne mo
b) mo zi ne
c) ki ne mo
d) mo zi ki
e) xi ka ta
7. What may be the possible code for work and money in the given code
language?
a) pa ga la
b) pa la lu
c) mo la pa
d) tu la ga
e) pa la ne
8.

What is the code for makes in the given code language?


a) mo
b) pa
c) ne
d) ho
e) None of these

Directions (Q. 9-13) Study the following information and answer the given
questions.
In a certain code language driving is not easy is written as jo ro ho go,
rough and tough driving is written as no sa jo da, looks easy but dangerous is
written as ai ro to po and is rough tough dangerous driving is written as ho jo no
ai da
9.

What is the code for easy in the given code language?


a) go
b) no
c) to
d) ro
e) so

10.

What is the code for tough in the given code language?


a) no
b) da
c) sa
d) Either sa or da
e) Either no or da

11.

What may be the possible code for rough but nice?


a) po bi no
b) no po ai
c) no to po
d) no to ai
e) ta jo bi

12.

What may be the possible code for easy and solved in the given code
language?
a) sa go ro
b) sa ro cu
c) no ro su
d) cu ro go
e) sa ro to

13.

What is the code for dangerous in the given code language?


a) no
b) sa
c) to
d) ai
e) ho

145

REASONING MADE EASY www.BankExamsToday.com


Directions (Q. 14-18) Study the following information carefully and answer the
questions.
In a certain code language, newspaper enhance the knowledge is written
as yo no co po, new enhance good version is written as co so ro jo, good
knowledge is necessary is written as bo t o ro po, the new brand book is written
as qo jo do yo, and book is necessary is written as bo do to.
14.

What is the code for version?


a) so
b) ro
d) Cant be determined
e) None of these

c) jo

15.

Which of the following is the code for brand enhance version?


a) do qo co
b) so qo co
c) do ro co
d) Cant say
e) None of these

16.

What is the code for new?


a) so
b) ro
d) yo
e) None of these

c) jo

What is the code for necessary?


a) bo
b) to
d) Cant say
e) None of these

c) ro

necessary is tal ent willl be coded as


a) mo bo to
b) mo lo to
d) cant say
e) None of these

c) bo to no

17.

18.

Directions (Q. 19-22) Study the following information to answer the given
questions.
In a certain code, ze lo ka gi is a code for must save some money, fe ka
so ni is a code for he made good money, ni lo da so is a code for he must be
good and we so ze da is a code for be good save grace.
19.

20.

21.

Which of the following is the code of must?


a) so
b) da
d) ni
e) None of these

c) lo

What does the code ze stand for?


a) some
b) must
d) grave
e) save

c) be

Which of the following is the code of good?


a) so
b) we
d) lo
e) fe

c) ze

146

REASONING MADE EASY www.BankExamsToday.com


22.

grace of money may be coded as


a) ka da fe
b) we ka so
d) ka we yo
e) ja ka ze

c) ja da we

Directions (Q. 23-27) In each question below is given a group of letters followed
by four combinations of digits/symbols numbered a, b, c, and d. You have to
find out which of the combinations correctly represents the group of letters
based on the following coding system and the conditions and mark the number
of that combination as your answer. If none of the combinations correctly
represents the group of letter, mark e) i.e., None of these as your answer.

Letter
Digit/Symbol Code

RATKFQEPJIMUDH
3@2194%58$67@

Conditions:
(i)
If the first letter is a consonant and the last letter is a vowel, their codes
are to be interchanged.
(ii)
If both the first and the last letters are vowels, both are to be coded as *.
(iii)
If both the first and the last letters are consonants, both are to be coded
as the code for the last letter.
23.

24.

25.

26.

27.

28.

DPEHQA
a) 7%#4@
d) @%#4@

b) @%#47
e) None of these

c) 7%#47

KEMRDF
a) *$379
d) *$37*

b) 1$379
e) None of these

c) 1$37*

AHDUQK
a) 1#764@
d) @#764@

b) 1#7641
e) None of these

c) *#764*

IDQJPT
a) 8745%2
d) 2745%2

b) 2745%8
e) None of these

c) *745%*

UDKFME
a) *719$
d) 719#$6

b) 6719$
e) None of these

c) *719$*

If white means black, black means red, red means blue. blue
means yellow and yellow means grey then which of the following
represents the colour of clear sky?
147

REASONING MADE EASY www.BankExamsToday.com


a) Blue
d) Cant be determined
29.

b) Red
e) None of these

c) Yellow

In a certain code, MODEL is written as 513#2 and DEAR is written as


3#%8. How is LOAD written in that code?
a) 21%3
b) 23%1
c) 25%3
d) 21#3
e) None of these

Directions (Q. 30-34) In each question below is given a group of letters followed
by four combinations of digits/symbols numbered a, b, c and d. You have to
find out which of the combinations correctly represents the group of letters
based on the following coding system and the conditions and mark the number
of that combination as your answer. If none of the four combinations correctly
represents the group of letters, give e) i.e. None of these as the answer.
Letters:
Digit/Symbold Code:

M D R P A T W E I F H U K
5 6 # 7 8 1 @ $ 2 % 3 4

Z
9

Conditions:
(i)
If the first letter is a coinsonant and the last letter is a vowel, their
codes are to be interchanged.
(ii)
If both the first and the last letters are vowels, both are to be coded as *.
(iii)
If the first letter is a vowel and the last letter is a consonant, both are to
be coded as the code for the consonant.
30.

31.

32.

33.

34.

TUKDIP
a) 14627
d) 16427

b) 14621
e) None of these

c) 74621

EFDMKA
a) $%6548
d) 8%6548

b) $%654$
e) None of these

c) *%654*

APWTUH
a) *7@1 *
d) 87@18

b) 87@13
e) None of the above

c) 37@@5

MARTWE
a) 58#1@$
d) $8#1@$

b) 58#1@5
e) None of these

c) $8#1@5

HEMKZI
a) 2$5493
d) 2$5492

b) 3$5492
e) None of these

c) 3$5493

148

REASONING MADE EASY www.BankExamsToday.com


35.

In a certain code SAFER is written as 5@3#2 and RIDE is written as


2%#. How is FEDS written in that code?
a) 3#5
b) 3@%5
c) 3#%5
d) 3#%2
e) None of these

36.

In a certain code CONFUSED is written as EMBEFTV. How is


SECLUDED written in that code?
a) RDBKEFEV
b) KBDRCDCT
c) KBDREFEV
d) MDFTCDCT
e) None of these

37.

In a certain code language bring the white board is written as ka na di pa


and white and black board is written as na di sa ra. How is the written
in that code?
a) ka
b) pa
c) ka or pa
d) Data inadequate
e) None of these

Directions (Q. 38-43) In each question below is given a group of letters followed
by four combinations of digits/symbols numbered a, b, c, and d. You have to
find out which of the combinations correctly represents the group of letter,
based on the following coding system and the conditions that follow and mark
the number of thatcombination as your answer. If none of the combination
correctly represents the group of letter, mark e) i.e. None of these as your
answer.
Letter:
Digit/Symbol Code:

RDAEJMKTBUIPWHF
4 8 5 $ * 1 2 6 % 7@ 3 9 #

Conditions:
(i)
If the first letter is a consonant and the last letter is a vowel both are to
be coded as ^.
(ii)
If both the first and the last letters are consonants both are to be coded
as the code for the last letter.
(iii)
If the first letter is a vowel and the last letter is a consonant their codes
are to be interchanged.
38.

39.

40.

HIFMJU
a) 7#1*9
d) ^7#1*^

b) 97#1*9
e) None of these

c) 7#1*

AKTRBW
a) 3264%5
d) 5264%3

b) 3264%3
e) None of these

c) 324%65

EBPDRI
a) 7%@847
d) $%8@47

b) 7%@84$
e) None of these

c) $%@847
149

REASONING MADE EASY www.BankExamsToday.com

41.

42.

43.

BKAJIM
a) %25*71
d) 15*271

b) 125*7%
e) None of these

UKPDMA
a) 52@815
d) @2815

b) 2@815
e) None of these

METUFB
a) %$6#1
d) 1$6#%

b) 1$6#1
e) None of these

c) %25*7%

c) 2@81

c) %$6#%

Directions (Q. 44-48) In each question below is given a group of letters followed
by four combinations of digits/symbols numbered a, b, c and d. You have to
find out whi ch of the combinations correctly represents the group of letters
based on the following conditions and the sub-conditions and mark the number
of that combination as your answer. If none of the four combinations correctly
represents the group of letters, mark e) i.e. None of these as your answer.
Letter:
A M P R D H J T W E I KN F U
Digit/Symbol: 3 % 4 9 1 @ 2 5 6 * 7 8 $ ^
Conditions:
(i)
If the first letter is a consonant and the last letter is a vowel, their codes
are to be interchanged.
(ii)
If both the first as well as last letters are consonants, both are to be
coded as the code for the last letter.
(iii)
If the first letter is a vowel and the last letter is a consonant, both are to
be coded as #.
44. APIWKN
a) 34*73
d) 84*73
45.

46.

47.

b) #4*7#
e) None of these

c) 84*78

HPEIUM
a) @46*^%
d) #46*^#

b) %46*^
e) None of these

c) @46*^@

RDEMKH
a) @16%79
d) 916%79

b) 916%7@
e) None of these

c) @16%7@

FWHRKE
a) 6@976
d) $@976

b) 6$ @97
e) None of these

c) 6@97$
150

REASONING MADE EASY www.BankExamsToday.com

48.

IDAPRU
a) *1349^
d) ^1349^

b) ^*1349
e) None of these

c) ^1349*

49.

In a certain code CROWNED is written as PSDVEFO. How is STREAMS


written in that code?
a) SUTDBNT
b) TUSDTNB
c) SUTFTNB
d) QSRDTNB
e) None of these

50.

DEAN is related to NDAE and ROAD is related to DRAO in the


same way as SOME is related to
a) ESMO
b) EOMS
c) EMOS
d) MSEO
e) None of these

51.

In a certain code TRAIN is written as 39*7% and MEAL is written as


4$*@. How is ITEM written in that code?
a) 7$34
b) 73$4
c) 79$4
d) 73*4
e) None of the above

52.

In a certain code language over and above is written as da pa ta and old


and beautiful is written as sa na pa. How is over written in that code
language?
a) da
b) ta
c) na
d) da or ta
e) None of these

Directions (Q. 53-57) Study the following information carefully and answer the
questions given below
In a certain code language, exercise for your health is written as ja ka na se
Health and exercise must is written as ka se re tu
Must to exercise regularly is written as az ka dk tu
To your good life is written as mo ja fu az
53.

Which of the following may represent exercise regularly good for health?
a) Ja m o re dk az
b) fu ja re tu dk
c) ka se mo na dk
d) ka re az tu se
e) na se tu az dk

54.

What is the code for life?


a) mo
b) fu
d) az
e) Either mo or fu

55.

c) ja

What is the code for must exercise?


151

REASONING MADE EASY www.BankExamsToday.com


a) tu se
d) fu tu

b) ka tu
e) na ka

c) ka az

56.

Find the odd one out.


a) January, May
b) April, June
c) July, August
d) January, December
e) None of these

57.

Which one of the given responses would be a meaningful order of the


following?
1. Earth
2. Jupiter
3. Venus
4. Mars
5. Mercury
a) 5, 3, 1, 2, 4
b) 5, 3, 4, 1, 2
c) 5, 3, 1, 4, 2
d) 5, 3, 2, 4, 1
e) None of these

58.

If NEWS is written as WENS then how MATE will be written in this


code?
a) TAME
b) META
c) EATM
d) AMET
e) None of these

59.

If REASON is coded as 5 and BELIEVED as 7, what is the code number


for GOVERNMENT?
a) 6
b) 8
c) 9
d) 10
e) None of these

Directions (Q. 60-65) In each question below is given a group of letters followed
by four combinations of digits/symbols numbered a, b, c and d. you have to
find out which of the combinations correctly represents the group of letters
based on the following digits/symbols coding system and the condition those
follow and mark the number of that combination as the answer. If none of the
combinations correctly represents the group of letters mark e) i.e. None of
these as the answer.
Letter:
Digit/Symbol Code

PMAI DEJK F N Q B U W T
695 #7$1%2 @8 3* 4

Conditions:
(i)
If the first letter is a consonant and the last letter is a vowel the codes
are to be interchanged.
(ii)
If the first letter is a vowel and the last letter is a consonant both are to
be coded as the code for the vowel.
152

REASONING MADE EASY www.BankExamsToday.com


(iii)

60.

61.

62.

63.

64.

65.

66.

If both the first and the last letters are consonants both are to be coded
as the code for the last letter.
MKJIDE
a) 9%1#7$
d) $%17#9

b) $%1#79
e) None of these

c) 91%#7$

INQBWU
a) #@8*3
d) 3#@8*

b) 3@8*#
e) None of these

c) #8@*3

KFBPAW
a) *265%
d) %265*

b) % 265%
e) None of these

EFDJTP
a) 62714$
d) $2714$

b) $27146
e) None of these

c) $27416

NWANUD
a) @*5@37
d) @*5@3@

b) 7*5@3@
e) None of these

c) @5*@37

APFTQI
a) #62485
d) 56248#

b ) #6248#
e) None of these

c) 562485

c) *265*

In a certain code if LOUD is written as JQSF then which of the following


English words shall b e coded as PKQG?
a) RISE
b) ROPE
c) ROAD
d) RICE
e) ROLE

Directions (Q. 67-72) In each question below is given a group of letters followed
by four combinations of digits/symbols numbered a, b, c, d, you have to find
out which of the combinations correctly represents the group of letters based
on the following letter coding system and mark the number of that
combination as the answer. If none of the digits/symbols combinations
correctly represents the group of letters, mark e) i.e., None of these as the
answer.
Letter:
DLEGZKRUBWFHIAP
Digit/Symbol: 4 8 $ 1 # 5 7 2 6 % * 3 9 @
Conditions:
(i)
If the first letter is a vowel and the last is a consonant, both are to be
coded as the code of the consonant.
153

REASONING MADE EASY www.BankExamsToday.com


(ii)
(iii)

67.

68.

69.

70.

71.

72.

If both first and the last letters are consonants, both are to be coded as ^.
If first letter is a consonant and last is a vowel, the codes for first and
last letters are to be interchanged.
ABWUPF
a) %26@%
d) %269@

b) %26@9
e) None of these

c) 926@%

BFIDWE
a) 2%346$
d) $2%634

b) %3426$
e) None of these

c) $%3426

WKGLBA
a) 951862
d) 951826

b) 651829
e) None of these

c) 651892

ZEFHIR
a) #$%*37
d) #%$*37

b) ^$%*3^
e) None of these

c) 7$%*3#

ELBGPU
a) $821@
d) $812@

b) 821@$
e) None of these

c) $821@

UHRKLZ
a) *758
d) #*758

b) *758#
e) None of these

c) #*758#

73.

In a certain code, LAWN is written as JCUP. How will SLIT be coded in


that code?
a) QNGV
b) QJGV
c) ONVG
d) NJGV
e) NJVG

74.

In a certain code PRAISE is written as #@$27% and RESPIRE is written as


@7#2@%. How is REPAIR written in that code?
a) @%#2$@
b) @%$#2@
c) @%#$2@
d) @%$2#@
e) None of these

75.

In a certain code SATELLITE is written as FUBTLDSHK. How is


LAUNCHING written in that code?
a) DOUBFGMHO
b) OVBMCFMHG
c) OVMBCFMGH
e) DOUBCFMHG
e) None of these
154

REASONING MADE EASY www.BankExamsToday.com


Directions (Q. 76-81) In each of these questions, a group of letters is given
followed by four combinations of digit/symbol numbered a, b, c, d. Letter are
to be coded as per the scheme and conditions given below. You have to find
out the serial number of the combination, which represents the letter group. Se
rial number of that combination is your answer. If none of the combinations is
correct, your answer is e) i.e., None of these
Letters

F
Digit/Symbol 9
Code

A
2

K
$

E
M
*

S
@

P
8

L
1

Conditions:
i) If the first letter is a vowel and the last a consonant, both are to be coded as
the code for the vowel.
ii) If the first letter is consonant and the last a vowel, the codes for the first and
the last letters are to be interchanged.
iii) If both the first and the last letters are consonant both are to be coded as ^
iv) If more than two vowels are there in the group of letters, all vowels are to
coded as -76.

77.

78.

79.

80.

81.

BSQEGU
a) 5@67%
d) 5@67%

b) ^@67%^
e) None of these

c) @67%

MGSELI
a) *%@71#
d) %#@17*

b) *%@17#
e) None of these

c) #%@71*

IPKUSR
a) #8$@#
d) #8$4@

b) 48$@#
e) None of these

c) #8$@4

TEFSUM
a) ^79@^
d) ^79@^

b) 379@*
e) None of these

c) *79@3

KAGFUB
a) $2%95
d) ^2%9^

b) 52%9$
e) None of these

c) $2%95

AQMMTE
a) 76*32
d) 26*23

b) 6--*3-e) None of these

c) 26*32

155

REASONING MADE EASY www.BankExamsToday.com


82.

In a certain code JUST is written as #@%$ and LATE is written as ^$*.


How is TASTE written in that code?
a) *^%$*
b) $^%$*
c) $^%*$
d) $%^%*
e) None of these

83.

If Blue is called Green, Green is called Orange, Orange is called Yellow,


Yellow is called Black, Black is called Red and Red is called White. What
is the colour of turmeric?
a) Orange
b) Green
c) White
d) Black
e) None of these

84.

In a certain code ROSE is wrtitten as #43$ and FIRST is written as 5*#37.


How is STORE written in that code?
a) 473$#
b) 473#$
c) 374#$
d) 347#$
e) None of these

85.

In a certain code, DESPAIR is written as TFEQSJB. How is NUMERAL


written in that code?
a) OVNFMBS
b) NVOFSBN
c) NVOMFBS
d) NVOFMBS
e) None of these

86.

If yellow is called red, red is called blue, blue is called white,


white is called black, black is called green and green is called
violet, what is the colour of clear sky?
a) Green
b) Violet
c) Yellow
d) Red
e) None of these

87.

In a certain code, STAR is written as 5$*2 and TORE is written as $32@.


How is OATS written in that code?
a) 3*5$
b) 3*$5
c) 3$*5
d) 35*$
e) None of these

88.

In a certain code language, food is good is written as ho na ta, eat food


regularly is written as sa ta la and keep good health is written as da na
ja. How is eat written in that code language?
a) sa
b) la
c) sa or la
d) Data inadequate
e) None of these

89.

In a certain code, CORDIAL is written as SPDCMBJ. How is


SOMEDAY written in that code?
a) NPTDEBZ
b) NPTFZBE
c) TPNDZBE
d) NPTDZBE
e) None of these

90.

In a certain code DOES is written as 5$3% and SITE is written as


%4#3. How is EDIT written in that code?
a) 354#
b) 3#54
c) 3$4#
156

REASONING MADE EASY www.BankExamsToday.com


d) 35$#

e) None of these

Answers:
1.
2.
3.
4.
5.
6.
7.
8.
9.
10.
11.
12.
13.
14.
15.
16.
17.
18.
19.
20.
21.
22.
23.

24.

25.

26.

Option B
Option A
Option B
Option E
Option E
Option A
Option B
Option D
Option D
Option E
Option A
Option B
Option D
Option A
Option B
Option C
Option D
Option A
Option C
Option E
Option A
Option D
Option B
D
P
@
%

H
#

Q
4

A
7

Option E
K
E
M
R
9

$
3
Condition (iii) follows

D
7

F
9

Option E
A
H
D
@
#
7
No condition follows.

U
6

Q
4

K
1

P
%

T
2

Option A
I
D
Q
8
7
4
No condition follows.

157

REASONING MADE EASY www.BankExamsToday.com


27.

Option C
U
D
K
*
7
1
Condition (ii) follows

F
9

M
$

E
*

28.

Option C
Colour of clear sky is blue and according to question, blue means yellow so
the colour of clear sky will be yellow.

29.

Option A
M
O
5
1

D
3

E
#

D
3

E
#

A
%

R
8

L
2

O
1

A
%

D
3

30.

31.

32.

33.

34.

35.

Option A
T
U
K
1

4
Condition (i) follows.

L
2

D
6

I
2

P
7

Option C
E
F
D
M
*
%
6
5
Condition (ii) follows.

K
4

A
*

Option E
A
P
W
T
3
7
@
1
Condition (iii) follows.

H
3

Option C
M
A
R
$
8
#
Condition (i) follows.

T
1

W
@

E
5

Option A
H
E
M
2
$
5
Condition (i) follows.

K
4

Z
9

I
3

Option C
158

REASONING MADE EASY www.BankExamsToday.com


S
5

A
@

F
3

E
#

R
2

D
%

E
#

F
3

E
#

D
%

S
5

R
2

36.

Option C

37.

Option C
Bring the white board ka na di pa
White and black board na di sa ra
From equations (i) and (ii),
White board = na di
Put the code of white board in equation (i)
Bring the = ka or pa

38.

39.

40.

41.

42.

43.

Option D
H
I
F
^
7
#
Condition (i) follows.

M
1

J
*

U
^

Option A
A
K
T
R
3
2
6
4
Condition (iii) follows.

B
%

W
5

Option C
E
B
P
$
%
@
No condition follows.

D
8

R
4

I
7

Option E
B
K
A
J
1
2
5
*
Condition (ii) follows.

I
7

M
1

Option B
U
K

P
@

D
8

M
1

A
5

Option C
M
E
%
$

T
6

F
#

B
%

(i)
(ii)

159

REASONING MADE EASY www.BankExamsToday.com


Condition (ii) follows.
44.

45.

46.

47.

48.

Option B
A
P
I
W
#
4
*

Condition (iii) follows.

K
7

N
#

Option E
H
P
E
I
%
4
6
*
Condition (ii) follows.

U
^

M
%

Option C
R
D
E
M
@
1
6
%
Condition (ii) follows.

K
7

H
@

Option C
F
W
H
6

@
Condition (i) follows.

R
9

K
7

E
$

R
9

U
^

Option A
I
D
*
1

A
3

P
4

Option E
Option A
1
2
D
E

3
A

4
N

4
N

1
D

3
A

2
E

And

1
R

2
O

3
A

4
D

4
D

1
R

3
A

2
O

So,

1
S

2
O

3
M

4
E

4
E

1
S

3
M

2
O

51.

Option B
T
R
3
9
M
E
4
$
I
T
7
3

A
*
A
*
E
$

I
7
L
@
M
4

49.
50.

And
So,

52.

N
%

Option D
160

REASONING MADE EASY www.BankExamsToday.com


Over and above
da pa ta
Old and beautiful
sa na pa
So, the code of over will be da or ta.
53.

54.

55.

Option C
Exercise ka
Regularly dk
Good mo or fu
For na
Health se

Option E
The code for life may be mo or fu.

56.

Option B
Must tu
Exercise ka

57.

Option B

58.

Option A
1
2
N
E
Similarly,
1
2
M
A

59.

60.
61.
62.
63.
64.
65.
66.
67.

Optoin B
Except the months of April and June, all other months have 31 days each.

3
W

4
S

3
W

2
E

1
N

4
S

3
T

4
E

3
T

2
A

1
M

4
E

Option C
REASON 5
Number of letters 1
BELIEVED 8 1 = 7
Similarly,
GOVERNMENT 10 1 = 9
Option B
Option A
Option C
Option D
Option E
Option D
Option C
Option A

161

REASONING MADE EASY www.BankExamsToday.com


68.
69.
70.
71.
72.
73.
74.
75.
76.
77.
78.
79.
80.
81.
82.
83.
84.
85.
86.
87.
88.
89.
90.

Option E
Option D
Option B
Option A
Option C
Option A
Option C
Option B
Option E
Option C
Option D
Option A
Option D
Option B
Option B
Option D
Option C
Option D
Option E
Option B
Option C
Option D
Option A

162

REASONING MADE EASY www.BankExamsToday.com

Chapter 10
Decision Making
Directions (Q. 1-5) Read the following information carefully and answer the
questions given below:
Following are the conditions for admitting students in an Engineering
College:
(i)
Have secured at least 75% agreegate marks in XII Std. exam in science
stream.
(ii)
Have secured an average of at least 85% marks in Physics, Chemistry
and Mathematics in XII Std. exam.
(iii)
Pay Rs.50, 000 at the time of admission.
(iv)
Be at least 17 years old as on 01st May 2014
However, in the case of a candidate who fulfils all the criteria except
(a) (ii) Above, but has secured 90% aggregate marks, he/she should be referred
to the Principal.
(b) (iii) Above, but pays at least Rs.40000 he/she should be referred to the
Chairman, Admission.
Based on the above criteria and information provided below, make a decision in
each case. You are not to assume anything. If the data given are not enough to take
a decision mark your answer as data I nadequate. These cases are given to you as
on 01.05.2014. Give answer
a)
b)
c)
d)
e)

If the case is to be referred to the Principal


If the student is to be admitted
If the data are inadequate to take a decision
If the student is not to be admitted
If the case is to be referred to the Chairman Admission.

1.

Nisha Malhotra has secured aggregate 80% marks I n XII Std. exam in
science stream. She can pay Rs.40,000 at the time of admission. She has
secured 86% average marks in Physics, Chemistry and Mathematics.
a) If the case is to be referred to the Principal
b) If the student is to be admitted
c) If the data are inadequate to take a decision
d) If the student is not to be admitted
e) If the case is to be referred to the Chairman Admission.

2.

Arindam Ghosh was born on 25 August 1995. He can pay Rs.60,000 at the
time of admission. He has secured 80% aggregate marks in XII Std. exam
163

REASONING MADE EASY www.BankExamsToday.com


in science stream and also 85% average marks in Physics, Chemistry and
Mathematics.
a) If the case is to be referred to the Principal
b) If the student is to be admitted
c) If the data are inadequate to take a decision
d) If the student is not to be admitted
e) If the case is to be referred to the Chairman Admission.
3.

Arun Khanna has secured 90% aggregate marks in XII Std. exam in science
stream and he can also pay Rs.50,000 at the time of admission. He was born
on Feb. 20, 1996. He has secured 89% average marks in Physics, Chemistry
and Mathematics.
a) If the case is to be referred to the Principal
b) If the student is to be admitted
c) If the data are inadequate to take a decision
d) If the student is not to be admitted
e) If the case is to be referred to the Chairman Admission.

4.

Kiran Thapar was born on July 15, 1994. He can pay Rs.40,000 at the time
of admission. He has secured 79% average in Physics, Chemistry and
Mathematics. He has secured 86% aggregate marks in XII Std. exam in
science stream.
a) If the case is to be referred to the Principal
b) If the student is to be admitted
c) If the data are inadequate to take a decision
d) If the student is not to be admitted
e) If the case is to be referred to the Chairman Admission.

5.

Vimal Upadhyaya was born on September 20, 1996. He has secured 78%
aggregate marks in XII Std. exam in science stream. He can pay Rs.42,000
at the time of admission. He has secured 88% marks in Physics, Chemistry
and Mathematics.
a) If the case is to be referred to the Principal
b) If the student is to be admitted
c) If the data are inadequate to take a decision
d) If the student is not to be admitted
e) If the case is to be referred to the Chairman Admission.

Directions (Q. 6-10) Study the following information carefully and answer the
questions given below.
Following are the conditions for selecting Marketing Manager in an organization:
The candidate must ____
(i)
Be at least 30 years old as on 01.03.2009
(ii)
Have secured at least 55% marks in graduation
164

REASONING MADE EASY www.BankExamsToday.com


(iii)
(iv)
(v)

Have secured at least 60% marks in Post Graduate4 Degree/Diploma in


Marketing.
Have post qualification work experience of at least five years in th e
Marketing Division of an organization
Have secured at least 45% marks in the selection process.

In the case of a candidates who satisfies all other conditions except


(a) At (iv) above, but has post qualification work experience of at least two
years as Deputy Marketing Manager, the case is to be referred to GMMarketing.
(b) At (ii) above, but has secured at least 65% marks in Post Graduate Degree
/Diploma in Marketing Management, the case is to be referred to Vice
President-Marketing.
In each question below is given details of one candidate. You have to take one of
the following courses of action based on the information provided and the
conditions and sub-conditions given above and mark your answer accordingly. You
are not to assume anything other than the information provided in each question. All
these cases are given to you as on 01.03.2009.
Give answer:
a) If the candidate is not to be selected.
b) If the candidate is to be selected.
c) If the data are inadequate to take a decision.
d) If the case is to be referred to Vice President-Marketing.
e) If the case is to be referred to GM-Marketing.
6. Suresh Mehta has secured 58% marks in Graduation. He was born on 19th May
1975. He has secured 50% marks in the selection process. He h as been working
for the past seven years in the Marketing devision of an organization after
completing his Post Graduation with 62% marks.
7. Sudha Gopalan has secured 50% marks in both selection process and
Graduation. She has been working for the past six years in the marketing
division of an organization after completing her Post Graduate Diploma in
Marketing with 70% marks. She was born on 14th October 1978.
8. Divya Kohli has been working for the past five years in Marketing Division of
an organization after completing her Post Graduate Diploma in Marketing with
65% marks. She has secured 55% marks in Graduation and 50% marks in the
selection process. She was born on 02nd April 1979.
9. Navin Marathe was born on 08th April 1975. He has secured 60% marks in both
Graduation and Post Graduate Degree in Marketing. He has been working for
the past six years in the Marketing division of an organization after completing
165

REASONING MADE EASY www.BankExamsToday.com


his PG Degree in Marketing. He has secured 50% marks in the selection
process.
10. Varun Malhotra was born on 03rd July 1976. He has been working as Deputy
Marketing Manager in an organization for the past three years after completing
his Post Graduate Degree in Marketing with 65% marks. He secured 55%
marks in both Graduation and selection process.
Directions (Q. 11-20) Study the following information carefully and answer the
questions given below.
Following are the conditions for selecting Personnel Manager in an organization:
The candidate must:
(i)
Be a graduate with at least 50% marks.
(ii)
Have a Post Graduate Degree/Diploma in personnel Management /HR
with at least 60% marks.
(iii)
Not be more than 35 years as on 01.06.2009
(iv)
Have post qualification work experience of at least five years in the
Personnel/HR Division of an organisation.
(v)
Have secured at least 45% marks in the selection process.
In the case of a candidate who satisfies all the conditions except(a) (iii) Above, but has post-qualification work experience of at least ten years,
the case is to be referred to the Director-Personnel.
(b) At (iv) above, but has post-qualification work experience as Deputy
Manager of at least three years, th e case is to be referred to PresidentPersonnel.
In each question below are given details of one candidate. You have to take one of
the following courses of action based on the information provided and the
conditions and sub-conditions given above and mark the number of that course of
action as your answer. You are not to assume anything other than the information
provided in each question. All these cases are given to you as on 01.06.2009.
Give answer:
a) If the candidate is to be selected.
b) If the candidate is not to be selected.
c) If the information provided is inadequate to take a decision.
d) If the case is to be referred to the Director-Personnel.
e) If the case is to be referred to the President-Personnel.
11. Meena Srivastava was born on 06th March 1978. She has been working as
Deputy Personnel Manager in an organization for the past four years after
completing her Post Graduate Diploma in HR with 68% marks. She has
secured 50% marks in both graduation and selection process.
166

REASONING MADE EASY www.BankExamsToday.com

12. Ketan Desai was born on 05th January 1979. He has been working for the past
five years in the personnel department of an organisation after completing his
Post Graduate Diploma in Personnel Management with 64 % marks. He has
secured 40% marks in the selection process and 52% marks in Graduation.
13. Anant Joshi has been working in the Personnel Department of an organisation
for the past six years. He was born on 07th November 1977. He has secured
60% marks in post Graduate Degree in Personnel Management. He has also
secured 55% marks in both Graduation and selection process.
14. Mohan Bajpai was born on 10th April 1975. He has secured 55% marks in
Graduation and 65% marks in Post Graduate Diploma in Personnel
Management. He has been working in the HR Department of an organisation
for the past six years after completing his Post Graduate Diploma.
15. Gopal Sharma has been working for the past five years in the HR Department of
an organisation after completing his Post Graduate Diploma in HR with 62%
marks. He has secured 50% marks in both Graduate and selection process. He
was born on 29th May 1974.
16. Arun Vohra has secured 55% marks in Graduation. He has been working in the
Personnel Department of an organisation for the past eleven years after
completing his post Graduate Degree in Personnel Management with 65%
marks. He has secured 50% marks in the selection process. He was born on 12th
August 1972.
17. Asha Dhar has secured 52% marks in Graduation and 62% marks in Post
Graduate Degree in Personnel Management. She has also secured 48% marks in
the selection process. She has been working for the past seven years in the
Personnel Department of an organisation after completing her Post Graduate
Degree. She was born on 08th June 1974.
18. Sudha Ghosal was born on 20th October 1976. She has been working as Deputy
Personnel Manager for the past four years in an organisation after completing
her Post Graduate Degree in HR with 67% marks. She has secured 60% marks
in Graduation and 45% marks in the selection process.
19. Amit Saxena was born on 25th July 1973. He has been working in the Personnel
Department of an organisation for the past eleven years after completing his
Post Graduate Diploma in HR with 70% marks. He has secured 60% ma rks in
both Graduation and selection process.
20. Navin Das was born on 14th April 1978. He has been working in the Personnel
Department of an organisation for the past six years after completing his Post
167

REASONING MADE EASY www.BankExamsToday.com


Graduate Diploma in HR with 65% marks. He has secured 45% marks in both
Graduation and selection process.
Directions (Q. 21-25) Study the following information carefully and answer the
questions given below
Following are the conditions for selecting Systems Manager in an organization:
The candidate must
(i)
Be a graduate engineer in IT, Computer Science, Electronics with at
least 60% marks
(ii)
Be at least 30 years and not more than 40 years as on 01.09.2009
(iii)
Have secured at least 40% marks in the written examination
(iv)
Have secured at least 50% marks in the selection interview
(v)
Have post qualification work experience of at least 10 years in the
Systems Department of an organization.
In the case of a candidate who satisfies all the conditions except
(a) At (i) above, but secure at least 60% marks in MEIT or Computer Science,
the case as to be referred to DGM-Systems.
(b) At (v) above, but has post qualification experince of at least five years as
Deputy Systems Manager the case is to be referred to the GM-Systems.
In each question below, details of one candidate are given. You have to take one of
the following courses of actions based on the information provided and the
conditions and sub-conditions given above and mark the number of that course of
action as your answer. You are not to assume anything else other than the
information provided for each candidate. All these cases are given to you as on
01.09.2009.
Give answer:
(a) If the candidate is to be selected
(b) If the candidate is not to be selected
(c) If the case is to be referred to DGM-Systems
(d) If the case is to be referred to GM-Systems
(e) If the data provided are not adequate to take a decision.
21. Samir Ghosh was born on 25th May 1978. He has secured 65% marks in BE-IT
in the year 1999. Since then he is working in the systems department of an
organization. He has secured 50% marks in both written examination and
selection interview.
22. Navin Prakash has secured 62% marks in 65% marks in BE-Computer Science.
He has been working in the systems department of an organization since July
1999 after completion of BE. He was born on 4 April 1974. He has secured
55% marks in selection interview and 45% marks in the written examination.
168

REASONING MADE EASY www.BankExamsToday.com

23. Neeta Pathak has been working as Deputy Systems Manager in an organization
for the past 7 years after completing her B.E. in IT with 70% marks. She has
secured 45% marks in selection interview and 55% marks in the written
examination. She was born on 12th November 1978.
24. Ashok Malhotra was born on 19th March 1977. He has secured 56% marks I n
both selection interview and written examination. He has secured 58% marks in
BE-IT and 72% marks in ME-IT. He has been working in the systems
department of an organization for the past 11 years after copleting ME-IT.
25. Geema DSouza was born on 15th December 1972. She has secured 60% marks
in both written examination and selection interview. She has been working as
Deputy Systems Manager for the last 6 years in an organization after
completing her BE-Electronics with 75% marks.
Directions (Q. 26-35) Study the following information carefully and answer the
questions given below.
Following are the conditions for selecting Accounts Manager in an organization.
The candidate must
(i)
Be a graduate in Commerce with minimum 55% marks
(ii)
Be a post graduate in Commerce with minimum 50 % marks
(iii)
Have post qualification work experience of at least three years in the
Accounts department in an organization.
(iv)
Not be less than 25 years and not more than 35 years old as on
01.05.2008.
(v)
Have secured at least 40% marks in the selection process.
In the case if candidate satisfies all other criteria exceptA. At (ii) above but h as work experience of at least four years as Assistant
Accounts Manager in an organization. His/her case is to be referred to GMAccounts of the organization.
B. At (iii) above but has successfully completed CA/ICWA, the case is to be
referred to Director Finance.
In each question below details of one candidate is given. You have to take one
of the following courses of action in each case based on the information
provided and the conditions and sub-conditions given above and mark the
number of that course of action as your answer. You are not to assume
anything other the information provided in each case. All t he cases are given
to you as on 01.05.2008.
Give answer:
a) If the data provided are not adequate to take a decision.
169

REASONING MADE EASY www.BankExamsToday.com


b) If the case is t o be referred to GM- Accounts.
c) If the case is to be referred to Director-Finance.
d) If the candidate is not to be selected.
e) If the candidate is to be selected.
26. Abhinav Chaturvedi is a Commerce graduate with 60% marks. He has been
working in the Accounts Department of an organization for the past four years
after completing his Post Graduation in Commerce with 55% marks. He was
born on 08th July 1980. He has secured 45% marks in the selection process.
27. Vaibhav Sinha was born on 12th April 1979. He has been working as Assistant
Accounts Manager in an organization for the past five years after completing
his post Graduation in Commerce with 40% marks. He has secured 55% marks
in both the selection process and Graduation in Commerce.
28. Seema Bhasin has secured 60% marks in Graduation in Commerce and 55%
marks in post Graduation in Commerce. She has secured 55% marks in the
selection process. She was born on 20th August 1978. She has also successfully
completed her CA. She does not have any work experience.
29. Nirmala Sawant was born on 04th July 1981. She has been working in the
Accounts department in an organization after completing her post graduation
Commerce with 65% marks. She has secured 50% marks in the selection
process.
30. Ashok Pradhan was born on 03rd May 1979. He has secured 42% marks in the
selection process. He has been working in the Accounts Department of an
organization for the past four years after completing his post Graduation in
Commerce with 62% marks. He has secured 54% marks in his Graduation in
Commerce.
31. Prabir Mazumdar has secured 58% marks in his Graduation in Commerce and
53% marks in post Graduation in Commerce. He has been working as Assistant
Accounts Manager in an organization for the past four years after completing
his post Graduation. He has secured 44% marks in the selection process. He
was born on 14th November 1977.
32. Neha Dev was born on 08th February 1976. She has been working in the
Accounts Department of an organization for the past three years after
completing her post Graduation in Commerce with 53% marks. She has secured
57% marks in Graduation in Commerce. She appeared in selection process.
33. Sudha Goswami was born on 19th October 1982. She has been working as
Assistant Accounts Manager in an organization for the past five years after
170

REASONING MADE EASY www.BankExamsToday.com


completing her Graduation in Commerce with 60% marks. She has secured
50% marks in the selection process.
34. Francis Dcosta has secured 60% marks in both graduation and Post Graduation
in Commerce. He has successfully completed ICWA after his Graduation. He
has been working in the Account Department of an organization for last one
year.
35. Prathama Sengupta has secured 55% marks in Post Graduation in Commerce
and 45% marks in the selection process. She was born on 11th April 1981. She
has been working in the Accounts Department of an organization for the past
four years after completing her Post Graduation. She has secured 50% marks in
Graduation in Commerce.

Answers:
1.
2.
3.
4.
5.
6.

Option C
Option B
Option B
Option D
Option E
Option C
Suresh Mehtas condition (iii) is not given so data is insufficient to take
decision.

7.

Option D
Sudha Gopalan fulfils condition (b) instead of condition (ii) so, her case is
to be referred to Vice President-Marketing.

8.

Option A
Divya Kohli does not fulfil condition (i) so, she is not to be selected.

9.

Option B
Navin Marathe fulfils all conditions so, he is to be selected.

10.

Option E
Varun Malhotra fulfils condition (a) instead of (iv) so, his case is to be
referred to GM-Marketing.

11.

Option E
Meena Srivastava fulfils conditions (b) instead of condition (iv) so, her case
is to be referred to the President-Personnel.

12.

Option B
Ketan Desai does not fulfil condition (v) so, he is not to be selected.
171

REASONING MADE EASY www.BankExamsToday.com

13.

Option C
Post qualification work of Anant Joshi is not cleared (which is least 5
years) so, data is insufficient.

14.

Option C
Marks of selection process (i.e. 45%) is not cleared so, data is Insufficient.

15.

Option B
Gopal Sharma is overaged.

16.

Option D
Arun Vohra fulfils condition (a) instead of (iii) so, his case is to be referred
to the Director-Personnel.

17.

Option A
Asha Dhar fulfils all conditions so, she is to be selected.

18.

Option E
Sudha Ghosal fulfils condition (b) instead of (iv) so, her case is to be
referred to President-Personnel.

19.

Option D
Amit Saxena fulfils condition (a) instead of (iii) so, his case is to be referred
to the Director-Personnel.

20.

Option B
Navin Das does not fulfil condition (i) so, he is not to be selected.

21.

Option E
Sameers working experience is not cleared in years. So, data is
insufficient.

22.

Naveen Prakash fulfils all conditions so, he is to be selected.

23.

Option B
Neeta Pathak does not fulfil condition (iv) so, she is not to be selected.

24.

Optoin C
Ashok Malhotra fulfils condition (a) so, his case is to be referred to DGMsystems.

25.

Option D
Geema DSouza fulfils condition (b) so, her case is to be referred to GMsystems.
172

REASONING MADE EASY www.BankExamsToday.com


26.
27.
28.
29.
30.
31.
32.
33.
34.
35.

Option E
Option B
Option C
Option A
Option D
Option E
Option D
Option B
Option C
Option D

173

REASONING MADE EASY www.BankExamsToday.com

Chapter 11
Direction Sense
1.

P walked 20 m towards North, took a left turn and walked 10 m, then took a
right turn and walked 20 m, again took a right turn and walked 10 m. How
far is he from his starting point?
a) 50 m
b) 60 m
c) 40 m
d) Cant be determined
e) None of these

2.

Kunal walked 10 m, towards the East, turned right and walked another 15
m. He then turns left and walks 3 m. He finally takes a left turn after
walking 6 m. Which direction is he facing now?
a) East
b) North
c) West
d) South
e) South-West

3.

Town D is to the West of town M. Town R is to the South of town D. Town


K is to the East of town R. Town K is towards which direction of town D?
a) South
b) East
c) North-East
d) South-East
e) None of these

4.

Ram walks 12 kms to the North, then 10 kms. To West, 12 kms. to South.
How far is Ram from the starting point?
a) 9 kms.
b) 13 kms.
c) 8 kms.
d) 10 kms.
e) None of these

5.

One morning Udai and Vishal were talking to each other face to face at a
crossing. If Vishals shadow was exactly to the left of Udai, which direction
was Udai facing?
a) East
b) West
c) North
d) South
e) None of these

6.

Two cars start from the opposite places of a main road, 150 km apart. First
car runs for 25 km and takes a right turn and then runs 15 km. It then turns
left and then runs for another 25 km and then takes the direction back to
reach the main road. In the mean time, due to minor break down the other
car has run only 35 km along the main road. What would be the distance
between two cars at this point?
a) 65 km
b) 75 km
c) 80 km
d) 85 km
e) None of these

7.

The length and breadth of a room are 8 m and 6 m respectively. A cat runs
along all the four walls and finally along a diagonal order to catch a rat.
How much total distance is covered by the cat?
a) 10
b) 14
c) 38
174

REASONING MADE EASY www.BankExamsToday.com


d) 48

e) None of these

8.

X started to walk straight towards south. After walking 5 m he turned to the


left and walked 3 m. After this he turned to right and walked 5 m now to
which direction X is facing?
a) North-East
b) South
c) North
d) South-West
e) None of these

9.

If A B means A is to the South of B, A + B means is to the North of B, A


% B means A is to the east of B, A B means A is to the West of B, then in
P % Q + R S, S is in which direction with respect to Q?
a) South-West
b) South-East
c) North-East
d) North-West
e) None of these

10.

One morning after sunrise, Suresh was standing facing a pole. The shadow
of the pole fell exactly to his right. To which direction was he facing?
a) East
b) South
c) West
d) Data is inadequate e) None of these

11.

Rohit walked 25 m towards South. Then he turned to his left and walked 20
m. He then turned to his left and walked 25 m. He again turned to his right
and walked 15 m. At what distance is he from the starting point and in
which direction?
a) 35 m East
b) 35 m North
c) 30 m West
d) 45 m East
e) None of these

12.

Sachin walks 20 km towards North. He turns left and walks 40 km. He


again turns left and walks 20 km. Finally he moves 20 km after turning to
the left. How far is he from his starting position?
a) 20 km
b) 30 km
c) 50 km
d) 60 km
e) None of these

Dirrections (Q. 13-16) Each of the following is based on the following


information:
(i) Six flats on a floor in two rows facing North and South are allotted to P, Q,
R, S, T and U.
(ii) Q gets a North facing flat and is not next to S.
(iii) S and U get diagonally opposite flats.
(iv) R next to U gets a South facing flat and T gets North facing flat.
13.

If the flats of P and T are interchanged then whose flat will be next to that
of U?
a) P
b) Q
c) R
d) T
e) None of these
175

REASONING MADE EASY www.BankExamsToday.com


14.

Which of the following combination get South facing flats?


a) QTS
b) UPT
c) URP
d) Data is inadequate e) None of these

15.

The flats of which of the other pair than SU, is diagonally opposite to each
other?
a) QP
b) QR
c) PT
d) TS
e) None of these

16.

Whose flat is between Q and S?


a) T
b) U
d) P
e) None of these

c) R

17.

Starting from a point, Raju walked 12 m North, he turned right and walked
10 km, he again turned right and walked 12 m, then he turned left and
walked 5 m. How far is he now and in which direction from the starting
point?
a) 27 m towards East
b) 5 m towards East
c) 10 m towards West
d) 15 m towards East
e) None of these

18.

Ravi travelled 4 km straight towards South. He turned left and travelled 6


km straight, then turned right and travelled 4 km straiht. How far is he from
the starting point?
a) 8 km
b) 10 km
c) 12 km
d) 18 km
e) None of these

19.

From point P, Akshay starts walking towards East. After walking 30 m, he


turns to his right and walks 10 m. He then turns to his right and walks for
30 m. He again turns to his right and walks 30 m. How far is he from point
P and in which direction?
a) Point P itself
b) 10 m, North
c) 20 m,
West
d) 20 m, North
e) None of these

20.

Facing towards South, Ram started walking and turned left after walking 30
m, he walked 25 m and turned left and walked 30 m. How far is he from his
starting position and in which direction?
a) At the starting point only
b) 25 m, West
c) 25 m, East
d) 30 m, East
e) None of these
176

REASONING MADE EASY www.BankExamsToday.com


21.

The town of Paranda is located on Gree lake. The town of Akram is West of
Paranda. Tokhada is East of Akram but West of Paranda. Kokran is East of
Bopri but West of Tokhada and Akram. If they are all in the same district,
which town is the farthest West?
a) Paranda
b) Kokran
c) Akram
d) Tokhada
e) Bopri

22.

B is to the South-West of A, C is to the East of B and South-East of A and


D is to the North of C in line with B and A. In which direction of A is D
located?
a) North
b) East
c) SouthEast
d) North-East
e) None of these

23.

A man is facing West. He turns 45 in the clockwise direction and then


another 180 in the same direction and then 270 in the anticlockwise
direction. Find which direction he is facing now?
a) South-West
b) West
c) South
d) East-South
e) None of these

24.

A dog runs 20 metre towards East and turns Right, runs 10 metre and turns
to right, runs 9 metre and again turns to left, runs 5 metre and then turns to
left, runs 12 metre and finally turns to left and runs 6 metre. Now which
direction dog is facing?
a) East
b) North
c) West
d) South
e) None of these

25.

Rahul put his timepiece on the table in such a way that at 06:00 p.m. hour
hand points to North. In which direction the minute hand will point at 09:15
p.m.?
a) South-East
b) South
c) North
d) West
e) None of these

26.

K is 40 m South-West of L. If M is 40 m South-East of L, then M is in


which direction of K?
a) East
b) West
c) NorthEast
d) South
e) None of these

27.

A child went 90 m in the East to look for his father, then he turned right and
went 20 m. After this he turned right and after going 30 m he reached to his
uncles house. His father was not there. From there he went 100 m to his
North and met his father. How far did he meet his father from the starting
point?
a) 80 m
b) 100 m
c) 140 m
d) 260 m
e) None of these
177

REASONING MADE EASY www.BankExamsToday.com

28.

Ravi wants to go to the University. He starts from his home which is in the
East and comes to a crossing. The road to the left ends in a theatre, straight
ahead is the hospital. In which direction is the University?
a) North
b) South
c) East
d) West
e) None of these

29.

A river flows West to East and on the way turns left and goes in a semicircle round a hillock, and then turns left at right angles. In which direction
in the river finally flowing?
a) West
b) East
c) North
d) South
e) None of these

30.

You go North, turn right, then right again and then go the left. In which
direction are you now?
a) North
b) South
c) East
d) West
e) None of these

Answers:
1.
2.
3.
4.
5.
6.

7.

Option C
Option B
Option D
Option D
Option C
Option A
Required distance

Option C
Required distance

= DF
= 150 (25 + 25 + 35)
= 150 85 = 65 km
= 8 + 6 + 8 + 6 + 8 + 6
= 28 + 100
= 28 + 10 = 38 m

8.

Option B
X will face in the end towards South.

9.

Option B
S is in the South-East of Q.

10.

Option B
Sun rises in the East in the morning. Since the shadow of Suresh falls to his
right. So, he is facing South.
178

REASONING MADE EASY www.BankExamsToday.com

11.

12.

13.

Option A
Required distance

= AE
= 20 + 15 = 35 m towards East

Option A
Required distance = 40 20 = 20 km
Option C
Flat R will be next to U

14.

Option C
URP flat combination get South facing flats.

15.

Option A
QP is diagonally opposite to each other.

16.

Flat T is between Q and S.

17.

Option D

18.

Option B

19.

Option D

20.

Option C

21.

Option E

22.

Option D

23.

Option A
The man firstly faces the direction OA. On moving 45 clockwise, he faces
the direction OB.
Now, again he moved 180 clockwise, now, he will be facing OC. From
here he moved 270 anticlockwise, Finally he is facing OD, which is SouthWest.

24.

Option B

25.

Option D

179

REASONING MADE EASY www.BankExamsToday.com

At 09:15 p.m., the minute hand will point towards West.


26.

Option A

M is in the East of K.
27.

Option B

28.

Option A
180

REASONING MADE EASY www.BankExamsToday.com


Starting from his house in the East, Ravi moves westwards. Then the
theatre, which is to the left, will be in the South. The hospital, which is
straight ahead, will be to the West. So, the University will be to the North.

29.

Option B

30.

Option C

181

REASONING MADE EASY www.BankExamsToday.com

Chapter 12
Letter and Number Series
1.

If in the word ISOLATE, all the consonants are replaced by the previous
letter in the alphabet and all the vowels are replaced by the next letter then
all the letters are arranged alphabetically, which letter will be third from the
right end?
a) P
b) B
c) N
d) Q
e) None of these

2.

If in the word EQUALITY, the positions of first and the fifth letters are
interchanged; similarly the positions of the second and the sixth letters are
interchanged and so on. Which letter will be third from the right end?
a) Q
b) U
c) I
d) T
e) None of these

3.

How many meaningful English words can be made from the letters AERT,
using each letter only once in each word?
a) None
b) One
c) Two
d) Three
e) Four

4.

How many such pairs of letters are there in the word REFRESHING each
of which has as many letters between them in the word as they have in the
English alphabet?
a) None
b) One
c) Two
d) Three
e) More than three

5.

If in the number 38564927 first all the even digits are arranged in ascending
order and then all the odd digits are arranged in ascending order, which
digit will be fourth from the right end?
a) 5
b) 3
c) 6
d) 4
e) None of these

6.

How many such pairs of letters are there in the word BOARDING each of
which has as many letters between them in the word as in the English
alphabet?
a) None
b) One
c) Two
d) Three
e) More than three

7.

How many such digits are there in the number 284371 each of which is as
far away from the beginning of the number as when they are arranged in
descending order?
a) None
b) One
c) Two
d) Three
e) More than three
182

REASONING MADE EASY www.BankExamsToday.com

8.

How many meaningful English words can be made with the letters only
once in each word?
a) None
b) One
c) Two
d) Three
e) More than three

9.

How many meaningful English words can be made with the letters ONDE
using each letter only once in each word?
a) None
b) One
c) Two
d) Three
e) More than three

10.

How many such pairs of letters are there in the word JUMPING each of
which has as many letters between them in the word as in the English
alphabet?
a) None
b) One
c) Two
d) Three
e) More than three

11.

How many such digits are there in the number 6837941 each of which is as
far away from the beginning in the number as when the digits are arranged
in descending order within the number?
a) None
b) One
c) Two
d) Three
e) More than three

12.

What should come next in the following letter series?


PPOPONPONMPONMLPONML
a) P
b) K
c) J
d) I
e) None of these

13.

How many such pairs of letters are there in the word STAPLER each of
which has as m any letters between them in the word as in the English
alphabet?
a) None
b) One
c) Two
d) Three
e) More than three

14.

How many such pairs of letters are there in the word OBJECTIVE each of
which has as many letters between them in the word as in the English
alphabet?
a) None
b) One
c) Two
d) Three
e) More than three

15.

If each consonant in the word TOLERANT is replaced by the previous


letter in the English alphabet and each vowel in the word is replaced by the
next letter in the English alphabet and the new set of letters are arranged
alphabetically, which of the following will be the 4th from the right end
after the replacement?
a) M
b) P
c) Q
183

REASONING MADE EASY www.BankExamsToday.com


d) K

e) None of these

16.

How many such pairs of digits are there in the number 8429516 each of
which has as many digits between them in the number as when the digits
are arranged in ascending order within the number?
a) None
b) One
c) Two
d) Three
e) More than three

17.

The positions of the first and the second digits in the number 85341279 are
interchanged. Similarly the positions of 3rd and 4th digits are interchanged
and so on till the position of 7th and 8th digits. Which of the following will
be 3rd to the right of three after the rearrangement?
a) 9
b) 7
c) 8
d) 2
e) None of these

18.

How many meaningful English words can be formed with the letters MRTA
using each letter only once in each word?
a) None
b) One
c) Two
d) Three
e) More than three

19.

The positions of the first and the fifth digit in the number 53216894 are
interchanged. Similarly, the position of the second and the sixth digit are in
terchanged and so on. Which of the following will be the second from t he
right end after the rearrangement?
a) 3
b) 2
c) 1
d) 9
e) None of these

20.

The positions of how many digits in the number 53147926 will remain
unchanged after the digits within the number are rearranged in descending
order?
a) None
b) One
c) Two
d) Three
e) More than three

21.

How many such pairs of letters are there in the word HOARDINGS each of
which has as many letters between them in the word as in the English
alphabet?
a) None
b) One
c) Two
d) Three
e) More than three

22.

How many such pairs of letters are there in the word DISCREDIT each of
which has as many letters between them in the word as in the English
alphabet?
a) None
b) One
c) Two
d) Three
e) More than three

184

REASONING MADE EASY www.BankExamsToday.com


23.

If it is possible to make only one meaningful English word with 3rd, 4th, 6th
and 8th letters of the word SINGLETON, which of the following will be 3rd
letter of that word? If no such word can be made, g ive X as the answer
and I f more than one such word can be made, give Y as the answer.
a) N
b) O
c) E
d) X
e) Y

24.

What should come next in the following number series?


987654321123456789987654322345678
a) 8
b) 1
c) 9
d) 2
e) None of these

25.

How many such digits are there in the number 764528 each of which is as
far away from the beginning of the number as when the digits are arranged
in descending order within the number?
a) None
b) One
c) Two
d) Three
e) More than three

26.

What should come next in the letter series given below?


ABABCABCDABCDEABCDEFABCDEFGABC
a) D
b) E
c) F
d) H
e) None of these

27.

How many meaningful English words can be made with the letters EPRY
using each letter only once in each word?
a) None
b) One
c) Two
d) Three
e) More than three

Directions (Q. 28-34) In each of the following questions, a series is given with
one term missing. Choose the correct alternative from the given ones that will
complete the series.
28.

29.

30.

31.

AD, EI, JN, PS, ?


a) WY
d) WW

b) XX
e) None of these

c) WX

PON, RQP, TSR, VUT, ?


a) WUY
d) UVW

b) YXZ
c) XWV
e) None of these

ar, cs, et, ?


a) wy
d) gu

b) gv
e) None of these

c) vb

9, 15, 23, 33, ?


a) 44

b) 36

c) 38
185

REASONING MADE EASY www.BankExamsToday.com

32.

33.

34.

d) 45

e) None of these

4, 7, 14, 24, 41, ?


a) 71
d) 51

b) 68
e) None of these

5, 16, 51, 158, ?


a) 481
d) 478

c) 72

b) 465
c) 441
e) None of these

CBA, ABC, ABCD, DCBA, ABCDE, ?


a) EDCBA
b) DBAC
d) BACD
e) None of these

c) CABD

35.

How many meaningful English words can be formed with the letters LTSO
using each letter only once in each word?
a) None
b) One
c) Two
d) Three
e) More than three

36.

How many such digits are there in the number 928416375 each of which is
as far away from th e beginning in the number as when the digits are
arranged in descending order within the number?
a) None
b) One
c) Two
d) Three
e) More than three

37.

What should come next in the following letter series?


Cbaacbaabcbaabccbaabcdcbaa
a) a
b) b
c) c
d) d
e) None of these

Directions (Q. 38-43) Study the number series given below and answer the
questions which follow.
6

78998797789787696897798976887

38.

How many such numbers are there in the given series each of which when
subtracted from the following number has a difference of 2?
a) Three
b) Four
c) Five
d) Nine
e) None of these

39.

Which of the following nu mbers is obtained when 18th number from the
left of the number series is added to 19th from the right?
a) 15
b) 20
c) 10
d) 17
e) None of these

186

REASONING MADE EASY www.BankExamsToday.com


40.

How many nines are there in the given series each of which is immediately
preceded by an odd number?
a) One
b) Two
c) Three
d) Four
e) More than four

41.

How many such even numbers are there in the given series each of which is
immediately followed by an even number?
a) None
b) One
c) Two
d) Three
e) More than three

42.

How many such odd numbers are there in the given series each of which is
immediately preceded by an even number?
a) Five
b) Seven
c) Nine
d) Eleven
e) None of these

43.

How many such sevens are there in the given series each of which is
immediately preceded by 9 and also immediately followed by 8?
a) None
b) One
c) Two
d) Three
e) More than three

Answers:
1.
2.
3.
4.
5.
6.
7.
8.
9.
10.

11.

12.

Option A
Option A
Option C
Option D
Option B
OptionC
Option D
Option C
Option C
DONE and NODE
Option C
J
U
M
10
21
13

P
16

I
9

N
14

G
7

Option C
Number
6
8
3
7
Decreasing Order
9
8
7
6
There will be no change in the place of 8 and 1.
Option B
Series are increasing as follows:
P
PO
PON
PONM PONML

9
4

4
3

1
1

PONMLK
187

REASONING MADE EASY www.BankExamsToday.com


13.

14.

15.

16.

17.

18.
19.

20.

21.
22.

23.

Option C
S
T
A
S T and S P

Option C
O
B
J
E
Two pairs formed IO, TV
Option B
T
O
L
E
According to question,
S
P
K
F
Alphabetically,
B
F
K
M
th
So, P is 4 from the right.
Option C
8
4
2
In increasing order
1
2
4
Option A
Number:
9
Arrangement: 5

Option C
The meaningful words are: MART, TRAM
Option B
Number:
5
3
2
1
After interchanging the digits
6
8
9
4
5
3
Option B
Number:
2
6
In decreasing order:

Option E
Option E
D
I
S
C
R
E
So, CE, DI, DE, EI are pairs of letters.
Option A

1
S

7
T

8
O

9
N

2
I

3
N

4
G

5
L

6
E

188

REASONING MADE EASY www.BankExamsToday.com

24.
25.

Meaningful word from NGEO is GONE.


3rd letter in the word GONE is N
Option C
987654321, 123456789, 98765432, 2345678
Option B
7
8

26.

6
7

4
6

5
5

2
4

8
2

Option A
AB, ABC, ADCD, ABCDE, ABCDEF, ABCDEFG, ABC D

27.

28.
29.
30.
31.

Option C
Meaningful words PREY, PYRE
Option C
Option C
Option D
Option D
9 + 6 = 15
15 + 8 = 23
23 + 10 = 33
33 + 12 = 45

32.

Option B
4+3=7
7 + 4 + 3 = 14
14 + 3 + 7 = 24
24 + 7 + 10 = 41
41 + 10 + 17 = 68

33.

Option A
5 3 + 1 = 16
16 3 + 3 = 51
51 3 + 5 = 158
158 3 + 7 = 481
Option A
CBA ABC
Reverse order
ABCD DCBA
ABCDE EDCBA
Option D
Option C
Option B
Option E

34.

35.
36.
37.
38.

189

REASONING MADE EASY www.BankExamsToday.com


39.
40.
41.
42.
43.

Option A
Option C
Option D
Option C
Option B

190

REASONING MADE EASY www.BankExamsToday.com

Chapter 13
Logical Arrangement
1.

Which one of the given responses would be a meaningful order of the


following?
(i) Earth
(ii) Jupiter
(iii) Venus
(iv) Mars
(v) Mercury
a) v, iii, i, ii, iv
b) v, iii, iv, i, ii
c) v, iii, i, iv, ii
d) v, iii, ii, iv, i
e) None of these

Arrange the words given below in a meaningful sequence.


2.

(i) Key (ii) Door (iii) Lock (iv) Room (v) Switch on
a) v, i, ii, iv, iii
b) iv, ii, i, v, iii
d) i, iii, ii, iv, v
e) None of these

c) i, ii, iii, v, iv

3.

(i) Windows (ii) Walls (iii) Floor (iv) Foundation (v) Roof (vi) Room
a) iv, v, iii, ii, i, vi
b) iv, iii, v, vi, ii, i
c) iv, ii, i, v, iii, vi
d) iv, i, v, vi, ii, iii
e) None of these

4.

(i) District (ii) Village (iii) State (iv) Block


a) ii, i, iv, iii
b) ii, iii, iv, i
d) iii, ii, i, iv
e) None of these

c) ii, iv, i, iii

(i) Site (ii) Plan (iii) Rent (iv) Money (v) Building
a) iv, i, ii, v, iii
b) iii, iv, ii, v, i
d) i, ii, iii, v, iv
e) None of these

c) ii, iii, v, i, iv

(i) Cut (ii) Put on (iii) Mark (iv) Measure (v) Tailor
a) iv, iii, i, v, ii
b) iii, i, v, iv, ii
d) i, iii, ii, iv, v
e) None of these

c) ii, iv, iii, i, v

(i) Mother (ii) Child (iii) Milk (iv) Cry (v) Smile
a) i, v, ii, iv, iii
b) ii, iv, i, iii, v
d) iii, ii, i, v, iv
e) None of these

c) ii, iv, iii, i, v

5.

6.

7.

8.

(i) Word (ii) Paragraph (iii) Sentence (iv) Letters (v) Phrase
a) iv, i, v, ii, iii
b) iv, i, iii, v, ii
c) iv, ii, v, i, iii
191

REASONING MADE EASY www.BankExamsToday.com


d) iv, i, v, iii, ii

e) None of these

9.

(i) Caste (ii) Family (iii) Newly married couple (iv) Clan (v) species
a) ii, iii, i, iv, v
b) iii, iv, v, i, ii
c) iii, ii, i, iv, v
d) iv, v, iii, ii, i
e) None of these

10.

(i) Elephant (ii) Cat (iii) Mosquito (iv) Tiger (v) Whale
a) v, iii, i, ii, iv
b) iii, ii, iv, i, v
c) i, iii, v, iv, ii
d) ii, v, i, iv, iii
e) None of these

11.

(i) Protect (ii) Pressure (iii) Relief (iv) Rain (v) Flood
a) ii, iv, iii, i, v
b) ii, iv, v, i, iii
d) iii, ii, iv, v, i
e) None of these

c) ii, v, iv, i, iii

(i) Rainbow (ii) Rain (iii) Sun (iv) Happy (v) Child
a) iv, ii, iii, v, i
b) ii, iii, i, v, iv
d) ii, i, iv, v, iii
e) None of these

c) iv, v, i, ii, iii

12.

13.

(i) Patients (ii) Diagnosis (iii) Bill (iv) Doctor (v) Treatment
a) i, iv, iii, ii, v
b) i, iv, ii, v, iii
c) i, iv, ii, iii, v
d) i, ii, iii, iv, v
e) None of these

14.

(i) Table (ii) Tree (iii) Wood (iv) Seed (v) Plant
a) i, ii, iii, iv, v
b) iv, v, iii, ii, i
d) iv, v, ii, iii, i
e) None of these

c) I, iii, ii, iv, v

15.

(i) Doctor (ii) Fever (iii) Prescribe (iv) Diagnose (v) Medicine
a) ii, iv, iii, v, i
b) i, iv, iii, ii, v
c) ii, i, iii, iv, v
d) ii, i, iv, iii, v
e) None of these

16.

(i) Index (ii) Contents (iii) Title (iv) Chapters (v) Introduction
a) iii, ii, v, i, iv
b) ii, iii, iv, v, i
c) iii, ii, v, iv, i
d) v, I, iv, ii, iii
e) None of these

17.

(i) Book (ii) Pulp (iii) Timber (iv) Jungle (v) Pape
a) iii, ii, v, i, iv
b) iv, iii, ii, v, i
d) v, iv, iii, i, ii
e) None of these

c) ii, v, i, iv, iii

18.

(i) College (ii) Child (iii) Salary (iv) School (v) Employment
a) iv, i, iii, v, ii
b) i, ii, iv, iii, v
c) ii, iv, i, v, iii
d) v, iii, ii, i, iv
e) None of these

19.

(i) Study (ii) Job (iii) Examination (iv) Earn (v) Apply
a) i, iii, v, ii, iv
b) i, iii, ii, v, iv
c) i, ii, iii, iv, v
d) i, iii, v, iv, ii
e) None of these
192

REASONING MADE EASY www.BankExamsToday.com


20.

(i) Frog (ii) Eagle (iii) Grasshopper (iv) Snake (v) Grass
a) v, iii, i, iv, ii
b) i, iii, v, ii, iv
c) v, iii, iv, ii, i
d) iii, iv, ii, v, i
e) None of these

Answers:
1.
2.
3.
4.
5.
6.
7.
8.
9.
10.
11.
12.
13.
14.
15.
16.
17.
18.
19.
20.

Option B
Option D
Option C
Option C
Option D
Option A
Option C
Option D
Option C
Option B
Option B
Option B
Option B
Option D
Option D
Option C
Option B
Option C
Option A
Option A

193

REASONING MADE EASY www.BankExamsToday.com

Chapter 14
Mathematical Operations
1.

If Q means +, T means , R means and K means then


2 4 R 4 Q 8 K 6 T 10 = ?
a) 48
b) 24
c) 44
d) 2/3
e) None of these

2.

If denotes , denotes +, denotes and + denotes then


28 12 + 4 6 4 = ?
a) 4
b) 1
c) 6
d) 5
e) None of these

3.

If + means , means , means and means + then 42


24 + 6 4 3 = ?
a) 22-3/4
b) 50
c) 58
d) 26
e) None of these

4.

Which of the following interchange of signs would make the given equation
correct?
(6 + 3) + (4 7) = 29
a) + and
b) and +
c) and +
d) and
e) None of these

5.

Change the sign to find the equation 28 7 + 2 2 = 0


a) Change + into
b) Change into +
c) Change into +
d) Change + into
e) None of these

6.

9 5 4 3 2 = ?
a) 2
d) 3

7.

If + means , means , means and means +, then 8 + 6 4 3 4


=?
a) 12
b) 20/3
c) 12
d) 20/3
e) None of these

8.

If means , means , means + and + means , then 3 15 19 8 +


6=?
a) 8
b) 4
c) 2
d) 1
e) None of these

b) 9
e) None of these

c) 3

194

REASONING MADE EASY www.BankExamsToday.com

9.

When 1100010 is divided by 0101, what will be the decimal remainder?


a) 2
b) 3
c) 4
d) 6
e) None of these

10.

How many basic binary subtraction combinations are possible?


a) 4
b) 3
c) 2
d) 1
e) None of these

11.

When multiplying in binary the decimal values 13 11, what is the third
partial product?
a) 100000
b) 100001
c) 0000
d) 1011
e) None of these

12.

If stands for addition, stands for subtraction, + stands for


multiplication and stands for division, then 20 8 8 4 + 2 = ?
a) 80
b) 25
c) 24
d) 5
e) None of these

Answers:
1.
2.

3.

4.

5.

6.

Option C
Option E
28 12 + 4 6 4 = ?
? = 28 + 12 4 6 4
? = 28 + 12/4 6 4
? = 28 + 3 24
?= 7
Option B
? = 42 24 + 6 4 3
= 42 24 6 + 4 3
= 42 24/6 + 12
= 42 4 + 12 = 50
Option C
(6 + 3) + (4 7) = 29
(6 3) + (4 + 7) = 29
18 + 11 = 29
Option A
28 7 + 2 2 = 0
28 7 2 2 = 0
28 28 = 0

Option E
9 5 4 3 2 = 9 5 4 3/2 = 19/2

195

REASONING MADE EASY www.BankExamsToday.com

7.

Option B

8.

Option C

9.

Option B

10.

Option A

11.

Option A

12.

Option C
20 + 8 8 4 2 = 20 + 8 2 2 = 20 + 8 4 = 24

196

REASONING MADE EASY www.BankExamsToday.com

Chapter 15
Odd Man Out Series
1.

Four of the following five are alike in a certain way and so form a group.
Which is the one that does not belong to the group?
a) PS
b) FI
c) AD
d) KN
e) GD

2.

Four of the following five are alike in a certain way and so form a group.
Which is the one that does not belong to the group?
a) 23
b) 29
c) 37
d) 39
e) 31

3.

Four of the following five are alike in a certain way and so form a group.
Which is the one that does not belong to the group?
a) Yellow
b) Blue
c) Pink
d) Green
e) Red

4.

Four of the following are alike in a certain way and so form a group. Which
is the one that does not belong to that group?
a) Table
b) Chair
c) Bench
d) Desk
e) Wood

5.

Four of the following are alike in a certain way and so form a group. Which
is the one that d oes not belong to that group?
a) 27
b) 64
c) 125
d) 216
e) 384

6.

Four of the following five are alike in a certain way and so form a group.
Which is the one that does not belong to that group?
a) 35
b) 80
c) 45
d) 60
e) 75

7.

Four of the following five are alike in a certain way and so form a group.
Which is the o ne that does not belong to that group?
a) Sweet
b) Cake
c) Pastry
d) Bread
e) Biscuit

8.

Four of the following five are alike in a certain way and so form a group.
Which is the one that does not belong to that group?
a) 39
b) 27
c) 48
d) 42
e) 24

197

REASONING MADE EASY www.BankExamsToday.com


9.

Four of the following five are a like in a certain way and so form a group.
Which is the one that does not belong to that group?
a) VT
b) MK
c) DB
d) KH
e) XV

10.

Four of the following five are alike in a certain way and so form a group.
Which is the one that does not belong to that group?
a) 187
b) 323
c) 119
d) 221
e) 289

Directions (Q. 11-19) In each of the following questions, select the one which is
different from the other three responses.
11.

a) Aptitude
d) Behaviour

b) Altitude
e) None of these

c) Attitude

12.

a) SSA
d) NASA

b) RMSA
e) None of these

c) RUSA

13.

a) 63
d) 66

b) 69
e) None of these

c) 65

14.

a) 108
d) 225

b) 91
e) None of these

c) 144

15.

187 : 11
d) 224 : 14

b) 194 : 12
e) None of these

c) 195 : 13

16.

a) D
d) J

b) G
e) None of these

c) H

17.

a) SRT
d) VUW

b) PON
e) None of these

c) KJL

18.

a) EVFU
d) GTHS

b) CXDW
e) None of these

c) AZBX

19.

a) Square
b) Equilateral Triangle
c) Rhombus
d) Right Angle Triangle

20.

Four of the following five are alike in a cretain way and so form a group.
Which is the one that does not belong to that group?
a) 21
b) 35
c) 42
d) 56
e) 49
198

REASONING MADE EASY www.BankExamsToday.com

21.

Four of the following five are alike in a certain way and so form a group.
Which is the one that does not belong to that group?
a) Table
b) Desk
c) Wardrobe
d) Computer
e) Chair

22.

Four of the following five are alike in a certain way and so form a group.
Which is one that does not belong to that group?
a) Orange
b) Apple
c) Guava
d) Papaya
e) Mango

23.

Find the odd one out.


a) January, May
b) April, June
c) July, August
d) January, December
e) None of these

Answers:
1.
2.
3.
4.
5.
6.

Option E
Option D
Option C
Option E
Option E
Option C
Apart from 45, sum of both digits of every number will be an even number
but 45 = 4 + 5 = 9 which is an odd number.

7.

Option A

8.
9.
10.

11.

Except sweet all the other are baked items.


Option B
27 is the cube of 3
Option D
Option E
289 = 17 17
So, 289 is a perfect square.
Option B
Altitude means the height above sea l evel. Except altitude, all other words
imply human nature and mental status.

12.

Option C
199

REASONING MADE EASY www.BankExamsToday.com

13.

SSA ASS (An Animal)


RMSA MARS (A Planet)
NASA American Space Agency
Option D

Except 66, all others are odd numbers.


14.

Option B
The number 91 is a product of two prime numbers. 13 7 = 91

15.

Option B

16.

Except in the number pair 194-12, in all other number pairs the first number
is completely divisible by the second number.
187/11 = 17; 195/13 = 15; 224/14 = 16
But, 194/12 = 16.166
Option B

17.
18.
19.

20.
21.
22.
23.

D position number 4
H position number 8
J position number 10
G position number 7 (An odd number)

Option B
Option C
Option D
Except in the case of right angle triangle, in all other geometrical figures, all
the sides are equal.
Option E
The number of 49 is a perfect square of a natural number.
Option D
Computer is a mechanical device. All others are wooden items.
Option A
Orange is citrus fruit.
Optoin B
Except the months of April and June, all other months have 31 days each.

200

REASONING MADE EASY www.BankExamsToday.com


Chapter 16
Ranking Based Problems
1.

In a row of 35 children, M is 15th from the right end and there are 10
children between M and R. What is Rs position from the left end of the
row?
a) 15th
b) 5th
c) 30th
d) Data inadequate
e) None of the above

2.

Among B, F, J, K and W each having a different weight, F is heavier than


only J. B I s heavier than F and W but not as h eavy as K. Who among
them is the third heaviest among them?
a) B
b) F
c) K
d) W
e) None of these

3.

J, D, L, H and F each travelling to station, each one reaches at a different


time. L reaches only after J and D reaches only before F. Who amongst
them is 3rd to reach?
a) F
b) L
c) D
d) Cannot be determined
e) None of these

4.

In a queue, Amrita is 10th from the front while Mukul is 25th from behind
and Mamta is just in the middle of the two. If there be 50 persons in the
queue. What position does Mamta occupy from the front?
a) 20th
b) 19th
c) 18th
th
d) 17
e) None of these

5.

Some boys are sitting in a row. P is sitting 14th from the left and Q is 7th
from the right. If there are four boys between P and Q. How many boys are
there in the row?
a) 25
b) 23
c) 21
d) 19
e) None of these

6.

Standing on a platform, Amit told Sunita that Aligarh was more than 10
kilometers but less than 15 kilometers from there. Sunita knew that it was
more than 12 but less than 14 kilometers from there. If both of them were
correct, which of the following could be the distance of Aligarh from the
platform?
a) 11 km
b) 12 km
c) 13 km
d) 14 km
e) 15 km

7.

The train for Lucknow leaves every two and a half hours from New Delhi
Railway Station. An announcement was made at the station that the train for
Lucknow had left 40 minutes ago and the next train will leave at 18.00 hrs.
At what time was the announcement made?
201

REASONING MADE EASY www.BankExamsToday.com


a) 15.30 hrs.
hrs.
d) 15.50 hrs.

b) 17.10 hrs.

c)

16.00

e) None of these

8.

How many 6s are there in the following series of numbers which are
preceded by 7 but not immediately followed by 9?
6795697687678694677695763
a) One
b) Two
c) Three
d) Four
e) None of these

9.

How many 7s are there in the following sequence which are preceded by 9
and followed by 6?
7897653428972459297647
a) 2
b) 3
c) 4
d) 5
e) None of these

10.

How many 6s are there in the following number sequence which are
immediately preceded by 9 but not immediately followed by 4?
5 6 4 3 2 9 6 3 1 6 4 9 6 4 21 5 9 6 7 2 1 4 7 4 9 6 4 2
a) One
b) Two
c) Three
d) Four
e) More than four

11.

51473985726315863852243496
How many odd numbers are there in the sequence which are immediately
preceded and also immediately followed by an even number?
a) 1
b) 2
c) 3
d) 4
e) More than 4

12.

The letters L, M, N, O, P, Q, R, S and T in their order are substituted by


nine integers 1 to 9 but not in that order. 4 is assigned to P. The difference
between P and T is 5. The difference between N and T is 3. What is the
integer asigned to N?
a) 4
b) 5
c) 6
d) 7
e) None of these

Answers:
1.
2.

3.

4.

Option D
Option D
K>B>W>F>J
So, W is the third heaviest.
Option E
J >L> H>D>F
H is 3rd to reach.
Option C
202

REASONING MADE EASY www.BankExamsToday.com

5.

Option A

6.

Option C

7.

Option E

8.

Option C

9.

Option A

10.

Option B

11.

Option D

12.

Option C

203

REASONING MADE EASY www.BankExamsToday.com

Chapter 17
Arithmatic Reasoning
1. The total of the ages of Amar, Akbar and Anthony is 80 years. What was
the total of their ages three years ago?
a) 71 years
b) 72 years
c) 74 years
d) 77 years
e) None of these
2. A woman says, If you reverse my own age, the figures represent my
husbands age. He is, of course, senior to me and the difference between
our ages is one-eleventh of their sum. The womans age is
a) 23 years
b) 34 years
c) 45 years
d) Cant be determined e) None of these
3. In a garden, there are 10 rows and 12 coloumns of mango trees. The
distance between the two trees is 2 metres and a distance of one metre is left
from all sides of the boundary of the garden. The length of the garden is
a) 20 m
b) 22 m
c) 24 m
d) 26 m
e) None of these
4. A motorist knows four different routes from Bristol to Birmingham. From
Birmingham to Shefield he knows three different routes and from Sheffield
to Carlisle he knows two different routes. How many routes does he know
from Bristol to Carlisle?
a) 4
b) 8
c) 12
d) 24
e) None of these
5. If 100 cats kill 100 mice in 100 days, then 4 cats would kill 4 mice in how
many days?
a) 1 day
b) 4 days
c) 40 days
d) 100 days
e) None of these
6. Five bells begin to toll together and toll respectively at intervals of 6, 5, 7,
10 and 12 seconds. How many times will they toll togethe in one hour
excluding the one at the start?
a) 7 times
b) 8 times
c) 9 times
d) 11 times
e) None of these
7. Today is Varuns birthday. One year, from today he will be twice as old as
he was 12 years ago. How old is Varun today?
a) 20 years
b) 22 years
c) 25
years
d) 27 years
e) None of these
204

REASONING MADE EASY www.BankExamsToday.com


8. When Rahul was born, his father was 32 years older than his brother and his
mother was 25 years older than his sister. If Rahuls brother is 6 years
oldern than him and his mother is 3 years younger than his father, how old
was Rahuls sister when he was born?
a) 7 years
b) 10 years
c) 14
years
d) 19 years
e) None of these
9. A shepherd had 17 sheep. All but nine died. How many was he left with
a) Nil
b) 8
c) 9
d) 17
e) None of these
10. The total number of digits used in numbering the pages of a book having
366 pages is
a) 732
b) 990
c) 1098
d) 1305
e) None of these
11. A student got twice as many sums wrong as he got right. If he attempted 48
sums in all, how did he solve correctly?
a) 12
b) 16
c) 18
d) 24
e) None of these
12. A father tells his son, I was of your present age when you were born. If
the father is 36 now, how old was the boy five years back?
a) 13
b) 15
c) 17
d) 20
e) None of these
13. Find the number which when added to itself 13 times, gives 112
a) 7
b) 8
c) 9
d) 11
e) None of these
14. At the end of a business conference the ten people present all shake hands
with each other once. How many handshakes will there be altogether?
a) 20
b) 45
c) 55
d) 90
e) None of these
15. A group of 1200 persons consisting of captains and soldiers is travelling in
a train. For every 15 soldiers there is one captain. The number of captains in
the group is
a) 85
b) 80
c) 75
d) 70
e) None of these
16. A placed three sheets with two carbons to get two extra copies of the
original. Then he decided to get more carbon copies and folded the paper in
such a way that the upper half of the sheets were on top of the lower half.
Then he typed. How many carbon copies did he get?
205

REASONING MADE EASY www.BankExamsToday.com


a) 1
d) 4

b) 2
e) None of these

c) 3

17. Aruna cut a cake into two halves and cuts one half into smaller pieces of
equal size. Each of the small pieces is 20 grams in weight. If she has seven
pieces of the cake in all with her, how heavy was the original cake?
a) 240 gm
b) 220 gm
c) 225 gm
d) 250 gm
e) None of these
18. The number of boys in a class is three times the number of girls. Which one
of the following numbers cannot represent the total number of children in
the class?
a) 48
b) 44
c) 42
d) 40
e) None of these
19. In a group of 15 people, 7 read French, 8 read English while 3 of them read
none of these two. How many of them read French and English both?
a) 2
b) 3
c) 4
d) 5
e) None of these
20. A tailor had a number of shirt pieces to cut from a roll of fabric. He cut
each roll of equal length into 10 pieces. He cut at the rate of 45 cuts a
minute. How many rolls would be cut in 24 minutes?
a) 30 rolls
b) 58 rolls
c) 120
rolls
d) 150 rolls
e) None of these
21. There are deer and peacocks in a zoo. By counting heads they are 80. The
number of their legs is 200. How many peacocks are there?
a) 10
b) 20
c) 40
d) 60
e) None of these
22. A farmer built a fence around his square plot. He used 27 fence poles on
each side of the square. How many poles did he need altogether?
a) 54
b) 84
c) 104
d) 108
e) None of these
23. I have a few sweets to be distributed. If I keep 2, 3 or 4 in pack, I am left
with one sweet. If I keep 5 in a pack, I am left with none. What is the
minimum number of sweets I have to pack and distribute?
a) 25
b) 35
c) 56
d) 65
e) None of these
24. A man has Rs.480 in the denominations of one rupee notes, five rupee notes
and ten rupee notes. The number of each denomination is equal. What is the
total number of notes that he has?
206

REASONING MADE EASY www.BankExamsToday.com


a) 45
d) 120

b) 75
e) None of these

c) 90

25. A is 3 years older to B and 3 years younger to C, while B and D are twins.
How many years older is C and D?
a) 3
b) 6
c) 9
d) 12
e) None of these
Answers:
1. Required sum = (80 3 3) years = (80 9) years = 71 years

2. Let x and y be the tens and units digits respectively of the numeral
denoting the womans age
Then, womans age = (10x + y) years, husbands age = (10y + x) years
Therefore (10y + x) (10x + y) = (1/11) (10y + x + 10x + y)
(9y 9x) = (1/11) (11y + 11x) = (x + y)
10x = 8y
x = 4/5y
Clearly, y should be a single digit multiple of 5, which is 5.
Hence, womans age = 10x + y = 45 years
3. Each row contains 12 plants.
There are 11 gapes between two corner trees (11 2) metres and 1 metre on
each side is left.
Therefore length = (22 + 2)m = 24 m
4. Total number of routes from Bristol to Carlisle = (4 3 2) = 24
5. Less cats, more days
(indirect proportion)
Less mice, less days
(direct proportion)
Let the required number of days be x.
Cat
4
:
100
:: x : 100
Mice
100
:
4
100 4 x = 4 100 100 or x = 4 100 100 / 100 4 = 100
6. L.C.M. of 6, 5, 7, 10 and 12 is 420
So, the bells will toll together after every 420 seconds i.e. 7 minutes.
Now, 7 8 = 56 and 7 9 = 63
Thus, in 1 hour (or 60 minutes), the bells will toll together 8 times,
excluding the one at the start.
7. Let Varuns age today = x years
Then, Varuns age after 1 year = x + 1 years
Therefore, x + 1 = 2 (x 12)
x + 1 = 2x 24

207

REASONING MADE EASY www.BankExamsToday.com


x = 25
8. When Rahul was born, his brothers age = 6 years, his fathers age = 6 + 32
years = 38 years, his mothers age = 38 3 = 35 years, his sisters age = 35
25 = 10 years

9. All but nine died means All except nine died i.e. 9 sheep remained alive.
10. Total number of digits = (Number of digits in 1 digit page numbers +
number of digits in 2 digit page numbers + number of digits in 3 digit
page numbers
= (1 9 + 2 90 + 3 267) = (9 + 180 + 801) = 990
11. Suppose the boy got x sums right and 2x sums wrong.
Then, x + 2x = 48
3x = 48
x = 16
12. Let the fathers age be x and the sons age be y.
Then, x y = y or x = 2y
Now, x = 36, So, 2y = 36 or y = 18
Therefore Sons present age = 18 years
So, sons age 5 years ago = 13 years
13. Let the number be x. Then x + 13x = 112
14x = 112
x=8
14. Total number of handshakes = 9 + 8 + 7 + 6 + 5 + 4 + 3 + 2 + 1 = 45
15. Out of every 16 persons, there is one captain, so, number of captains
1200/16 = 75
16. Since the number of carbons is 2, only two copies can be obtained.
17. Seven pieces consist of 6 smaller equal pieces and one half cake piece.
Weight of each small piece = 20 gm
So, total weight of the cake = 2 (20 6) = 240 gm
18. Let number of girls = x and number of boys = 3x
Then, 3x + x = 4x = total number of students
Thus, to find exact value of x, the total number of students must be divisible
by 4
19. Option B
208

REASONING MADE EASY www.BankExamsToday.com


20.
21.
22.
23.
24.
25.

Option C
Option D
Option C
Option A
Option C
Option B

209

REASONING MADE EASY www.BankExamsToday.com

Chapter 18
Classification
Directions (Q. 1-5): In each of the following questions, five words have been given
out of which four are alike in some manne, while 5th one is different. Choose the
word which is different from the rest.
1. Choose the word which is different from the rest
a) Chicken
b) Snake
d) Crocodile
e) Frog

c) Swan

2. Choose the word which is different from the rest


a) Cap
b) Turban
d) Veil
e) Hat

c) Helmet

3. Choose the word which is different from the rest


a) Kiwi
b) Eagle
d) Ostrich

c) Emu

4. Choose the word which is different from the rest


a) Rigveda
b) Yajurveda
d) Ayurveda
e) Samveda

c) Atharvaveda

5. Choose the word which is different from the rest


a) Curd
b) Butter
d) Cheese
e) Cream

c) Oil

Directions (Q. 6-10): Out of the five figures marked (1), (2), (3), (4) and (5), four
are similar in a certain manner. However, one figure is not like the other four.
Choose the figure which is different from the rest.
6. Choose the figure which is different from the rest.

(1)
a) 1
d) 4

(2)

(3)

(4)

(5)
b) 2
e) 5

c) 3

7. Choose the figure which is different from the rest.

(1)
a) 1

(2)

(3)

(4)

(5)
b) 2

c) 3
210

REASONING MADE EASY www.BankExamsToday.com


d) 4

e) 5

8. Choose the figure which is different from the rest.

(1)
a) 1
d) 4

(2)

(3)

(4)

(5)
b) 2
e) 5

c) 3

9. Choose the figure which is different from the rest.

(1)
a) 1
d) 4

(2)

(3)

(4)

(5)
b) 2
e) 5

c) 3

10. Choose the figure which is different from the rest.

(1)
a) 1
d) 4

(2)

(3)

(4)

(5)
b) 2
e) 5

c) 3

Directions (Q. 11-15): In each of the following questions, five words have been
given out of which four are alike in some manner, while 5th one is different. Choose
the word which is different from the rest.
11. Choose the word which is different from the rest
a) Pear
b) Apple
d) Guava
e) Orange
12. Choose the word which is different from the rest
a) Dagger
b) Hammer
d) Sword
e) Blade

c) Litchi

c) Knife

13. Choose the word which is different from the rest


a) Kanpur
b) Allahabad
d) Mathura

c) Varanasi

14. Choose the word which is different from the rest


a) Oyster
b) Clam
d) Mussel

c) Scallop

15. Choose the word which is different from the rest


211

REASONING MADE EASY www.BankExamsToday.com


a) Deck
d) Bow

b) Quay
e) Mast

c) Stern

Directions (Q. 16-20): Out of the five figures marked (1), (2), (3), (4) and (5), four
are similar in a certain manner. However, one figure is not like the other four.
Choose the figure which is different from the rest.
16. Choose the figure which is different from the rest.

(1)
a) 1
d) 4

(2)

(3)

(4)

(5)
b) 2
e) 5

c) 3

17. Choose the figure which is different from the rest.

(1)
a) 1
d) 4

(2)

(3)

(4)

(5)
b) 2
e) 5

c) 3

18. Choose the figure which is different from the rest.

(1)
a) 1
d) 4

(2)

(3)

(4)

(5)
b) 2
e) 5

c) 3

19. Choose the figure which is different from the rest.

(1)
a) 1
d) 4

(2)

(3)

(4)

(5)
b) 2
e) 5

c) 3

20. Choose the figure which is different from the rest.

(1)
a) 1
d) 4

(2)

(3)

(4)

(5)
b) 2
e) 5

c) 3

212

REASONING MADE EASY www.BankExamsToday.com


Answers:
1.
2.
3.
4.
5.
6.
7.
8.
9.
10.
11.
12.
13.
14.
15.
16.
17.
18.
19.
20.

Option A
Option D
Option B
Option D
Option C
Option A
Option E
Option D
Option B
Option B
Option E
Option B
Option D
Option C
Option B
Option D
Option C
Option C
Option D
Option C

213

REASONING MADE EASY www.BankExamsToday.com

Chapter 19
Dice Problem
1. Which symbol will be on the face opposite to the face with symbol *

a) @
d) +

b) $
e) None of these

c) 8

2. Two positions of a dice are shown below. When number 1 on the top.
What number will be at the bottom?
a) 3
b) 5
c) 2
d) 6
e) None of these
3. From the four positions of a dice given below, find the colour which is
opposite to yellow?
a) Violet
b) Red
c) Rose
d) Blue
e) None of these
4. From the positions of a cube are shown below, which letter will be on the
face opposite to face with A?
a) D
b) B
c) C
d) F
e) None of these
5. Which digit will appear on the face opposite to the face with number 4?

a) 3
d) 2/3

b) 5
e) None of these

c) 6

6. Which number is on the face opposite to 6?

a) 4
d) 3

b) 1
e) None of these

c) 2
214

REASONING MADE EASY www.BankExamsToday.com

7. When the digit 5 is on the bottom then which number will be on its upper
surface?

a) 1
d) 6

b) 3
e) None of these

c) 4

8. Observe the dots on the dice (one to six dots) in the following figures. How
many dots are contained on the face opposite to the containing four dots?

a) 2
d) 6

b) 3
e) None of these

c) 5

9. Four usual dice are thrown on the ground. The total of number on the top
faces of these four dice is 13 as the top faces showed 4, 3, 1 and 5
respectively. What is the total of the faces touching the ground?
a) 12
b) 13
c) 15
d) Cant be determined e) None of these
10. A dice is numbered from 1 to 6 in different ways.
If 1 is opposite to 5 and 2 is opposite to 3, then
a) 4 is adjacent to 3 and 6
b) 2 is adjacent to 4 and 6
c) 4 is adjacent to 5 and 6
d) 6 is adjacent to 3 and 4
11. Which symbol will be on the face opposite to the face with symbol C?

a) D
d) F

b) E
e) None of these

c) B or D

12. Which number is on the face opposite to 56?


215

REASONING MADE EASY www.BankExamsToday.com

a) 54
b) 55
d) Cant be determined e) None of these

c) 50

13. If there are five dots at the bottom, then how many dots will be on its upper
surface?

a) 1
d) 4

b) 2
e) None of these

c) 3

14. Two positions of dice are shown below. What will be on the top when B
is at bottom?

a) F
d) E

b) D
e) None of these

c) A

15. Two positions of dice are shown below. What number will appear on the
opposite to the face containing 5?

a) 2
d) 6

b) 4
e) None of these

c) 1

216

REASONING MADE EASY www.BankExamsToday.com

16. Two positions of a cube with surfaces are shown below. When the surface
D touch the bottom, what surface will be on top?

a) D
d) F

b) A
e) None of these

c) B

17. Two positions of a dice are shown below. When C is at the bottom then
what is on top?

a) A
d) D

b) B
e) None of these

c) C

18. Which digit will appear on the face opposite to the face with alphabet d?

a) a
d) f

b) g
e) None of these

c) c

19. Two positions of a cube are shown below. When the surface D touch the
bottom, what surface will be on the top?

217

REASONING MADE EASY www.BankExamsToday.com


a) B
b) C
d) Cant be determined e) None of these

c) E

20. How many dot/dots will appear on the face opposite to the face with Four
Dots?

a) 3 dots
d) 5 dots

b) 2 dots
e) None of these

c) 1 dot

Answers:
1. Option C
The symbols of the adjacent faces to the face with symbol * are @, -, + and
$. Hence the required symbol is 8.
2. Option B
According to the rule (2) when one is at the top, then 5 will be at the
bottom.
3. Option A
The colours adjacent to yellow are orange, blue, red and rose. Hence violet
will be opposite to yellow.
4. Option A
The letters of the adjacent faces to the face with letter A, are B, F, C and E.
Hence D is the letter of the face opposite to the face with letters (A).
5. Option A
Here the common faces with number 3, are in same positions. Hence 6 is
opposite to 2 and 5 is opposite to 1. Therefore 4 is opposite to 3.
6. Option B
As the numbers 2, 3, 4 and 5 are adjacent to 6. Hence the number on the
face opposite to 6 is 1.
7. Option A
According to the rule No.3, common faces with number 2 are in same
positions. Hence when the digit 5 is on the bottom then 1 will on the upper
surface.
218

REASONING MADE EASY www.BankExamsToday.com


8. Option A
9. Option C
In a usual dice, the sum of the numbers on any two opposite faces is always
7. Thus, 1 is opposite 6, 2 is opposite 5 and 3 is opposite 4. Consequently,
when 4, 3, 1 and 5 are the numbers on the top faces, then 3, 4, 6 and 2
respectively are the numbers on the face touching the ground. The total of
these numbers = 3 + 4 + 6 + 2 = 15
10. Option B
If 1 is opposite to 5 and 2 is opposite to 3, then 4 definitely lies opposite to
6. Therefore, 2 cannot lie opposite to any of the two numbers 4 or 6.
Hence, 2 necessarily lies adjacent to both 4 and 6.
11.
12.
13.
14.
15.
16.
17.
18.
19.
20.

Option D
Option E
Option A
Option B
Option B
Option B
Option E
Option C
Option E
Option A

219

REASONING MADE EASY www.BankExamsToday.com

Chapter 20
Statement and Arguments
Directions (Q. 1-5): Each question given below consists of a statement, followed
by two arguments numbered I and II. You have to decide which of the arguments is
a strong argument and which is a weak argument.
Give answer:
a) If only argument I is strong
b) If only argument II is strong
c) If either I or II is strong
d) If neither I nor II is strong
e) If both I and II are strong
1. Statement: Should India encourage exports, when most things are
insufficient for internal use itself?
Arguments:
I.
Yes, we have to earn foreignexchange to pay for our imports.
II.
No, even selective encouragement would lead to shortages.
a) Only argument I is strong
b) Only argument II is strong
c) Either I or II is strong
d) Neither I nor II is strong
e) Both I and II are strong
2. Statement: Should all the drugs patented and manufactured in Western
countries be first tried out on sample basis before giving licence for sale to
general public in India?
Arguments:
I.
Yes, many such drugs require different doses and duration for
Indian population and hence it is necessary.
II.
No, this is just not feasible and hence cannot be implemented.
a) Only argument I is strong
b) Only argument II is strong
c) Either I or II is strong
d) Neither I nor II is strong
e) Both I and II are strong
3. Statement: Should India make efforts to harness solar energy to fulfil its
energy requirements?
Arguments:
I.
Yes, most of the energy sources used at present is exhaustible.
II.
No, harnessing solar energy requires a lot of capital, which India
lacks in.
220

REASONING MADE EASY www.BankExamsToday.com


a) Only argument I is strong
b) Only argument II is strong
c) Either I or II is strong
d) Neither I nor II is strong
e) Both I and II are strong
4. Statement: Should there be students union in college/university?
Arguments:
I.
No, this will create a political atmosphere in the campus.
II.
Yes, it is very necessary students are future political leaders.
a) Only argument I is strong
b) Only argument II is strong
c) Either I or II is strong
d) Neither I nor II is strong
e) Both I and II are strong
5. Statement: Should India give away Kashmir to Pakistan?
Arguments:
I.
No, Kashmir is a beautiful State. It earns a lot of foreign exchange
for India.
II.
Yes, this would help settle conflicts.
a) Only argument I is strong
b) Only argument II is strong
c) Either I or II is strong
d) Neither I nor II is strong
e) Both I and II are strong
Directions (Q. 6-10): Each question given below consists of a statement, followed
by two arguments numbered I and II. You have to decide which of the arguments is
a strong argument and which is a weak argument.
Give answer:
a) If only argument I is strong
b) If only argument II is strong
c) If either I or II is strong
d) If neither I nor II is strong
e) If both I and II are strong
6. Statement: Should there be an upper age limit of 65 years for contesting
Parliamentary/Legislative Assembly elections?
Arguments:
I.
Yes, generally people above the age of 65 lose their dynamism and
will power.
II.
No, the life span is so increased that people remain physically and
mentally active even upto the age of 80.
a) Only argument I is strong
221

REASONING MADE EASY www.BankExamsToday.com


b)
c)
d)
e)

Only argument II is strong


Either I or II is strong
Neither I nor II is strong
Both I and II are strong

7. Statement: Should new big industries be started in Mumbai?


Arguments:
I.
Yes, it will create job opportunities.
II.
No, it will further add to pollution of the city.
a) Only argument I is strong
b) Only argument II is strong
c) Either I or II is strong
d) Neither I nor II is strong
e) Both I and II are strong
8. Statement: Should high chimneys be installed in industries?
Arguments:
I.
Yes, it reduces pollution at ground level.
II.
No, it increases pollution in upper atmosphere.
a)
Only argument I is strong
b)
Only argument II is strong
c)
Either I or II is strong
d)
Neither I nor II is strong
e)
Both I and II are strong
9. Statement: Does India need so many plans for development?
Arguments:
I.
Yes, nothing can be achieved without proper planning.
II.
No, too much time, money and energy is wasted on planning.
a) Only argument I is strong
b) Only argument II is strong
c) Either I or II is strong
d) Neither I nor II is strong
e) Both I and II are strong
10. Statement: Should articles of only deserving authors be allowed to be
published?
Arguments:
I.
Yes, it will save a lot of paper which is in short reply.
II.
No, it is not possible to draw a line between the deserving and the
undeserving.
a) Only argument I is strong
b) Only argument II is strong
c) Either I or II is strong
d) Neither I nor II is strong
e) Both I and II are strong
222

REASONING MADE EASY www.BankExamsToday.com

Directions (Q. 11-15): Each question given below consists of a statement, followed
by two arguments numbered I and II. You have to decide which of the arguments is
a strong argument and which is a wweak argument.
Give answer:
a) If only argument I is strong
b) If only argument II is strong
c) If either I or II is strong
d) If neither I nor II is strong
e) If both I and II are strong
11. Statement: Should all the unauthorized structures in the city be
demolished?
Arguments:
I.
No, where will the people residing in such houses live?
II.
Yes, this will give a clear message to general public and they will
refrain from constructing unauthorized buildings.
a) Only argument I is strong
b) Only argument II is strong
c) Either I or II is strong
d) Neither I nor II is strong
e) Both I and II are strong
12. Statement: Should there be a maximum limit for the number of ministers
in the Central Government?
Arguments:
I.
No, the political party in power should have the freedom to decide
the number of ministers to be appointed.
II.
Yes, the number of ministers should be restricted to a certain
percentage of the total number of seats in the Parliament to avoid
unncessary expenditure.
a) Only argument I is strong
b) Only argument II is strong
c) Either I or II is strong
d) Neither I nor II is strong
e) Both I and II are strong
13. Statement: Should foreign films be banned in India?
Arguments:
I.
Yes, they depict an alien culture which adversely affects our values.
II.
No, foreign films are of a high artistic standard.
a) Only argument I is strong
b) Only argument II is strong
c) Either I or II is strong
d) Neither I nor II is strong
223

REASONING MADE EASY www.BankExamsToday.com


e) Both I and II are strong
14. Statement: Is buying things on installments profitable to the customer?
Arguments:
I.
Yes, he has to pay less.
II.
No, paying installments upsets the family budget.
a) Only argument I is strong
b) Only argument II is strong
c) Either I or II is strong
d) Neither I nor II is strong
e) Both I and II are strong
15. Statement: Should Doordarshan be given autonomous status?
Arguments:
I.
Yes, it will help Doordarshan to have fair and impartial coverage of
all important events.
II.
No, the coverage of events will be decided by a few who may not
have healthy outlook.
a) Only argument I is strong
b) Only argument II is strong
c) Either I or II is strong
d) Neither I nor II is strong
e) Both I and II are strong
Directions (Q. 16-20): Each question given below consists of a statement, followed
by two arguments numbered I and II. You have to decide which of the arguments is
a strong argument and which is a weak argument.
Give answer:
a) If only argument I is strong
b) If only argument II is strong
c) If either I or II is strong
d) If neither I nor II is strong
e) If both I and II are strong
16. Statement: Should Government jobs in rural areas have more incentives?
Arguments:
I.
Yes, incentives are essential for attracting government servants
there.
II.
No, rural areas are already cheaper, healthier and less complex than
big cities. So? Why offer extra incentives.
a) Only argument I is strong
b) Only argument II is strong
c) Either I or II is strong
d) Neither I nor II is strong
e) Both I and II are strong
224

REASONING MADE EASY www.BankExamsToday.com

17. Statement: Should there be a cap on maximum number of contestants for


Parliamentary elections in any constituency?
Arguments:
I.
Yes, this will make the Parliamentary elections more meaningful as
the voters can make a considered judgment for casting their vote.
II.
No, in a democracy any person fulfilling the eligibility criteria can
contest Parliamentary elections and there should be no restrictions.
a) Only argument I is strong
b) Only argument II is strong
c) Either I or II is strong
d) Neither I nor II is strong
e) Both I and II are strong
18. Statement: Should so much money be spent on advertisements?
Arguments:
I.
Yes, it is an essential concomitant in a capitalist economy.
II.
No, it leads to wastage of resources.
a) Only argument I is strong
b) Only argument II is strong
c) Either I or II is strong
d) Neither I nor II is strong
e) Both I and II are strong
19. Statement: Should all the legislators be forced to resign from their
profession?
Arguments:
I.
Yes, they will be able to devote more time for the country.
II.
No, nobody will contest election.
a) Only argument I is strong
b) Only argument II is strong
c) Either I or II is strong
d) Neither I nor II is strong
e) Both I and II are strong
20. Statement: Should computer knowledge be made a compulsory subject for
all the students at secondary school level?
Arguments:
I.
No, our need is bread for everyone, we cannot follow western
models.
II.
Yes, we cannot compete in the international market without
equipping our children with computers.
a) Only argument I is strong
b) Only argument II is strong
c) Either I or II is strong
d) Neither I nor II is strong
225

REASONING MADE EASY www.BankExamsToday.com


e) Both I and II are strong
Directions (Q. 21-25): Each question given below consists of a statement, followed
by two arguments numbered I and II. You have to decide which of the arguments is
a strong argument and which is a weak argument.
Give answer:
a) If only argument I is strong
b) If only argument II is strong
c) If either I or II is strong
d) If neither I nor II is strong
e) If both I and II are strong
21. Statement: Should religion be banned?
Argument:
I.
Yes, it develops fanaticism in people.
II.
No, religion binds people together.
a) Only argument I is strong
b) Only argument II is strong
c) Either I or II is strong
d) Neither I nor II is strong
e) Both I and II are strong
22. Statement: Should India become a permanent member of UNs Security
Council?
Arguments:
I.
Yes, India has emerged as a country which loves peace and amity.
II.
No, let us first solve problems of our own people like poverty,
malnutrition.
a) Only argument I is strong
b) Only argument II is strong
c) Either I or II is strong
d) Neither I nor II is strong
e) Both I and II are strong
23. Statement: Should fashionable dresses be banned?
Arguments:
I.
Yes, fashions keep changing and hence consumption of cloth
increases.
II.
No, fashionable clothes are a pesons self expression and therefore
his/her fundamental right.
a) Only argument I is strong
b) Only argument II is strong
c) Either I or II is strong
d) Neither I nor II is strong
e) Both I and II are strong
226

REASONING MADE EASY www.BankExamsToday.com

24. Statement: Should an organization like UNO be dissolved?


Arguments:
I.
Yes, with cold war coming to an end, such organizations have no
role to play.
II.
No, in the absence of such organizations there may be a World
War.
a) Only argument I is strong
b) Only argument II is strong
c) Either I or II is strong
d) Neither I nor II is strong
e) Both I and II are strong
25. Statement: Should there be no place of interview in selection?
Arguments:
I.
Yes, it is very subjective in assessment.
II.
No, it is the only instrument to judge candidates motives and
personality.
a) Only argument I is strong
b) Only argument II is strong
c) Either I or II is strong
d) Neither I nor II is strong
e) Both I and II are strong
Directions (Q. 26-30): Each question given below consists of a statement, followed
by two arguments numbered I and II. You have to decide which of the arguments is
a strong argument and which is a weak argument.
Give answer:
a) If only argument I is strong
b) If only argument II is strong
c) If either I or II is strong
d) If neither I nor II is strong
e) If both I and II are strong
26. Statement: Should children be legally made responsible to take care of
their parents during their old age?
Arguments:
I.
Yes, such matter can only be solved by legal means.
II.
Yes, only this will bring some relief to poor parents.
a) Only argument I is strong
b) Only argument II is strong
c) Either I or II is strong
d) Neither I nor II is strong
e) Both I and II are strong
227

REASONING MADE EASY www.BankExamsToday.com


27. Statement: Should there be reservation in Government jobs for candidates
from single child family?
Arguments:
I.
No, this is not advisable as the jobs should be offered to only
deserving candidates without any reservation for a particular group.
II.
Yes, this will help reduce the growing population in India as the
parents will be encouraged to adopt single child norm.
a) Only argument I is strong
b) Only argument II is strong
c) Either I or II is strong
d) Neither I nor II is strong
e) Both I and II are strong
28. Statement: Should higher education be completely stopped for some time?
Arguments:
I.
No, it will hamper the countrys future progress.
II.
Yes, it will reduce the educated unemployment.
a) Only argument I is strong
b) Only argument II is strong
c) Either I or II is strong
d) Neither I nor II is strong
e) Both I and II are strong

29. Statement: Should we scrap the Public Distribution System in India?


Arguments:
I.
Yes, protectionism is over, everyone must get the bread on his/her
own.
II.
Yes, the poor do not get any benefit because of corruption.
a) Only argument I is strong
b) Only argument II is strong
c) Either I or II is strong
d) Neither I nor II is strong
e) Both I and II are strong
30. Statement: Should India have no military force at all?
Arguments:
I.
No, other countries in the World do not believe in non-violence.
II.
Yes, many Indian believe in non-violence.
a) Only argument I is strong
b) Only argument II is strong
c) Either I or II is strong
d) Neither I nor II is strong
e) Both I and II are strong

228

REASONING MADE EASY www.BankExamsToday.com


Directions (Q. 31-35): Each question given below consists of a statement, followed
by three or four arguments numbered I, II, III and IV. You have to decide which of
the arguments is/are strong arguments and which is/are weak arguments and
accordingly choose your answer from the alternatives given below each question.
31. Statement: Should people with educational qualification higher than the
optimum requirements be debarred from seking jobs?
Arguments:
I.
No, it will further aggrevate the problem of educated
unemployment.
II.
Yes, it creates complexes among employees and affects the work
adversely.
III.
No, this goes against the basic rights of the individuals.
IV.
Yes, this will increase productivity.
a) Only argument I is strong
b) Only argument II is strong
c) Either I or II is strong
d) Neither I nor II is strong
e) Both I and II are strong
32. Statement: Should India go in for computerization in all possible sectors?
Arguments:
I.
Yes, it will bring efficiency and accuracy in the work.
II.
No, it will be an injustice to the monumental human resources
which are at present underutilized.
III.
No, computerization demands a lot of money. We should not waste
money on it.
IV.
Yes, when advanced countries are introducing computers in every
field, how can India afford to lag behind?
a) Only argument I is strong
b) Only argument II is strong
c) Either I or II is strong
d) Neither I nor II is strong
e) Both I and II are strong
33. Statement: Should all the school teachers be debarred from giving private
tuitions?
Arguments:
I.
No, the needy students will be deprived of the expertise of these
teachers.
II.
Yes, this is an injustice to the unemployed educated people who
can earn their living by giving tuitions.
III.
Yes, only then the quality of teaching in schools will improve.
IV.
Yes, now salary of these teachers is reasonable.
a) Only argument I is strong
b) Only argument II is strong
229

REASONING MADE EASY www.BankExamsToday.com


c) Either I or II is strong
d) Neither I nor II is strong
e) Both I and II are strong
34. Statement: Should education be made compulsory for all children upto the
age of 14?
Arguments:
I.
Yes, this will help to eradicate the system of forced employment of
these children.
II.
Yes, this is an effective way to make the entire population
educated.
III.
No, we do not have adequate infrastructure to educate the entire
population.
IV.
Yes, this would increase the standard of living.
a) Only argument I is strong
b) Only argument II is strong
c) Either I or II is strong
d) Neither I nor II is strong
e) Both I and II are strong
35. Statement: Should trade unions be banned completely?
Arguments:
I.
Yes, workers can concentrate on production.
II.
No, this is the only way through which employees can put their
demands before the management.
III.
Yes, employees get their illegal demands fulfilled through these
unions.
IV.
No, trade unions are not banned in other economically advanced
countries.
a) Only argument I is strong
b) Only argument II is strong
c) Either I or II is strong
d) Neither I nor II is strong
e) Both I and II are strong

Answers:
1. Option A
Clearly, India can export only the surplus and that which can be saved after
fulfilling its own needs, to pay for its imports. Encouragement to export
cannot lead to shortages as it shall provide the resources for imports. So,
only argument I holds.
2. Option A
230

REASONING MADE EASY www.BankExamsToday.com


Clearly, health of the citizens is an issue of major concern for the
Government. So, a product like drugs, must be first studied and tested in the
Indian context before giving licence for its sale. So, only argument I holds
strong.
3. Option A
4. Option E
The students union formation shall be a step towards giving to students the
basic education in the field of politics. However, it shall create the same
political atmosphere in the campus. Thus, both the arguments hold strong.

5. Option A
India cannot part with a State that is a major foreign exchange earner to it.
So, argument I holds strong. Further, giving away a piece of land
unconditionally and unreasonably is no solution to settle disputes. So,
argument II is vague.
6. Option D
The age of a person is no criterion for judging his mental capabilities and
administrative qualities. So, none of the arguments holds strong.
7. Option C
Opening up of new industries is advantageous in opening more employment
avenues, and disadvantageous in that it adds to the pollution. So, either of
the arguments holds strong.
8. Option A
Pollution at ground level is the most hazardous in the way of being
injurious to human and animal life. So, argument I alone holds.
9. Option A
Before indulging in new development programme it is much necessary to
plan the exact target, policies and their implementation and the allocation of
funds which shows the right direction to work. So, argument I holds strong.
Also, planning ensures full utilization of available resources and funds and
stepwise approach towards the target. So, spending a part of money on it is
no wastage. Thus, argument II is not valid.
10. Option B
I does not provide a strong reason in support of the statement. Also, it is not
possible to analyze the reaally deserving and not deserving. So, argument II
holds strong.
11. Option B
231

REASONING MADE EASY www.BankExamsToday.com


The demolition of unauthorized buildings would teach a lesson to the
unscrupulous builders and also serve as a warning for the citizens not to
indulge in such activities in the future. This is essential, as unauthorized
constructions impose undue burden on the citys infrastructure. So, only
argument II holds strong.
12. Option B
There should be some norms regarding the number of ministers in the
Government, as more number of ministers would unnecessarily add to the
Government expenditure. So, argument II holds strong. Also, giving liberty
to the party in power could promote extension of unreasonable favour to
some people at the cost of Government funds. So, argument I does not hold.
13. Option D
Foreign films depict the alien culture but this only helps in learning more.
So, argument I does not hold. Also, the reason stated in argument II is not
strong enough in contradicting the ban. So, it also does not hold.
14. Option D
In buying things on installments, a customer has to pay more as the interest
is also included. So, argument I does not hold. Moreover, one who buys an
item on installments maintains his future budget accordingly as he is well
acquainted with when and how much he has to pay, beforehand. So,
argument II is also not valid.
15. Option A
16. Option A
Government jobs in rural areas are underlined with several difficulties. In
lieu of these, extra incentives are needed. So, only argument I holds strong.
17. Option E
If there were less candidates, the voters would find it easy to make a choice.
So, argument I holds. Also, every person satisfying the conditions laid
down by the Constitution must be given an opportunity and should not be
denied the same just to cut down the number of candidates. So, argument II
also holds strong.
18. Option A
The advertisements are/the means to introduce people with the produce and
its advantages. So, arguments I holds strong. But argument II is vague
because advertisements are an investment for better gain and not a wastage.
19. Option A
The legislators should surely not be engaged in any other profession
because only then will they be able to work with devotion. So, argument I
holds. Also, if such a law is enforced, only those people will contest
232

REASONING MADE EASY www.BankExamsToday.com


elections who are really preparred to work for the country. So, argument II
is vague.
20. Option B
Now a days, computers have entered all walks of life and children need to
be prepared for the same. So, argument II is strong. Argument I holds no
relevance.
21. Option C
Religion binds people togethe through the name of God and human values.
But at the same time it may create differences and ill-will people, So, either
of the arguments holds strong.
22. Option A
A peace loving nation like India can well join an international forum which
seeks to bring different nations on friendly terms with each other. So,
argument I holds strong. Argument II highlights a different aspect. The
internal peoblems of a nation should not debar it from strengthening
international ties. So, argument II is vague.
23. Option B
Imposing ban on fashionable dresses will be a restriction on the pesonal
choice and hence the right to freedom of an individual. So, only argument II
is strong.
24. Option B
An organization like UNO is meant to maintain peace all over and will
always serve to prevent conflicts between countries. So, its role never ends.
So, argument I does not hold. Also, lack of such an organization may in
future lead to increased mutual conflicts and international wars, on account
of lack of a common platform for mutual discussions. So, argument II
holds.
25. Option A
Besides interview, there can be other modes of written examination to judge
candidates motives. So argument II is not strong enough. However, the
interview is a subjective assessment without doubt. So, argument I holds.
26. Option D
Taking care of the parents is a moral duty of the children and cannot be
thrust upon them legally, nor such a compulsion can ensure good care of the
old people, So, none of the arguments holds strong.
27. Option D
The Government has already made provisions for reservation of jobs for the
economically backward sections, which is a must. So, abolishing the
practice of reservation altogether has no meaning. Thus, argument I is
233

REASONING MADE EASY www.BankExamsToday.com


vague. Also, more reservations would lead to non-recruitment of many
more deserving candidates. Besides, such a reservation, if implemented,
will cater to the job requirements of only a small section of population and
not a major part of it. So, argument II also does not hold strong.
28. Option A
Higher education is not the cause of unemployment. Infact, it has created
greater job opportunities. So, argument II is vague. Also, higher education
promotes the countrys development. So, argument I holds.
29. Option D
The Public Distribution System is indeed necessary to provide basic
amenities to the economically backward sections of population. So,
argument I is vague. Also, if the objectives, of a system are not fulfilled
because of corruption, then getting rid of the system is no solution. Instead,
efforts should be made to end corruption and extend its benefits to the
people for whom it is meant. So, argument II also does not hold.
30. Option D
India needs to have military force to defend itself against the threat of other
military powers in the World. So, none of the arguments holds strong.
31. Option D
The issue discussed in the statement is nowhere related to increase in
unemployment, as the number of vacancies filled in will remain the same.
Also, in a working place, it is the peformance of the individual that matters
and that makes him more or less wanted, and not his educational
qualifications. So, neither I nor II holds strong. Besides, the needs of a job
are laid down in the desired qualifications for the job. So, recruitment of
more qualified people cannot augment productivity. Thus, IV also does not
hold strong. However, it is the right of an individual to get the post for
which he fulfils the eligibility criteria, whatever be his extra merits. Hence,
argument III holds strong.
32. Option A
The need of today is to put to better use the underutilized human resources.
Computers with better and speedy efficiency can accomplish this. So,
argument I holds, while II does not. Computerization is a much beneficial
project and investment in it is not at all a waste. So, III is not strong.
Further, development in a new field is not a matter of merely following up
other countries. So, IV also does not hold strong.
33. Option E
Only III is strong. The lure of earning private tuitions reduces the efforts
and devotion of the teachers towards the students in schools. So, if tuitions
are banned, students can benefit from their teachers knowledge in the
234

REASONING MADE EASY www.BankExamsToday.com


school itself. So, argument III holds strong while I does not. However, a
person cannot be barred from earning more just because he already has a
good salary. So, argument IV is vague. Further, the unemployed people
thriving on tuitions can survive with the school teachers holding tuitions
too, if they are capable enough to guide the students well. So, argument II
also does not hold strong.
34. Option D
Todays children are to make up future citizens of the country and so it is
absolutely essential to make them learned, more responsible, more
innovative and self-dependent by imparting them education. So, argument
II holds strong while I and IV do not. Besides, the goal of literacy cannot be
denied for want of infrastructure. So, argument III also does not hold.
35. Option B
Trade unions provide a common platform for the workers to voice their
demands and protests and thus ensure that they are not subdued or
exploited. So, argument II holds strong, while I and III do not, besides, the
idea of imitation of other countries in the implementation of a certain policy
holds no relevance. So, argument IV also does not hold strong.

235

REASONING MADE EASY www.BankExamsToday.com

Chapter 21
Statement and Assumptions
Directions (Q. 1-5): In each question below is given a statement followed by two
assumptions numbered I and II. You have to consider the statement and the
following assumptions and decide which of the assumptions is implicit in the
statement.
Give answer:
a) If only assumption I is implicit
b) If only assumption II is implicit
c) If either I or II is implicit
d) If neither I nor II is implicit
e) If both I and II are implicit
1. Statement: You are hereby appointed as a programmer with a probation
period of one year and your performance will be reviewed at the end of the
period for confirmation.
Assumptions:
I.
The performance of an individual generally is not known at the
time of appointment offer.
II.
Generally an individual tries to prove his worth in the probation
period.
a) Only assumption I is implicit
b) Only assumption II is implicit
c) Either I or II is implicit
d) Neither I nor II is implicit
e) Both I and II are implicit
2. Statement: It is desirable to put the child in school at the age of 5 or so.
Assumptions:
I.
At that age the child reaches appropriate level of development and
is ready to learn.
II.
The schools do not admit children after six years of age.
a) Only assumption I is implicit
b) Only assumption II is implicit
c) Either I or II is implicit
d) Neither I nor II is implicit
a) Both I and II are implicit
3. Statement: In order to bring punctuality in our office, we must provide
conveyance allowance to our employees. In charge of a company tells
Personnel Manager.
Assumptions:
236

REASONING MADE EASY www.BankExamsToday.com


I.
II.

Conveyance allowance will not help in bringing punctuality.


Discipline and reward should always go hand in hand.
a) Only assumption I is implicit
b) Only assumption II is implicit
c) Either I or II is implicit
d) Neither I nor II is implicit
a) Both I and II are implicit

4. Statement: Unemployment allowance should be given to all unemployed


Indian youth above 18 years of age.
Assumptions:
I.
There are unemployed youth in India who needs monetary support.
II.
The Government has sufficient funds to provide allowance to all
unemployed youth.
a) Only assumption I is implicit
b) Only assumption II is implicit
c) Either I or II is implicit
d) Neither I nor II is implicit
a) Both I and II are implicit
5. Statement: If you trouble me, I will slap you. A mother warns her child.
Assumptions:
I.
With the warning, the child may stop troubling her.
II.
All children are basically naughty.
a) Only assumption I is implicit
b) Only assumption II is implicit
c) Either I or II is implicit
d) Neither I nor II is implicit
a) Both I and II are implicit
Directions (Q. 6-10): In each question below is given a statement followed by two
assumptions numbered I and II. You have to consider the statement and the
following assumptions and decide which of the assumptions is implicit in the
statement.
Give answer:
a) If only assumption I is implicit
b) If only assumption II is implicit
c) If either I or II is implicit
d) If neither I nor II is implicit
e) If both I and II are implicit
6. Statement: The Government has decided to levy 2% on the tax amount
payable for funding drought relief programmes.
Assumptions:
237

REASONING MADE EASY www.BankExamsToday.com


I.
II.

The Government does not have sufficient money to fund drought


relief programmes.
The amount collected by way of surcharge may be adequate to fund
the drought relief programes.
a) Only assumption I is implicit
b) Only assumption II is implicit
c) Either I or II is implicit
d) Neither I nor II is implicit
e) Both I and II are implicit

7. Statement: Detergents should be used to clean clothes.


Assumptions:
I.
Detergents form more lather.
II.
Detergents help to dislodge grease and dirt.
a) Only assumption I is implicit
b) Only assumption II is implicit
c) Either I or II is implicit
d) Neither I nor II is implicit
e) Both I and II are implicit
8. Statement: It will be a substantial achievement in the field of education if
one provides one school for every village in our country and enforce
attendance.
Assumptions:
I.
Children in villages do not attend school regularly.
II.
Providing school to every village is desirable.
a) Only assumption I is implicit
b) Only assumption II is implicit
c) Either I or II is implicit
d) Neither I nor II is implicit
e) Both I and II are implicit
9. Statement: The Government has decided to disinvest large chunk of its
equity in select public sector undertaking for a better fiscal management.
Assumptions:
I.
The amount generated out of the disinvestment process may reduce
substantially the mounting fiscal deficits.
II.
There will be enough demand in the market for the shares of these
undertakings.
a) Only assumption I is implicit
b) Only assumption II is implicit
c) Either I or II is implicit
d) Neither I nor II is implicit
e) Both I and II are implicit
10. Statement: Never before such a lucid book was available on the topic.
238

REASONING MADE EASY www.BankExamsToday.com


Assumptions:
I.
Some other books were available onthis topic.
II.
You can write lucid books on very few topics.
a) Only assumption I is implicit
b) Only assumption II is implicit
c) Either I or II is implicit
d) Neither I nor II is implicit
e) Both I and II are implicit
Directions (Q. 11-15): In each question below is given a statement followed by two
assumptions numbered I and II. You have to consider the statement and the
following assumptions and decide which of the assumptions is implicit in the
statement.
Give answer:
a) If only assumption I is implicit
b) If only assumption II is implicit
c) If either I or II is implicit
d) If neither I nor II is implicit
e) If both I and II are implicit
11. Statement: The Government has decided to pay compensation to the tune
of Rs.1 Lakh to the family membes of those who are killed in railway
accidents.
Assumptions:
I.
The Government has enough funds to meet the expenses due to
compensation.
II.
There may be reduction in incidents of railway accidents in near
future.
a) Only assumption I is implicit
b) Only assumption II is implicit
c) Either I or II is implicit
d) Neither I nor II is implicit
e) Both I and II are implicit
12. Statement: Films have become indispensable for the entertainment of
people.
Assumptions:
I.
Films are the only media of entertainment.
II.
People enjoy films.
a) Only assumption I is implicit
b) Only assumption II is implicit
c) Either I or II is implicit
d) Neither I nor II is implicit
e) Both I and II are implicit
239

REASONING MADE EASY www.BankExamsToday.com


13. Statement: Of all the newspapers published in Mumbai, readership of the
Times is the largest in the Metropolis.
Assumptions:
I.
Times is not popular in mofussil areas.
II.
Times has the popular feature of cartoons on burning social and
political issues.
a) Only assumption I is implicit
b) Only assumption II is implicit
c) Either I or II is implicit
d) Neither I nor II is implicit
e) Both I and II are implicit
14. Statement: Apart from the entertainment value of television, its
educational value cannot be ignored.
Assumptions:
I.
People take television to be a means of entertainment only.
II.
The educational value of television is not realised properly.
a) Only assumption I is implicit
b) Only assumption II is implicit
c) Either I or II is implicit
d) Neither I nor II is implicit
e) Both I and II are implicit
15. Statement: Children are influenced more by their teachers nowadays.
Assumptions:
I.
The children consider teachers as their models.
II.
A large amount of childrens time is spent in school.
a) Only assumption I is implicit
b) Only assumption II is implicit
c) Either I or II is implicit
d) Neither I nor II is implicit
e) Both I and II are implicit
Directions (Q. 16-20): In each question below is given a statement followed by two
assumptions numbered I and II. You have to consider the statement and the
following assumptions and decide which of the assumptions is implicit in the
statement.
Give answer:
a) If only assumption I is implicit
b) If only assumption II is implicit
c) If either I or II is implicit
d) If neither I nor II is implicit
e) If both I and II are implicit

240

REASONING MADE EASY www.BankExamsToday.com


16. Statement: As advice to B Go to Jammu via Amritsar The shortest
route.
Assumptions:
I.
B wishes to go to Jammu.
II.
A gives advice to everybody.
a) Only assumption I is implicit
b) Only assumption II is implicit
c) Either I or II is implicit
d) Neither I nor II is implicit
e) Both I and II are implicit
17. Statement: All existing inequalities can be reduced, if not utterly
eradicated, action of governments or by revolutionary change of
government.
Assumptions:
I.
Inequality is a manmade phenomenon.
II.
No person would voluntarily part with what he possesses.
a) Only assumption I is implicit
b) Only assumption II is implicit
c) Either I or II is implicit
d) Neither I nor II is implicit
e) Both I and II are implicit
18. Statement: The campaign of Keep Your City Clean started by the Civil
Council did not evoke any response from the citizens.
Assumptions:
I.
People do not desire to keep their city clean.
II.
The Civil Council has failed in its campaign.
a) Only assumption I is implicit
b) Only assumption II is implicit
c) Either I or II is implicit
d) Neither I nor II is implicit
e) Both I and II are implicit
19. Statement: The district administration has issued a circular to all the
farmers under its jurisdiction advising them for not using pesticides
indiscriminately as it may pollute the ground water.
Assumptions:
I.
People may stop using ground water if the farmers continue to use
pesticides indiscriminately.
II.
Farmers may refrain from using pesticides indiscriminately.
a) Only assumption I is implicit
b) Only assumption II is implicit
c) Either I or II is implicit
d) Neither I nor II is implicit
e) Both I and II are implicit
241

REASONING MADE EASY www.BankExamsToday.com

20. Statement: The coffee powder of company X is quite better in taste than
the much advertised coffee of company Y.
Assumptions:
I.
If your product is not good, your spend more on advertisement.
II.
Some people are tempted to buy a product by the advertisement.
a) Only assumption I is implicit
b) Only assumption II is implicit
c) Either I or II is implicit
d) Neither I nor II is implicit
e) Both I and II are implicit
Directions (Q. 21-25): In each question below is given a statement followed by two
assumptions numbered I and II. You have to consider the statement and the
following assumptions and decide which of the assumptions is implicit in the
statement.
Give answer:
a) If only assumption I is implicit
b) If only assumption II is implicit
c) If either I or II is implicit
d) If neither I nor II is implicit
e) If both I and II are implicit
21. Statement: Because of the large number of potholes in road X, reaching
airport in time has become difficult.
Assumptions:
I.
Reaching airport in time may not be always necessary.
II.
There is no other convenient road to the airport.
a) Only assumption I is implicit
b) Only assumption II is implicit
c) Either I or II is implicit
d) Neither I nor II is implicit
e) Both I and II are implicit
22. Statement: Safety and health practices in many Indian companies are well
below the international standards.
Assumptions:
I.
International standards of health and safety are ideal and unrealistic.
II.
Indian organizations do not consider safety and health management
as their prime social responsibility.
a) Only assumption I is implicit
b) Only assumption II is implicit
c) Either I or II is implicit
d) Neither I nor II is implicit
e) Both I and II are implicit
242

REASONING MADE EASY www.BankExamsToday.com

23. Statement: Greater public participation results in good civic governance.


Statement of Municipal Commissioner of City A.
Assumptions:
I.
The municipal office is not competent to effect good civic
administration.
II.
Good civic governance is a matter of collective will and effort of
the people and administration.
a) Only assumption I is implicit
b) Only assumption II is implicit
c) Either I or II is implicit
d) Neither I nor II is implicit
e) Both I and II are implicit
24. Statement: The regulatory authority has set up a review committee tofind
out the reasons for unstable stock prices.
Assumptions:
I.
The investors may regain confidence in stock market by this
decision.
II.
The review committee has the expertise to find out the causes for
volatility in the stock market.
a) Only assumption I is implicit
b) Only assumption II is implicit
c) Either I or II is implicit
d) Neither I nor II is implicit
e) Both I and II are implicit
25. Statement: Please note that the company will provide accommodation to
only outside candidates if selected. A condition in an advertisement.
Assumptions:
I.
The local candidates would be having some other arrangement for
their stay.
II.
The company plans to select only local candidates.
a) Only assumption I is implicit
b) Only assumption II is implicit
c) Either I or II is implicit
d) Neither I nor II is implicit
e) Both I and II are implicit
Directions (Q. 26-30): In each question below is given a statement followed by two
assumptions numbered I and II. You have to consider the statement and the
following assumptions and decide which of the assumptions is implicit in the
statement.
Give answer:
a) If only assumption I is implicit
243

REASONING MADE EASY www.BankExamsToday.com


b)
c)
d)
e)

If only assumption II is implicit


If either I or II is implicit
If neither I nor II is implicit
If both I and II are implicit

26. Statement: Many historians have done more harm than good by distorting
truth.
Assumptions:
I.
People believe what is reported by the historians.
II.
Historians are seldom expected to depict the truth.
In each question below is given a statement followed by two assumptions numbered
I and II. You have to consider the statement and the following assumptions and
decide which of the assumptions is implicit in the statement.
Give answer:
a) If only assumption I is implicit
b) If only assumption II is implicit
c) If either I or II is implicit
d) If neither I nor II is implicit
e) If both I and II are implicit
27. Statement: As there is a great demand, every person seeking tickets of the
programme will be given five tickets.
Assumptions:
I.
The organizers are not keen on selling the tickets.
II.
No one is interested in getting more than five tickets.
a) Only assumption I is implicit
b) Only assumption II is implicit
c) Either I or II is implicit
d) Neither I nor II is implicit
e) Both I and II are implicit
28. Statement Computer education should start at schools itself.
Assummptions:
I.
Learning computers is easy.
II.
Computer education fetches jobs easily.
a) Only assumption I is implicit
b) Only assumption II is implicit
c) Either I or II is implicit
d) Neither I nor II is implicit
e) Both I and II are implicit
29. Statement: If he is intellegent, he will pass the examination.
Assumptions:
I.
To pass, he must be intelligent.
II.
He will pass the examination.
244

REASONING MADE EASY www.BankExamsToday.com


a)
b)
c)
d)
e)

Only assumption I is implicit


Only assumption II is implicit
Either I or II is implicit
Neither I nor II is implicit
Both I and II are implicit

30. Statement: Today I must satisfy myself only by looking at a pink headed
duck in an encyclopaedia.
Assumptions:
I.
Pink headed ducks are as good as extinct now.
II.
People refer to encyclopaedia to know only about things which do
not exist now.
a) Only assumption I is implicit
b) Only assumption II is implicit
c) Either I or II is implicit
d) Neither I nor II is implicit
e) Both I and II are implicit
Answers:
1. Option E
The performance of the individual has to be tested over a span of time as
the statement mentions. So, I is implicit. The statement mentions that the
individuals worth shall be reviewed before confirmation. So, II is also
implicit.
2. Option A
Since the statement talks of putting the child in school at the age of 5, it
means that the child is mentally prepared for the same at this age. So, I is
implicit, But nothing about admission after 6 years of age is mentioned in
the statement. So, II is not implicit.
3. Option B
Assumption I goes against the statement. So, it is not implicit. The
allowance will serve as a reward to the employees and shall provoke them
to come on time. So, II is implicit.
4. Option A
I directly follows from the statement and so is implicit. Also, the statement
is a suggestion and does not tell about a Government policy or its position
of funds. So, II is not implicit.
5. Option A
The mothe warns her child with the expectation that he would stop
troubling her. So, I is implicit. The general nature of children cannot be
derived from the statement. So, II is not implicit.
245

REASONING MADE EASY www.BankExamsToday.com

6. Option E
Since a surcharge has been levied to fund drought relief programmes, it
follow the Government does not have sufficient money for the same. So, I
is implicit. Besides, the percentage of surcharge must have been decided
after studying the expected inflow in relation to amount of funds required.
So, II is also implicit.
7. Option B
Nothing is mentioned about lather formation by the detergent. So, I is not
implicit. Also, detergents should be used as they clean clothes better and
more easily. So, II is implicit.
8. Option E
The statement lays stress on enforcing attendance. This implies that
children in villages do not attend school regularly. So, I is implicit. Besides,
the statement calls one school for every village a substantial achievement.
So, II is also implicit.
9. Option A
The fact given in I directly follows from the phrase -------- for a better fiscal
management in the statement. So, I is implicit. However, the public
response to the new policy cannot be ascertained. So, II is not implicit.
10. Option A
It follows from the statement that books on this topic were available before
also but they were not lucid. So, I is implicit. But a general comment as II
cannot be made from the given statement. So, II is not implicit.
11. Option A
The amount of compensation must have been decided keeping in mind the
monetary position of the Government. So, I is implicit. However, nothing
can be said about the frequency of railway accidents in future. So, II is not
implicit.
12. Option B
Films are indispensable does not mean that they are the only means of
entertainment. So, I is not implicit. II follows from the statement. So, it is
implicit.
13. Option D
Neither the volume of readership of the Times in areas other than the
Metropolis nor the reason for its huge acclamation is evident from the
statement. So, neither I nor II is implicit.
14. Option E
246

REASONING MADE EASY www.BankExamsToday.com


The statement makes the first assumption clear though educational value is
not to be ignored. So, I is implicit. That the educational value must not be
ignored also shows that educational value is not realised properly. So, II is
also implicit.
15. Option A
It is because children consider teachers as their model that they are more
influenced by them. So, I is implicit. It is not necessary that the children are
influenced by teachers because they spend much time in school. So, II is not
implicit.
16. Option A
A has advised B the route to Jammu. This means that B wishes to go to
Jammu. So, I is implicit. The statement mentions only As advice to B. So,
II is not implicit.
17. Option A
Since inequality can be reduced, it means that it is not natural but created.
So, I is implicit. Nothing is mentioned about peoples response. So, II is not
implicit.
18. Option E
According to the statement, the campaign did not get any response from
citizens. This means that people are not interested in keeping the city clean
and the campaign has failed. So, both I and II implicit.
19. Option B
The district administration has issued a circular to make the farmers aware
of hazards that indiscriminate use of pesticides poses to ground water and
plead them to refrain from the same. So, II is implicit. However, I cannot be
assumed from the given statement and so it is not implicit.
20. Option B
21. Option B
The statement presents the issue of not reaching airport in time as a
problem. This means that reaching airport in time is necessary. So, I is not
implicit. Besides, it is mentioned that reaching airport in time has become
difficult due to large number of potholes in road X. This implies that road X
is the only possible way. So, II is implicit.
22. Option B
The statement talks about the safety and health practices in Indian
companies being far below international standards. It is clearly a criticism
of Indian organizations not paying considerable attention to these aspects.
247

REASONING MADE EASY www.BankExamsToday.com


So, II is implicit. The intenationalstandards demand perfection and are, in
no way, non-achievable. So, I is not implicit.
23. Option B
The statement stresses on the fact that though civic governance is the task
of the municipal body, but all the tasks done come ou to be more fruitful if
the general public lends a helping hand in the same. So, only II is implicit.
24. Option E
I mentions the aim for which the step talked about in the statement, has
been undertaken while II mentions the essential requirement for it. So, both
I and II are implicit.
25. Option A
The statement mentions that the company intends to provide
accommodation only to outside candidates. This means that local
candidates would have to arrange accommodation on their own and that the
company may select local as well as outside candidates. Thus, only I is
implicit.
26. Option A
The fact that historians have done harm by distorting truth, means that
people believe what is reported by the historians. So, I is implicit. II does
not follow from the statement and is not implicit.
27. Option D
The organizers are adopting this policy not to reduce the sale but to cope up
with great demand so that everyone can get the ticket. So, I is not implicit.
Also, due to great demand, the maximum number of tickets one person can
get has been reduced to five So, II is also not implicit.
28. Option A
Computer educabe started at the school level only if it is easy. So, I is
implicit. In the statement, nothing is mentioned about the link between jobs
and computer education. So, II is not implicit.
29. Option A
The statement mentions that he will pass if he is intelligent. So, I is implicit.
Further, this means that it is not necessary that he will pass. So, II is not
implicit.
30. Since the narrator talks ofsatisfying himself by just looking at a picture in
encyclopaedia, it means that pink headed ducks are not to be seen alive. So,
I is implicit. But II does not follow from the statement and is not implicit.

248

REASONING MADE EASY www.BankExamsToday.com

Chapter 22
Statement and Conclusions
Directions (Q. 1-5): In each question below is given a statement followed by two
conclusions numbered I and II. You have to assume everything in the statement to
be true, then consider the two conclusions together and decide which of them
logically follows beyond a reasonable doubt from the information given in the
statement.
Give answer:
a) If only conclusion I follows
b) If only conclusion II follows
c) If either I or II follows
d) If neither I nor II follows and
e) If both I and II follow
1. Statement: In a one day cricket match, the total runs made by a team were
200. Out of these 160 runs were made by spinners.
Conclusions:
I.
80% of the team consists of spinners.
II.
The opening batsmen were spinners.
a) Only conclusion I follows
b) Only conclusion II follows
c) Either I or II follows
d) Neither I nor II follows
e) Both I and II are follow
2. Statement: The old order changed place to new.
Conclusions:
I.
Change is the law of nature.
II.
Discard old ideas because they are old.
a) Only conclusion I follows
b) Only conclusion II follows
c) Either I or II follows
d) Neither I nor II follows
a) Both I and II are follow
3. Statement: Government has spoiled many top ranking financial institutions
by appointing bureaucrats as Directors of these institutions.
Conclusions:
I.
Government should appoint Directors of the financial institutes
taking into consideration the expertise of the person in the area of
finance.
249

REASONING MADE EASY www.BankExamsToday.com


II.

The Director of the financial institute should have expertise


commensurate with the financial work carried out by the institute.
a) Only conclusion I follows
b) Only conclusion II follows
c) Either I or II follows
d) Neither I nor II follows
a) Both I and II are follow

4. Statement: Population increase coupled with depleting resources is going


to be the scenario of many developing countries in days to come.
Conclusions:
I.
The population of developing countries will not continue to
increase in future.
II.
It will be very difficult for the governments of developing countries
to provide its people decent quality of life.
a) Only conclusion I follows
b) Only conclusion II follows
c) Either I or II follows
d) Neither I nor II follows
a) Both I and II are follow
5. Statement: Prime age school going children in urban India have now
become avid as well as more regular viewers of television, even in
households without a T.V. As a result, there has been an alarming decline in
the extent of readership of newspapers.
Conclusions:
I.
Method of increasing the readership of newspapes should be
devised.
II.
A team of experts should be sent to other countries to study the
impact of T.V. on the readership of newspapers.
a) Only conclusion I follows
b) Only conclusion II follows
c) Either I or II follows
d) Neither I nor II follows
a) Both I and II are follow
Directions (Q. 6-10): In each question below is given a statement followed by two
conclusions numbered I and II. You have to assume everything in the statement to
be true, then consider the two conclusions together and decide which of them
logically follows beyond a reasonable doubt from the information given in the
statement.
Give answer:
a) If only conclusion I follows
b) If only conclusion II follows
c) If either I or II follows
250

REASONING MADE EASY www.BankExamsToday.com


d) If neither I nor II follows and
e) If both I and II follow
6. Statement: The manager humiliated Sachin in the presence of his
colleagues.
Conclusions:
I.
The manager did not like Sachin.
II.
Sachin was not popular with his colleagues.
a) Only conclusion I follows
b) Only conclusion II follows
c) Either I or II follows
d) Neither I nor II follows
a) Both I and II are follow
7. Statement: Womens organizations in India have welcomed the
amendment of the Industrial Employment Rules 1946 to curb sexual
harassment at the work place.
Conclusions:
I.
Sexual harassment of women at work place is more prevalent in
India as compared to other developed countries.
II.
Many organizations in India will stop recruiting women to avoid
such problems.
a) Only conclusion I follows
b) Only conclusion II follows
c) Either I or II follows
d) Neither I nor II follows
a) Both I and II are follow
8. Statement: Nation X faced growing international opposition for its
decision to explode eight nuclear weapons at its test site.
Conclusions:
I.
The citizens of the nation favoured the decision.
II.
Some powerful countries do not want other nations to become as
powerful as they are.
a) Only conclusion I follows
b) Only conclusion II follows
c) Either I or II follows
d) Neither I nor II follows
a) Both I and II are follow
9. Statement: In a highly centralised power structure, in which even senior
cabinet ministers are prepared to reduce themselves to pathetic countries or
yesmen airing views that are primarily intended to anticipate or reflect the
Prime Ministers own performances, there can be no place for any
consensus that is quite different from real or contrived unanimity of
251

REASONING MADE EASY www.BankExamsToday.com


opinion, expressed through a well orchestrated endorsement of the leaders
actions.
Conclusions:
I.
The Ministers play safe by not giving anti-government views.
II.
The Prime Minister does not encourage his colleagues to render
their own views.
a) Only conclusion I follows
b) Only conclusion II follows
c) Either I or II follows
d) Neither I nor II follows
a) Both I and II are follow
10. Statement: National Aluminium Company has moved India from a
position of shortage to self sufficiency in the metal.
Conclusions:
I.
Previously, India had to import aluminium.
II.
With this speed, it can soon become a foreign exchange earner.
a) Only conclusion I follows
b) Only conclusion II follows
c) Either I or II follows
d) Neither I nor II follows
a) Both I and II are follow
Directions (Q. 11-15): In each question below is given a statement followed by two
conclusions numbered I and II. You have to assume everything in the statement to
be true, then consider the two conclusions together and decide which of them
logically follows beyond a reasonable doubt from the information given in the
statement.
Give answer:
a) If only conclusion I follows
b) If only conclusion II follows
c) If either I or II follows
d) If neither I nor II follows and
e) If both I and II follow
11. Statement: People who speak too much against dowry are those who had
taken it themselves.
Conclusions:
I.
It is easier said than done.
II.
People have double standards.
a) Only conclusion I follows
b) Only conclusion II follows
c) Either I or II follows
d) Neither I nor II follows
a) Both I and II are follow
252

REASONING MADE EASY www.BankExamsToday.com

12. Statement: The national norm is 100 beds per thousand populations but in
this state, 150 beds per thousand are available in the hospitals.
Conclusions:
I.
Our national norm is appropriate.
II.
The states health system is taking adequate care in this regard.
a) Only conclusion I follows
b) Only conclusion II follows
c) Either I or II follows
d) Neither I nor II follows
a) Both I and II are follow
13. Statement: Our securities carry market risk. Consult your investment
advisor or agent before investing.
Conclusions:
I.
One should not invest in securities.
II.
The investment advisor calculates the market risk with certainty.
a) Only conclusion I follows
b) Only conclusion II follows
c) Either I or II follows
d) Neither I nor II follows
a) Both I and II are follow
14. Statement: Money plays a vital role in politics.
Conclusions:
I.
The poor can never become politicians.
II.
All the rich men take part in politics.
a) Only conclusion I follows
b) Only conclusion II follows
c) Either I or II follows
d) Neither I nor II follows
a) Both I and II are follow
15. Statement: Vegetable prices are soaring in the market.
Conclusions:
I.
Vegetables are becoming a rare commodity.
II.
People cannot eat vegetables.
a) Only conclusion I follows
b) Only conclusion II follows
c) Either I or II follows
d) Neither I nor II follows
a) Both I and II are follow
Directions (Q. 16-20): In each question below is given a statement followed by two
conclusions numbered I and II. You have to assume everything in the statement to
be true, then consider the two conclusions together and decide which of them
253

REASONING MADE EASY www.BankExamsToday.com


logically follows beyond a reasonable doubt from the information given in the
statement.
Give answer:
a) If only conclusion I follows
b) If only conclusion II follows
c) If either I or II follows
d) If neither I nor II follows and
e) If both I and II follow
16. Statement: Any student who does not behave properly while in the school
brings bad name to himself and also for the school.
Conclusions:
I.
Such student should be removed from the school.
II.
Stricter discipline does not improve behaviour of the students.
a) Only conclusion I follows
b) Only conclusion II follows
c) Either I or II follows
d) Neither I nor II follows
a) Both I and II are follow
17. Statement: A Corporate General Manager asked four managers to either
submit their resignations by the next day or face termination orders from
service. Three of them had submitted their resignations by that evening.
Conclusions:
I.
The next day, the remaining manager would also resign.
II.
The General Manager would terminate his services the next day.
a) Only conclusion I follows
b) Only conclusion II follows
c) Either I or II follows
d) Neither I nor II follows
a) Both I and II are follow
18. Statement: Only good singers are invited in the conference. No one
without sweet voice is a good singer.
Conclusions:
I.
All invited singers in the conference have sweet voice.
II.
Those singers who do not have sweet voice are not invited in the
conference.
a) Only conclusion I follows
b) Only conclusion II follows
c) Either I or II follows
d) Neither I nor II follows
e) Both I and II are follow

254

REASONING MADE EASY www.BankExamsToday.com


19. Statement: To cultivate interest in reading, the school has made it
compulsory from June this year for each student to read two books per
week and submit a weekly report on the books.
Conclusions:
I.
Interest in reading can be created by force.
II.
Some students will eventually develop interest in reading.
a) Only conclusion I follows
b) Only conclusion II follows
c) Either I or II follows
d) Neither I nor II follows
e) Both I and II are follow
20. Statement: Applications of applicants who do no fulfil eligibility criteria
and/or who do not submit applications before last date will be summarily
rejected and will not be called for the written test.
Conclusions:
I.
Those who are called for the written test are those who fulfil
eligibility criteria and have submitted their applications before last
date.
II.
Written test will be held only after scrutiny of applications.
a) Only conclusion I follows
b) Only conclusion II follows
c) Either I or II follows
d) Neither I nor II follows
e) Both I and II are follow
Answers:
1. Option D
According to the statement, 80% of the total runs were made by spinners.
So, I does not follow. Nothing about the opening batsmen is mentioned in
the statement. So, II also does not follow.
2. Option A
I directly follows from the given statement. Also, it is mentioned that old
ideas are replaced by new ones, as thinking changes with the progressing
time. So, II does not follow.
3. Option E
According to the statement, Government has spoiled financial institutions
by appointing bureacrats as Directors. This means that only those persons
should be appointed as Directors who are experts in finance and are
acquainted with the financial work of the institute. So, both I and II follow.
4. Option B
255

REASONING MADE EASY www.BankExamsToday.com


The fact given in I is quite contrary to the given statement. So, I does not
follow. II mentions the direct implications of the state discussed in the
statement. Thus, II follows.
5. Option D
The statement concentrates on the increasing viewership of T.V. and does
not stress either on increasing the readership of newspapers or making
studies regarding the same. So, neither I nor II follows.

6. Option D
The manager might have humiliated Sachin not because of his dislike but
on account of certain negligence or mistake on his part. So, I does not
follow. Also, nothing about Sachins rapport with his colleagues can be
reduced from the statement. So, II also does not follow.
7. Option D
The fact that a certain rule has been more welcomed in a certain coutry does
not imply that the problem is more prevalent there. So, I does not follow.
Also, the amendment seeks to discourage only sexual harassment of women
and shall in no way discourage employment of women. So, II also does not
follow.
8. Option D
Neither the citizens response to the decision nor the reason for opposition
by other nations can be deduced from the statement. So, neither I nor II
follows.
9. Option A
According to the statement, even senior cabinet ministers are always ready
to conform to the Prime Ministers views. So, I follows. However, II
contradicts the given statement and so does not follow.
10. Option E
According to the statement, National Aluminium Company has moved
India from a position of shortage in the past to self sufficiency in the
present. This means that previously, India had to import aluminium. So, I
follows. Also, it can be deduced that if production increases at the same
rate, India can export it in future. So, II also follows.
11. Option E
The statement implies that it is easier to say than to do something and what
people say is different from what they do. So, both I and II follow.
12. Option B
256

REASONING MADE EASY www.BankExamsToday.com


Whether the national norm is appropriate or not cannot be said. So, I does
not follow. However, more numbe of beds per thousand population are
available in the state. So, II follows.
13. Option B
Investment in securities involves risk. This does not mean that one should
not invest in securities. So, I does not follow. Since the statement advises
one to consult investment advisor before investing. So, II follows.
14. Option D
Neither the poor nor the rich, but only the role of money in politics is being
talked about in the statement. So, neither I nor II follows.
15. Option D
The availability of vegetables is not mentioned in the given statement. So, I
does not follow. II is not directly related to the statement and so it also does
not follow.
16. Option D
I cannot be deduced from the statement, Also, nothing about discipline is
mentioned in the statement. So, neither I nor II follows.
17. Option C
It is mentioned in the statement that either the managers should resign by
the next day or their services would be terminated. So, either I or II follows.
18. Option E
The statement asserts that a good singer always has a sweet voice and only
good singers are invited in the conference. This implies that all those
invited in the conference have sweet voice and those who do not have sweet
voice are not invited. So, both I and II follow.
19. Option B
The new scheme intends to develop interest in reading by incorporating the
habit in their routine. So, only II follows while I does not.
20. Option E
The statement mentions that fulfilling the eligibility criteria and submitting
the application before the stipulated date are both essential to avoid
rejection. So, I follows. Also, since it is given that the candidates whose
applications are rejected shall not be called for written test, so II also
follows.

257

Вам также может понравиться